Sunteți pe pagina 1din 184

175

CAPTULO 7:
Movimientos lineales bsicos.
A esta altura tenemos las herramientas necesarias para analizar cualquier movimiento, y
ya hemos visto algunos ejemplos interesantes. Este es el momento de analizar
especficamente detalles de los movimientos ms simples y tpicos, tanto para conocerlos
como para lograr una comprensin mejor y ms profunda sobre el significado de estas
leyes.

7.1.- Discusin general bsica.


Vimos en algunos ejemplos cmo la fuerza resultante a lo largo del movimiento hace variar la
rapidez del mismo, mientras que la fuerza resultante en la direccin transversal determina la
curvatura de la trayectoria.
Apoyndonos en la independencia de las acciones en las diferentes direcciones del espacio,
juntaremos todos los elementos para tratar cualquier caso general.
Denominaremos tangencial, e indicaremos con el subndice T, a la direccin del movimiento
en cada instante, es decir a la direccin de la tangente a la trayectoria en el punto
correspondiente, y normal, o transversal, (indicada con el subndice N) a la direccin
perpendicular a la anterior en cada instante.
Las figuras 7.1, 7.2, y 7.3, ilustran varios casos. En ellas, se elegi un intervalo supuesto de
duracin suficientemente corta como para que la fuerza resultante sea ms o menos constante
en l. Dicha fuerza resultante est indicada con un vector hueco, y descompuesta segn las
direcciones tangencial ( T ) y normal ( N ). En cada figura se agrega un diagrama de
cantidades de movimiento correspondiente al intervalo considerado, que ilustra la aplicacin
de la Ley del Impulso a la situacin.

eje N

p1
F
FN p0 I

eje T p1 Diagrama de cantida-
p0 FT des de movimiento
Fig. 7.1 : Caso de un intervalo con componente tangencial de la fuerza resultante en el
mismo sentido del movimiento. La rapidez aumenta. Adems se muestra una componente
normal de la fuerza desviando el movimiento hacia la izquierda.
176


FT p1 p1
T

p2
p2
FN I
F N
Diagrama de cantidades
de movimiento
Fig. 7.2 : Caso de un intervalo con componente tangencial de la fuerza resultante en
sentido opuesto al movimiento. La rapidez disminuye. Adems se muestra una
componente normal de la fuerza desviando el movimiento hacia la derecha.


p2
p1 t2
I0;1
I1;2

t1 p1

p0

p2
p0
Diagrama de cantidades
t0 de movimiento

Fig. 7.3: Caso en que la trayectoria se recorre uniformemente. Slo existe fuerza resultante
normal actuando de manera de lograr la curvatura o desviacin del movimiento

En todos los casos, la trayectoria se curva hacia donde apunta la componente normal de la
fuerza aplicada. El vector impulso (representado tambin con una flecha hueca en cada
diagrama de cantidades de movimiento) tiene tamao proporcional y exactamente la misma
direccin y sentido, que la fuerza resultante dibujada sobre la trayectoria.

La ley del impulso para las fuerzas tangenciales


Si adoptamos el criterio (arbitrario pero usual y conveniente) de elegir como positivo el
sentido del movimiento, podremos hablar siempre de pT y vT positivos, es decir que pT ser

igual al mdulo de p , y correspondientemente vT ser lo mismo que el mdulo de v (recordar

que v y p no tienen componente normal).
Con esta convencin nunca tendremos el caso de pT o vT negativos, y consideraremos
positivas las fuerzas tangenciales hacia adelante, y negativas hacia atrs.
En estas condiciones, la ley del impulso, escrita para la componente tangencial, dir:

FT t = p = m v (7.1)


Expresin que indica directamente lo que sucede con el mdulo de p o de v , a partir de
caractersticas de la componente tangencial de las fuerzas aplicadas.
Por ejemplo digamos que, si FT>0, es decir hacia adelante, entonces p>0, lo que significa

que el mdulo de p , (y tambin de v ), aumenta. Y viceversa, FT<0 significar hacia atrs
177


con respecto al movimiento, y entonces p<0, indicar que el mdulo de p , (y tambin de v ),

disminuye, o sea que FT est frenando al mvil.

La ley del impulso para las fuerzas normales


Consideremos un movimiento en el cual acta una fuerza estrictamente normal, es decir, sin
componente tangencial, de manera que la trayectoria se desva un ngulo en un lapso t,
sin que cambie la rapidez del movimiento.
Ya sabemos que para lograr esto la fuerza debe ir continuamente cambiando de direccin para
mantenerse siempre perpendicular a la trayectoria, y slo falta averiguar el mdulo necesario
para producir cierta desviacin. En la prxima figura analizamos los vectores para un ngulo
, que se supone suficientemente pequeo como para poder ignorar el cambio en la direccin

de F , as obtendremos el mdulo de la fuerza en ese instante; luego la fuerza se mantiene
aplicada con ese mismo mdulo todo el tiempo que sea necesario para completar la desviacin
que se quiera.


eje N

p1

F I = F t

p0

p1
Diagrama de cantida-

p0 des de movimiento

Fig. 7.4: Esquema para calcular la fuerza normal capaz de producir una desviacin.

En la figura 7.4 no se puede mostrar el ngulo tan pequeo como debera ser, pero es posible
imaginarlo, y en esas condiciones, en el diagrama vectorial de la derecha vale:
F t
(7.2)
p
Para entender esta expresin se debe recordar que el valor de un ngulo expresado en radianes
es el cociente del arco sobre el radio, y en el caso de ngulo muy pequeo queda un pequeo
triangulito en el cual el arco es lo mismo que el lado pequeo.
De aqu podemos despejar el valor que debe tener la fuerza para lograr esta desviacin:

Fp =p (7.3)
t t
Si denominamos velocidad angular, , a la desviacin por unidad de tiempo, tenemos otras
expresiones tiles:
F=p=mv (7.3)

En donde = /t, o, en valores medios, /t, es la velocidad angular.


Ms adelante, al estudiar el movimiento circular profundizaremos en estas ideas.
178

NOTA PRCTICA
Ya sabemos que el efecto de las fuerzas tangenciales es independiente de la
presencia o ausencia de las fuerzas normales, y viceversa (Principio de
Independencia de los movimientos). As es que los clculos que hay que hacer
para aplicar la ley del impulso para las fuerzas tangenciales en casos de
trayectorias curvas (en las cuales adems hay fuerza normal resultante) son
exactamente los mismos que se deben hacer en los casos de movimientos
rectilneos , que sern ampliamente ilustrados en prximos puntos.
Por ello en este momento podemos prescindir de ejemplos de aplicacin de la ley
del impulso tangencial, y en cambio s es interesante analizar el siguiente
ejemplo de aplicacin de ley del impulso para las fuerzas normales.

Ejemplo desarrollado
Un automvil de 1000 kg (incluida la masa de los ocupantes) viaja por una carretera rectilnea
horizontal a razn de 20 m/s. En el punto A la carretera se curva suavemente hacia la izquierda, de
manera que 10 m ms adelante, en el punto B, contina en lnea recta en una direccin que forma 5
con la anterior. En todo el proceso el conductor regula la aplicacin de fuerza motriz de manera de
mantener constante el valor de la velocidad en 20 m/s.
En el esquema se muestra la situacin desde arriba.

v0 B
A 5
10 m

Suponga para tener valores aproximados que las fuerzas de rozamiento valen unos 1500 N = 1,50 kN,
redondeando a tres cifras significativas, ya que ms no tendra sentido.
Calcule la fuerza horizontal lateral (hacia la izquierda) que debe aplicar el piso a los neumticos (por
efecto del giro que aplica el conductor al volante) mientras el automvil viaja siguiendo la curva, entre
A y B. Qu sienten los pasajeros? Qu pasa con esa fuerza despus de B, cuando el conductor
mantiene derecha la direccin?
Desarrollo
En el tramo rectilneo de la pista (antes de A), dado que el automvil viaja con velocidad constante,
todas las fuerzas estn equilibradas, y la resultante es nula. En particular, en la direccin vertical
tenemos el peso (P = m g = 9800 N) equilibrado con la reaccin normal del piso, que por lo tanto debe
valer 9,80 kN en adelante no hablaremos ms de las fuerzas verticales, ya que no interesan en estos
planteos. Y en la direccin horizontal, que s interesa, tenemos el rozamiento de 1,50 kN equilibrado
con la fuerza impulsora aplicada por el piso a las ruedas motrices (reaccin a la accin hacia atrs de
estas ruedas sobre el piso) tambin de 1,50 kN.
El equilibrio tambin vale para el tramo posterior a B. Las fuerzas impulsora y la de rozamiento se
mantienen mutuamente opuestas alineadas con la nueva direccin de la trayectoria.
Pero en la curva debe haber una resultante no nula, que debe ser normal a la trayectoria, horizontal,
hacia la izquierda, para lograr la desviacin de la cantidad de movimiento, y debe valer, segn (7.2): FR
= m v /t .
Para calcular debemos expresar la desviacin en radianes: = 5 = 52 / 360 0,0873 rad, y
calcular el tiempo demorado: t = 10 m / 20 (m/s) = 0,50 s.
179

El mdulo de la fuerza resultante, inmediatamente despus de pasar A, y hasta B, debe valer FR


1000 kg20 m/s0,0873 / 0,5 s 3,49 kN.
3,49 kN
3,49 kN 3,49 kN 1,50 kN
1,50 kN
1,50 kN
1,50 kN 1,50 kN 1,50 kN

A B

Los pasajeros sentirn una fuerza (proporcional a la masa de cada uno) con la cual el vehculo les da
un empujn (slo dura 0,5 s) hacia la izquierda este empujn los desva mantenindolos dentro del
vehculo, en la trayectoria curva. Subjetivamente, vindose inmviles dentro del vehculo, los pasajeros
tendrn la sensacin de que una fuerza los lanz contra el lado derecho del mismo (fuerza centrfuga).

7.2.- Movimientos rectilneos


En estos movimientos la trayectoria es una lnea recta que contiene la direccin tangencial y
todos los vectores interesantes para el movimiento, es decir vector desplazamiento, velocidad,
cantidad de movimiento, y fuerza (resultante).
Si arbitrariamente sobre esta lnea ubicamos el eje x (equivalentemente podra elegirse el y),
ganamos enormemente en comodidad ya que en ese caso slo tendremos que considerar la
componente x de todos los vectores que intervienen, pudiendo prescindir de las otras, como
se ve a continuacin.


p

F
m v
r

Fig. 7.5 : eleccin tpica de ejes para un movimiento rectilneo. El vector fuerza se
dibuj arbitrariamente de manera de representar una fuerza que est frenando al
mvil y debe estar claro que es la fuerza resultante de todas las acciones
exteriores en ese instante.
180

Cada vector podra ser descripto con las dos componentes que corresponden, pero al elegir el
eje x sobre la trayectoria, la componente y es siempre nula:

posicin: r=(x;0)

r x ; 0 x
velocidad: v = (vx ; 0 ) ; 0
t t t

cantidad de movimiento: p = ( px ; 0 ) = ( m vx ; 0 )

fuerza (resultante): F = ( Fx ; 0 )

Dado que todas las operaciones que debemos efectuar con vectores (suma, resta, y
multiplicacin por nmeros) se efectan por separado sobre cada componente, entonces
podemos sobreentender la componente y, y escribir todas las expresiones slo para la
componente x. De este modo, sin olvidar que siempre trabajamos con la componente del
vector que corresponda, podremos escribir simplemente funciones.
Se destacan casos tpicos que analizamos en detalle a continuacin.

Movimiento rectilneo uniforme (MRU)


En este caso la fuerza resultante es nula, y el cuerpo mantiene inalterada su cantidad de
movimiento, y con ella su velocidad:
Fx = 0 px = cte vx = x/t, tambin debe ser constante

Para este movimiento slo tenemos expresiones simples. Todas se obtienen a partir de la
definicin de velocidad, considerando que vx se puede calcular con cualquier cociente x/t,
independientemente de que el intevalo de tiempo sea grande o pequeo, porque siempre
arrojar el mismo valor vx .

Es decir que de:


x
vx (7.4)
t
Podremos despejar:
x = vx t (7.5)
Esto se lee diciendo que la distancia recorrida es directamente proporcional al intervalo de
tiempo transcurrido, y la velocidad es la constante de proporcionalidad.
Si queremos expresar la posicin x, en funcin de t, reemplazamos x = x - x0 , y obtenemos,
x = x0 + vx t, en donde, si se toma (arbitrariamente) t0 = 0, entonces t = t, y se tiene la
expresin ms habitual, que caracteriza a una funcin lineal de t:

x = x 0 + vx t (7.6)

En nuestras aplicaciones prcticas es probable que slo necesitemos utilizar la definicin de


velocidad (7.4), y segn el caso, despejar de ella (7.5). Pero sta es una buena ocasin para
familiarizarnos con el manejo de funciones y representaciones grficas, porque al conectarnos
con los conceptos estudiados en Matemtica, esto nos permite ganar claridad en la visin
global de las situaciones, y habilidad para el tratamiento de movimientos ms complicados.
181

Para este caso puede interesarnos mostrar en una grfica como vara la posicin x en funcin
de t, y por ser una funcin lineal, su representacin grfica es una lnea recta1, como se ve en
la figura 7.6.
x
x1
x
x0

t
t
t0 t1
t0 t1
Fig. 7.6 : Funcin x(t) lineal. Los tringulos sombreados, al ser todos semejantes,
sirven para ver que los cocientes x/t tienen igual valor en los distintos instantes:
x1 /t1 = x0 /t0 = x /t .

Esta figura es til porque permite obtener la velocidad, x /t, como pendiente de la grfica.
Como se muestra en la figura, todos los cocientes x /t que se realicen en distintos instantes,
con intervalos grandes o pequeos, dan el mismo valor de velocidad, como corresponde a una
funcin lineal. Si la grfica fuese curva, eso no significara que la trayectoria es curva, sino
que su pendiente, o sea, la velocidad del movimiento, va cambiando, como veremos en otros
movimientos.
Todo esto es informacin que iremos aprendiendo a extraer de las grficas.

NOTA PRCTICA:
Para cualquiera de los tringulos rectngulos sombreados en la figura 5.6, la
pendiente est dada por los cocientes x/t, los cuales tambin definen la
funcin trigonomtrica denominada tangente del ngulo : tg() = x /t.
Esta pendiente se debe calcular con las unidades de cada eje:
[longitud/tiempo] = [velocidad], para obtener la velocidad del mvil con las
unidades correspondientes. No debe calcularse la funcin tg() a partir del
ngulo medido sobre la figura, a menos que se utilice una escala tal que la
unidad de las abscisas (tiempo) tenga exactamente el mismo tamao en la
figura que la de las ordenadas (distancia).

Ejemplo desarrollado.
Obtenga la velocidad y la funcin x(t) a partir de la siguiente grfica.
x (m)

50

10 t
1 2 3 5 8 (s)

1
Algunos textos ms estrictos reservan la denominacin de funcin lineal para aquellas cuyas grficas son rectas que
pasan por el origen, es decir (7.6) con x0 = 0. Nosotros adheriremos al uso ms laxo, y llamaremos lineal a cualquier funcin
del tipo (7.6).
182

Desarrollo
Primeramente agregamos a la figura un tringulo con un t arbitrario y el correspondiente x.
x (m)

50
x = 40 m
x(0) t = 8 s
10 t
1 2 3 5 8 (s)

La velocidad resulta v = pendiente = 40 m / 8 s = 5 m/s; y con ella tenemos la constante de


proporcionalidad. Ahora, para escribir la funcin falta x(0), que es el valor de x en t 0 = 0 s, o sea el
valor de ordenada donde la grfica corta al eje de ordenadas, y lo obtenemos de la figura: x(0) = 20 m.
Entonces:
x(t) = 20 m + 5 (m/s)t

FUNCIONES Y REPRESENTACIONES GRFICAS


sta es una ocasin en la cual el estudio de los movimientos se relaciona con el
estudio de diversas funciones, y conviene revisar alguna nomenclatura de
matemtica, aplicada concretamente a nuestros fines.
Ya hemos dicho que con las coordenadas (x ; y) formamos el vector que nos
indica la posicin de un punto en un sistema de ejes cartesianos que hemos
elegido de referencia.
Si el punto es mvil, entonces la posicin va cambiando, y tenemos que ir
considerando distintas posiciones para cada instante: as, un vector (x1 ; y1)
corresponder al instante t1, otro vector (x2 ; y2) corresponder al instante t2,
etc. Cada componente del vector posicin, a su vez, indica la posicin referida
al eje correspondiente: para el eje x , x1 es la posicin en el instante t1, x2 en t2,
etc, mientras que de manera similar, para el eje y , y1, y2, etc., son las posiciones
en t1, t2, etc.
Es decir que para cada eje la posicin va variando con el tiempo de manera
que define una funcin de t. As, para el eje x, tenemos la funcin posicin x(t),
en la que t es la variable independiente, y x es la dependiente; y para el eje y
tenemos y(t), con y variable dependiente y t siempre variable independiente.
Cada una de estas funciones puede representarse grficamente, cuando
conviene para comprender mejor alguna situacin. Es decir, cualquier variable
se puede representar grficamente en funcin de cualquier otra, pero nosotros
slo haremos algunas representaciones grficas tpicas, y siempre, por razones
fsicas, ser t la variable independiente (desde el punto de vista matemtico
cualquiera de las variables se podra elegir como independiente).
En nuestras aplicaciones, los diagramas con ejes (x ,y) no son representaciones
grficas de funciones, sino que son dibujos que muestran algo que est
ubicado en el espacio. Estos ejes no indican variables que dependen una de
otra.
As, por ejemplo, en un diagrama con ejes (x ,y), dibujamos una recta para
mostrar una trayectoria rectilnea, una curva para mostrar la trayectoria de
una piedra arrojada oblicuamente, y una circunferencia sera la trayectoria
de un punto con movimiento circular.
Ahora bien, estos dibujos no pueden mostrar cmo ocurre el movimiento a
medida que el tiempo transcurre. Para eso recurriremos a las representaciones
183

grficas en funcin del tiempo: colocaremos la variable independiente, t, en


abscisas, y en ordenadas la variable que queramos mostrar cmo depende de t.
Puede ser x(t), y(t), v(t), F(t), Fx(t), etc. Cualquier variable que interese en
determinada situacin, podr ser graficada en funcin del tiempo.
Para ir familiarizndonos de a poco, como hemos dicho, slo graficaremos
algunas pocas cosas de inters.
Hemos comenzado con movimientos que ocurren en el eje x. No interesa
graficar y en funcin del tiempo, ya que y se mantiene constantemente nulo. S
puede interesar graficar la funcin x(t), la cual, para el movimiento simple
que estamos viendo, es una recta.
Esta grfica es una recta porque la funcin x(t) es lineal, y de la grfica
podemos obtener informacin tal como el valor de la velocidad. Aunque la
trayectoria fuese curva, la grfica de la distancia recorrida en funcin del
tiempo sera recta siempre que la velocidad fuese constante.
Y si la grfica no fuese recta, eso no indicara trayectoria curva, sino que
indicara que va cambiando la velocidad.
Debemos reflexionar sobre estas ideas al analizar los ejemplos que se presentan,
aprovechando al mismo tiempo para revisar todas las nociones adquiridas en
Matemtica relacionadas con las representaciones grficas de funciones,
especialmente las nociones de funcin lineal, y pendiente de una grfica.

Ejemplo desarrollado.
Escriba la funcin y(t) de un mvil que viaja uniformemente en lnea recta a lo largo del eje y,
suponiendo que en t0 = 0 s pasa por el lugar y(0) = 60 m, acercndose hacia el origen, al cual llega en
t1 = 6 s. Realice la grfica y(t).
Desarrollo
Para escribir la funcin y(t) debemos averiguar la velocidad. A partir de los datos esto es fcil:
y y1 y 0 0m 60m
v= = = = -60 m / 6 s = -10 m/s;
t t1 t 0 6s 0s
Aunque hemos omitido el subndice, se sobreentiende que hemos calculado vy, la componente y del
vector velocidad.
Dado que ya tenemos el trmino independiente, y(0) = 60 m; entonces la funcin y(t) es
y(t) = 60 m 10 (m/s)t
Para realizar la grfica, podemos trazar una recta que pasa por los dos puntos dados, pero como
ejercicio aqu queremos trazarla a partir de la pendiente.
Para ello tenemos la primer posicin en t0 = 0, es decir marcamos el punto y0 = 60 m en el eje de
ordenadas, y a partir de all, una pendiente de -10 m/s, significa que por cada segundo la ordenada
debe disminuir en 10 m; en 6 segundos se recorren los 60 metros que faltan para el origen (del eje y).
y (m)
t = 1 s
50 y = -10 m

t1
10 t
1 2 3 5 8 (s)
184

Movimiento rectilneo uniformemente variado (MRUV).


Como veremos inmediatamente, ste es el caso que tiene lugar cuando se aplica una fuerza
tangencial constante. Efectivamente en este caso el impulso aplicado es proporcional al
tiempo, y la cantidad de movimiento vara linealmente con el tiempo:
Fx = cte ; I x Fx t
px = p0 + Fx t (7.7)
Dividiendo esta expresin por la masa, se tiene automticamente la expresin para la
velocidad, que tambin debe ser una funcin lineal del tiempo:
vx = v0 + (Fx /m)t (7.8)

Ntese cmo en el caso de este movimiento, la velocidad y no la posicin, es la que vara


linealmente con el tiempo. Ahora nos resulta interesante graficar la velocidad en funcin del
tiempo, aplicando las mismas ideas relacionadas con la funcin lineal. Como vemos a
continuacin, tenemos grficas parecidas a las anteriores, pero con un significado totalmente
diferente.
vx (m/s) vx (m/s)

v0
v0
t t
(s) t* (s)
Caso Fx > 0, v0 > 0 Caso Fx < 0, v0 > 0

Fig. 7.7: La funcin vx(t) es lineal cuando Fx = cte.

Observando la segunda grfica de vx(t), encontramos que: dado que la fuerza se mantiene
orientada hacia los x negativos en todo momento, resulta que, mientras vx > 0 , es decir hasta
t = t*, la fuerza tiene sentido contrario al movimiento y por lo tanto est frenando al mvil.
Precisamente en t = t* es que ste se detiene, y a partir de ese instante la vx es negativa y por
lo tanto la fuerza resulta a favor del movimiento y hace aumentar el valor absoluto de la
velocidad.
Por otra parte, dado que la grfica es una recta, queda claro que el movimiento es simtrico
respecto del instante de la detencin, y esperando un cierto tiempo despus de t*, la velocidad
tiene el mismo valor absoluto que tena el mismo tiempo antes de t*.

La aceleracin

Si recordamos la definicin de aceleracin a v t , segn la Ley del Impulso resulta:

a F m , para este caso a es un vector constante que tambin tiene la misma direccin del
movimiento. Esto significa que en todas las expresiones anteriores se puede reemplazar (F/m)
por a, obteniendo expresiones como:
vx = v0 + a t (7.9)
185

Clculo de la distancia recorrida


Aqu se nos presenta un problema interesante: cmo calculamos la distancia recorrida, o la
posicin, en cada instante?
La posicin es una funcin x(t) tal que vx = x/t, de donde podemos despejar x = vx t.
Si esta expresin fuera vlida para un intervalo cualquiera, no slo para los pequeos,
escribiramos x = vx t, esto dira que x es una funcin lineal de t (esto es lo que hicimos con
el MRU, al pasar de la expresin (7.4) a la (7.5)).
Ahora eso no corresponde, porque vx no es constante: no vale lo mismo al comenzar el
intervalo, al medio, o al final.
La forma de resolver este problema es el mtodo que ya hemos utilizado para calcular el
impulso o el trabajo de una fuerza que no se mantiene constante, y lo desarrollamos a
continuacin para este propsito particular.

La distancia cuando la velocidad va cambiando, y el rea bajo una grfica.


Cuando la velocidad vara no podemos aplicar x = vx t para calcular la distancia recorrida
en un intervalo cualquiera (t0 , t1), suponiendo que ste no es muy pequeo, simplemente
porque no est determinado el valor de la velocidad que habra que utilizar.
No obstante, en un intervalo suficientemente pequeo, siempre vale calcular la distancia
recorrida (que es muy pequea) aplicando x = vx t, si colocamos el valor vx que corresponde
a ese intervalo, prcticamente un instante.
Y ya sabemos que si subdividimos el intervalo total en muchsimos intervalos suficientemente
pequeos, siempre podremos expresar la distancia total recorrida como la suma de todas las
distancias recorridas en cada uno de los intervalos pequeos, aunque no intentaremos hacer
este clculo efectivamente en la prctica, porque si la cantidad de intervalos pequeos es muy
grande, el procedimiento podra resultar tremendamente tedioso, y tal vez hasta imposible.
Es una idea que ya hemos aplicado antes: tenemos el problema de averiguar el resultado de
esta suma de toda una enorme cantidad de pequeas distancias, pero sin hacerla realmente.
Y sabemos que este problema se puede soluionar con la ayuda de las representaciones
grficas.
Consideremos una grfica de la velocidad en funcin del tiempo de un movimiento
cualquiera, como la de la figura 7.8.
vx (m/s)
vx(t1)
cualquier funcin vx(t)
vx(t)

vx(t0)
t
t0 t t1
Fig. 7.8: Si para cualquier intervalo de base t, multiplicamos la base por la altura
vx(t) en algn punto intermedio, obtenemos el rea sombreada, que a la vez debe
ser la distancia x recorrida en ese lapso. Si t es suficientemente pequeo se
puede considerar un instante, y la altura del rectangulito es la velocidad en ese
instante.
186

Si trazamos lneas verticales subdividiendo el intervalo (t0 , t1) en muchos intervalitos de


duracin t suficientemente pequea cada uno, el espacio bajo la grfica, hasta el eje
horizontal, queda subdividido en rectngulos (o trapecios rectangulares) muy angostos, cuya
base, o ancho, es t, y cuya altura es el valor de vx all, en ese el intervalo. Si ahora, para
calcular x, efectuamos el producto vx t, obtenemos el rea de cada rectngulo.
Esto significa que la suma de todas las distancias x recorridas en todos los intervalos, es lo
mismo que la suma de todas las reas de todos estos delgados rectngulos, y eso es lo mismo
que el rea total bajo la grfica.
Es decir, al igual que en ocasiones anteriores, con el concepto de que la distancia total es la
suma de un nmero inmenso de pequeas contribuciones (suma que nunca podramos efectuar
en la prctica), llegamos a la conclusin de que lo que necesitamos es saber calcular el rea de
una figura geomtrica.
De manera que en general, para la distancia recorrida (en x) por un mvil en el intervalo
cualquiera desde t0 hasta t1, con una velocidad dada por cualquier funcin vx(t), siempre vale:

x = rea bajo la grfica, entre t0 y t1

Area est entre comillas porque no es la verdadera rea geomtrica de la figura, sino que se
calcula con las escalas de cada eje, con dimensiones de tiempo en el eje de abscisas, y de
velocidad en el de ordenadas: esta rea resulta con dimensiones de longitud ya que es una
distancia.
vx (m/s)
vx(t1)

x(t1) x(t0) = area sombreada


vx(t0)
t
t0 t1
Fig. 7.9: Para cualquier velocidad vx(t) dada por una grfica, la distancia recorrida
queda determinada por el rea sombreada.

Vale notar que si en vez de hablar de la componente x del vector velocidad, hablamos de su
mdulo, v(t) (suponiendo que lo tenemos graficado en funcin del tiempo), dado que ste se
refiere a la distancia recorrida en el espacio, y no sobre el eje x, ahora tendremos que el rea
de la grfica de v(t), en cualquier intervalo, representa la distancia recorrida en el espacio a
lo largo de la trayectoria.

Velocidad media.
La definicin (2.3) de velocidad media (referida al eje x) vm,x = x / t, segn se discute en el
Captulo 2, es vlida para cualquier movimiento. Esto se interpreta diciendo que la velocidad
media (siempre referida al eje x), vm,x, es aquella que el mvil debera haber mantenido
constante durante todo el intervalo considerado, para recorrer la misma distancia (en el mismo
tiempo).
Dado que ahora sabemos que para cualquier movimiento x es el rea bajo la grfica de la
funcin vx(t), podemos decir:
187

rea bajo v x ( t )
vm,x = (7.10)
t
Esto se puede interpretar en la figura 7.10 viendo que si dibujamos una lnea de altura
(constante) igual a vm,x , queda un rectngulo cuya rea, dada por el producto vm,x t, debe
ser, segn (7.10), igual al rea bajo la grfica de la funcin vx(t).
vx (m/s)
vx(t1) cualquier funcin vx(t)

vm,x rectngulo cuya rea es igual


vx(t0) al rea bajo grfica vx(t)
t
t0 t1
Fig. 7.10: La vm,x define un rectngulo de la misma rea que la figura
entre la vx(t) y el eje de abscisas.

Ahora podemos decir que la velocidad media vm,x, de un movimiento cuya velocidad va
variando segn lo indica la funcin vx(t) cualquiera, en cualquier intervalo t, es la altura de
un rectngulo que tiene la misma rea que queda bajo la curva representativa de la funcin
vx(t).

Distancia recorrida en el MRUV


Luego de estos conceptos generales, podemos volver al problema particular de determinar la
distancia recorrida en un movimiento en el cual la velocidad vara linealmente.
En este caso debemos calcular el rea de tringulos o trapecios.
vx [m/s]
vx(t) rea total =
v clculo por medio de rectngulo + tringulo:
vo t + v t
vo clculo por medio del trapecio:
t (vo + v) t
t0 t t (s)

Fig. 7.11: Se muestran opciones para calcular el rea correspondiente a una grfica lineal de v x(t).

En la figura se muestra que hay ms de una forma de calcular el rea, o sea x (el subndice
x est sobreentendido en todo lugar en donde falte). Todas las formas son equivalentes, pero
unas pueden ser ms cmodas que otras segn sean los datos que se posean.
Las expresiones que ms se utilizan son:
Reemplazando v = a t , en rectngulo + tringulo:
x = v0 t + a (t)2 (7.11)
Expresin en la cual se pone de relieve que x es funcin cuadrtica del tiempo.

Reemplazando t = v / a , en trapecio:
188

x = (v + vo ){(v vo ) / a}

v 2 v02
= (7.12)
2a

O tambin: v2 = v02 +2 a x (7.13)

La expresin para el rea del trapecio tal cual est en la figura:

v0 v
x t = vpromedio t (7.14)
2
En donde vpromedio es el promedio entre la velocidad inicial y la final.
Esta expresin (7.14) nos permite ver que para este movimiento en el que la velocidad vara
uniformemente (no es as en otros), la velocidad vpromedio coincide con x/t, que es la
definicin de velocidad media de cualquier movimiento.
Podra parecer que en esta expresin (7.14) x depende linealmente de t, pero eso no es as, ya
que vpromedio no es constante, sino que tambin aumenta linealmente con el tiempo.

NOTA PRCTICA
Cuando la fuerza se aplica en sentido contrario a la velocidad el mvil se va
frenando, y si la fuerza se mantiene aplicada luego de que el mvil se detiene,
el movimiento se reinicia instantneamente en el sentido de la fuerza, es decir
en sentido contrario al del movimiento inicial. En este caso x no sirve para
saber la distancia recorrida, ya que slo indica la diferencia entre la posicin
inicial y la final.
Si en particular hacemos x = 0 en (7.12) o (7.13), obtenemos v2 = v02. Esto
muestra nuevamente que el movimiento es simtrico respecto del punto de
detencin: si teniendo velocidad v0 en x0, el mvil avanza en contra de la
fuerza hasta una posicin x* en la cual se detiene e invierte la marcha,
cuando luego pasa por la misma coordenada (x1 = x0), lo hace con la misma
velocidad cambiada de signo (v1 = v0).

v1 x(t1) = x(t0) v(t*) = 0


F x
v0 F x*
x(t0)
d = x(t*) x(t0)

Fig. 7.12: Mvil que avanza una distancia d contra la fuerza. sta lo
detiene y luego lo acelera, y al cabo de la distancia d el mvil tiene la
misma velocidad inicial, con signo opuesto. Pero x ya no indica la
distancia recorrida.

Algunas representaciones grficas tpicas, para las diversas condiciones que se indican, son las
siguientes:
189

x(0)=0 x x x(0)=0 x
v(0)=0 v(0)< 0
Fx , ax > 0 F x , ax > 0
t t* t t
t* t*

x x x(0) >0 ; v(0)=0


x(0)>0 x(0)>0 Fx , ax < 0
v(0)>0 v(0)>0
Fx , ax < 0 Fx , ax > 0
t
t* t
t*
Fig. 7.13 : algunos casos de grficas x(t). Ntese que:
El signo de x(0) se refiere a dnde corta la grfica al eje vertical.
El signo de v(0) indica cmo corta la curva al eje vertical (con pendiente hacia arriba o hacia abajo).
El signo de Fx (que es el mismo de ax) indica si la curva aumenta o disminuye de pendiente a partir de
un instante cualquiera. O tambin si el movimiento se inicia hacia los x positivos o hacia los x
negativos a partir del instante de velocidad nula (t*).

NOTA PRCTICA.
Para los razonamientos fsicos se suele descomponer el movimiento en dos fases:
hasta que se detiene (v = 0), y luego comenzando all desde el reposo (v0 = 0). De
esta manera, ignorando los signos y la distincin entre v0 y v (que luego se
deciden para cada caso), todo se puede resolver con dos expresiones muy
simples:
d = a (t)2 (7.12)
v2 = 2 a d (7.13)

Ejemplo desarrollado.
Un automvil de 800 kg de masa que viaja a 20 m/s debe detenerse en un semforo que est 55 m
ms adelante.
a) Calcular la fuerza mnima necesaria, suponiendo que se la comienza a aplicar cuando faltan 50 m
para el semforo, y se mantiene constante.
b) Calcular el tiempo demorado para frenar.
c) Calcular la velocidad cuando faltan 2 m para el semforo.
d) Graficar posicin y velocidad en funcin del tiempo.

Desarrollo
a) Aplicamos v2 = 2 a d a = v2 / (2 d) = (20 m/s)2 / 100 m = 4 m/s2 F = m a = 3200 N. En estos
clculos v se refiere a la velocidad inicial, y a es el mdulo de la aceleracin. Si se hubiera utilizado la
expresin completa (7.13) se hubiera obtenido la aceleracin negativa, y luego la fuerza tambin
hubiera resultado con signo menos, porque obviamente debe aplicarse hacia atrs.
b) Aplicamos la Ley del Impulso: 3200 N t = 800 kg 20 m/s t = 5 s.
Otra forma es aplicar t = d / vm = 50 m / 10 (m/s) = 5 s.
190

c) Aplicamos v2 = 2 a d , considerando el movimiento desde el punto para el que se calcula la


velocidad, y d la distancia hasta el punto de velocidad nula (semforo), o sea: d = 2 m. Resulta v 2 = 2
4 (m/s2) 2 m = 16 m2/s2 v = 4 m/s (dado que la aceleracin es 4 m/s2, esta velocidad se alcanza 1
s antes de la detencin). Con la expresin completa, (7.13), hubiera sido v2 = v02 + 2 a d, en donde d
hubiese sido la distancia desde el punto inicial hasta el punto para el que se calcula la velocidad, o sea
d = 48 m. De manera que el clculo debera haber sido v2 = 202 + 2 (4) 48 = 16 m2/s2 v = 4
m/s. Obviamente, de las dos maneras obtenemos el mismo valor.
d)
x (m) pendiente = 20 m/s vx (m/s)
48 50 20
Area = 50 m
10
10 t 4 t
0
1 2 3 5 7 (s) 1 2 3 5 7 (s)

Movimiento oscilatorio.
Este es un caso para el cual no podremos hacer clculos hasta ms adelante, pero son muy
importantes los razonamientos cualitativos que se plantearemos ahora, as como algunas
conclusiones a las que llegaremos. Es importante entender que saber fsica significa poder
hacer estos razonamientos antes de hacer cuentas.
Consideremos un cuerpo de masa m que se mueve horizontalmente, sin friccin, en el
extremo de un resorte de constante elstica k. El movimiento es a lo largo del eje x, cuyo
origen se elige en la posicin de equilibrio del resorte (figura 7.14)).
Las fuerzas verticales estn equilibradas entre s (peso y reaccin normal del piso), y en
ausencia de rozamiento la resultante es exactamente la fuerza que el resorte aplica al cuerpo,
la nica que se necesita considerar. Esta fuerza tiene un valor y un sentido que va cambiando
segn el grado de estiramiento instantneo del resorte: es una fuerza elstica dada por Fx =
k x , que tiende a llevar al cuerpo hacia la posicin de equilibrio.
Analicemos el movimiento a partir de una condicin inicial arbitraria como la siguiente: un
agente externo mantiene al cuerpo en un valor negativo de x (x0 < 0, o sea resorte
comprimido), y en t = 0 lo suelta.
Tenemos un cuerpo que parte del reposo impulsado por una fuerza que inicialmente vale k x0,
y que va disminuyendo a medida que el resorte se aproxima a su posicin de equilibrio.
Es importante la siguiente idea: aunque la fuerza va disminuyendo la velocidad va
aumentando, porque la fuerza siempre es hacia adelante. An si la fuerza desapareciera, la
velocidad no tendra que disminuir. Una imagen que ayuda es pensar en cuando se empuja un
automvil: se comienza aplicndole una fuerza grande para iniciar el movimiento, y a medida
que la velocidad aumenta se va disminuyendo la fuerza, pero mientras sea hacia delante, por
pequea que sea, contribuir con un pequeo aumento de la velocidad.
Bien, el caso es que la velocidad no aumentar tanto como si fuera una fuerza constante
(MRUV), pero necesariamente aumentar hasta que el cuerpo llegue a x = 0.
191

F v=0 t=0
velocidad si la fuerza
se hubiese mantenido x
vx (m/s) x0
constante
v1 0 < t < t1
F v
x
t (s)
t1
v1 t = t1

x = 0: posicin equilibrio resorte

Fig. 7.14: Un resorte empuja un cuerpo hacia la posicin de equilibrio


con una fuerza que disminuye, mientras la velocidad va aumentando.
A la izquierda grfica cualitativa de v(t). A la derecha esquema de la
situacin.

En el instante que denominaremos t1 en el cual el resorte alcanza su longitud de equilibrio (x


= 0), la fuerza exactamente se ha anulado y el cuerpo ha alcanzado la velocidad que llamamos
v1. Aunque esa es la posicin de equilibrio, el cuerpo obviamente no puede quedarse all, ya
que se est moviendo, y all exactamente no hay fuerza que lo frene y aunque la hubiera el
cuerpo seguira avanzando, necesitara un tiempo y una distancia para frenarse.
Efectivamente eso es lo que sucede: el cuerpo pasa por la posicin de equilibrio y comienza a
frenarse por accin de la fuerza elstica que crece negativamente a partir de all. Esto significa
que al pasar la posicin de equilibrio recin empieza a disminuir la velocidad, o sea: la
velocidad v1 es la mxima que alcanzar este cuerpo.
Luego el cuerpo deber detenerse en algn lugar x2, ya que la fuerza que lo frena crece
mientras el cuerpo avanza.
Y este es un buen momento para interrumpir con una pregunta:
Dnde/cundo se detendr el cuerpo?
Elija la opcin correcta y reflexione sobre todas ellas, antes de continuar leyendo.
a) Se detendr cuando se termine su fuerza de avance.
b) Se detendr cuando la fuerza (que es negativa) iguale a la velocidad (en valor absoluto).
c) Se detendr cuando la fuerza del resorte (que es negativa) iguale a la fuerza del cuerpo
(en valor absoluto).
d) Se detendr cuando el impulso aplicado por la fuerza de frenado (que es negativo) iguale
a la cantidad de movimiento m v1.
Son todas absurdas excepto la correcta, pero pueden ser tentadoras. Si encuentra algo de
tentador en a), b), o c), entonces usted debe revisar sus ideas ms bsicas. Al final de este
captulo estn las respuestas y comentarios.
192

No estamos en condiciones de determinar el valor de t1 ni de t2, pero s de afirmar que se


llegar a un instante tal que Ix(t1 t2) = m v1, y en ese instante ser v2 = 0. Adems es fcil
imaginar, por la simetra del proceso (ms adelante podremos demostrarlo) que t2 t1 = t1, y
que x2 = x1.
Una vez que el cuerpo se detiene en x2 es claro que no puede permanecer all porque el resorte
est estirado, aplicando una fuerza hacia el origen. De manera que el movimiento se reinicia
instantneamente, ahora con velocidad creciendo negativamente, y se repite exactamente lo
mismo que ocurri desde la partida, pero en sentido contrario, ya que la fuerza que aplica el
resorte es exactamente igual, salvo el signo, para el resorte estirado o comprimido. As es que
en t3 = t2 + t1 el cuerpo pasar por la posicin de equilibrio con v3 = v1, y en t4 = 2 t2 = 4 t 1,
se detendr por un instante en la posicin inicial, para recomenzar y repetir exactamente todo
indefinidamente (en la suposicin de no haber rozamientos ni influencias extraas).
vx (m/s) x (m)
x0

t t
t1 t2 t3 t4 (s) t1 t2 t3 t4 (s)
x0
v1
Fig.7.15: funciones v(t) y x(t) para una masa en el extremo de un resorte. Ntese cmo la
velocidad, es mxima justo cuando el mvil pasa por la posicin de fuerza nula, x=0.

Este movimiento es peridico, de perodo T = 2 t2 = 4 t1; se denomina perodo, T, al tiempo


transcurrido en el cual todo se repite, y frecuencia, f, al nmero de veces que se repite cada
ciclo completo por unidad de tiempo:
1
f= (7.15)
T
Se denomina elongacin a la posicin con respecto a la posicin de equilibrio (x para este
caso), y amplitud de la oscilacin al mximo valor de la elongacin (x0 para este caso).
An no podemos efectuar clculos analticos; aunque sabemos que el impulso aplicado por el
resorte en cada cuarto de perodo, desde la mxima elongacin hasta la posicin de equilibrio,
es igual a la cantidad de movimiento adquirida: I(0; t1 ) = p1 = m v1 , no estamos en
condiciones de calcular ese impulso, porque no conocemos la funcin Fx (t). Si la
conociramos tal vez podramos calcular el impulso como el rea bajo la grfica. Pero aunque
sabemos que Fx = k x , no conocemos la funcin x(t), de la cual lo nico que sabemos es que
debe tener un representacin grfica como la de la figura 7.15.
Tambin podemos determinar que, si aumentamos la masa del cuerpo, para igual posicin
inicial, deber tardar ms en llegar a la posicin de equilibrio, mientras que deber tardar
menos si aumentamos la constante k del resorte, ya que eso hara que aplique fuerzas mayores.
Esto nos permite decir que el perodo aumentar con m, y disminuir con k.
Aunque no tenemos las herramientas matemticas para averiguar ms detalles, esta
explicacin cualitativa es suficientemente valiosa an sin frmulas y clculos numricos.
En el Anexo 7.1 veremos que las funciones x(t) y vx(t) mostradas en la figura 7.15, son
funciones seno o coseno, que se denominan funciones armnicas.
193

Pero a travs de este tratamiento debe sernos posible analizar cualquier fuerza de tipo
restaurador (que empuja hacia una posicin de equilibrio), aunque no sea exactamente
elstica (proporcional a la elongacin), y debemos saber que siempre encontraremos que la
partcula sometida a esta fuerza realizar oscilaciones peridicas. En general la funcin x(t)
ser ms o menos parecida a una funcin armnica, sin serlo exactamente.
Por ejemplo: podemos pensar en la bolita de un pndulo, o en una bolita que rueda por la parte
ms baja de una pista o canaleta curvada hacia arriba.

El movimiento de cualquier cuerpo sometido a una fuerza restauradora es


oscilatorio, y si la fuerza es elstica las oscilaciones son armnicas.

7.3.- Movimientos curvilneos

Tiro oblicuo y cada libre.


ste es un ejemplo de superposicin de movimientos. Consideraremos un cuerpo de masa m

con cantidad de movimiento inicial arbitraria: p0 mv0 , sometido a partir de ese instante
inicial a una nica fuerza constante, orientada verticalmente hacia abajo, de valor P = m g , la
fuerza ejercida por el campo gravitatorio (nos estamos limitando al caso ideal sin existencia
del aire).
Este caso se presta especialmente para aplicar la ley del impulso por componentes, para un
sistema de ejes fijos, ( x , y ) , horizontal el x , vertical hacia arriba el y .

Consideremos entonces: FResultante ( 0 ; mg) .

Fx = 0 vx (t) = vo x = cte. MRU en x

ay = m g / m = g = cte MRUV en y
Se aprecia que hay un comportamiento sencillo en la proyeccin sobre cada eje. El
movimiento del proyectil puede considerarse la superposicin de dos movimientos simples
distintos:
Eje x :
Fuerza nula Movimiento uniforme
Velocidad y cantidad de movimiento mantienen sus valores iniciales
x(t) es una funcin lineal
Eje y :
Fuerza constante movimiento uniformemente variado, acelerado hacia abajo
Velocidad y cantidad de movimiento varan linealmente con el tiempo. La
aceleracin, vy /t , tiene exactamente el valor de la intensidad del campo
gravitatorio, independientemente de la masa del cuerpo que cae.
y(t) es una funcin cuadrtica
194

Veamos los aspectos notables que resultan de esto.

1.- Forma de la trayectoria


Como y es funcin cuadrtica de t, y ste a su vez est ligado con x por una relacin lineal,
entonces y debe ser funcin cuadrtica de x, y eso corresponde a una parbola de eje vertical
(tanto para la funcin y(t), como la y(x).

Ejemplo desarrollado
Se arroja un proyectil con una velocidad de 60 m/s en una direccin que forma 60 con la horizontal.
Encuentre las funciones x(t), y(t), e y(x), y grafquelas.
Desarrollo
Comenzamos definiendo que el lanzamiento o partida ocurre en t = 0 (se dispara un cronmetro en
ese instante), en el origen de los ejes x, y (el eje y se ubica verticalmente, positivo hacia arriba).
Las velocidades iniciales en cada eje son: v0x = 60 cos60 = 30 m/s; v0y = 60 sen60 52 m/s.
Con esto tendremos: x(t) = 30 (m/s) t ; y(t) 52 (m/s) t 4,9 (m/s2) t2 ; si ahora sustituimos t = x / 30 en
y(t), obtenemos: y(x) 1,733 x 0,00544 (1/m) x2 .
Graficar es ms fcil si ubicamos antes algunos puntos importantes: el tiempo para la altura mxima es
t1 = v0y / g = 5,3 s; con ese tiempo averiguamos x1 159m, y1 138 m. Por otra parte, y1 tambin se
puede obtener aplicando d = g t2/2, o tambin d = vy2/(2 g).
x (m) y (m)
x = 30 t y = 52 t 4,9 t2 y (m)
318 138 y = 1,733 x 0,00544 x2
100 138
200 100
100 50
t 20 t x
1 2 3 4 5 (s) 1 2 3 (s) 50 100 (m)
10,6 5,3 10,6 159 318

2.- Superposicin de movimiento horizontal con vertical


Este movimiento es un claro ejemplo de independencia de los movimientos segn diferentes
direcciones, las figuras siguientes tratan de destacarlo.
La figura 7.16 pretende destacar que lo que desciende un proyectil por accin nicamente de
la fuerza peso, adems de ser independiente de la masa, es independiente de que est
avanzando o no en sentido horizontal. Y de la misma manera, lo que avanza horizontalmente
mientras cae libremente, no es influido por esta cada. Estas afirmaciones tienen que ver con
las dudas de DESCARTES y MERSENNE enunciadas en el grabado que hemos presentado en los
primeros captulos: si un can que est apuntado de manera exactamente vertical viaja
horizontalmente, cunto avanza horizontalmente un proyectil lanzado por este can? Ahora
respondemos sobre la base de las Leyes de la Dinmica: en ausencia de aire avanza
exactamente igual que si no hubiese salido del can.
195

Fig. 7.16: en negro se muestran las posiciones cada 0,05s de una bola que rueda uniformemente
sobre una mesa, y cae luego de llegar al borde. En blanco se muestra una bola que cae desde el
reposo por el eje y, comenzando su cada en el mismo instante en que lo hace la anterior, y otra
que contina uniformemente en lnea recta. La figura destaca que la bola que cae oblicuamente,
desciende (en el mismo tiempo) exactamente tanto como la que lo hace verticalmente, y avanza
exactamente tanto como lo hubiera hecho si hubiese continuado en lnea recta. Los movimientos
de las bolas blancas, por otra parte, tambin pueden ser interpretados como las proyecciones del
movimiento de la bola negra sobre los respectivos ejes.

Experiencia sencilla
Coloque dos monedas justo al borde de una mesa, una de ellas, A, a unos 5 cm de la esquina, y la
otra, B, a 20 o 30 cm de la primera.
Luego consiga una regla o varilla rectilnea de unos 40 cm de longitud, a la cual se tratar de dar un
movimiento semicircular, de manera que deslizndose sobre la mesa le pegue a ambas monedas
hacindolas caer simultneamente.
Se trata de que una de las monedas reciba un impulso horizontal grande, y la otra pequeo, de
manera que la primera caiga a unos metros de la mesa, mientras la otra cae muy cerca, prcticamente
al pie de la mesa.
Para ello se recomienda sujetar la varilla de manera que pueda pivotear alrededor de un punto como
O, acomodarla en la posicin 1, paralela al borde de la mesa de manera que toque a ambas monedas,
luego llevarla hasta la posicin 2, y desde all, pivoteando en O, girarla rpidamente para golpear las
monedas.
1
moneda B
2
B
mesa vista mesa vista
desde arriba moneda A
lateral A
O

Con un poco de prctica puede lograrse que la moneda B caiga a 2 o ms metros de la mesa, mientras
la A lo hace prcticamente al pie de la misma. Lo que se pretende es comprobar, escuchando el golpe
contra el piso, que la cada de ambas monedas es simultnea independientemente de que una tiene
una gran velocidad horizontal y la otra no.
El sonido permite detectar lo que se busca con gran sensibilidad, y es posible idear muchas variantes
que mejoren la experiencia.

El tiro presentado en la figura 7.16 es un tiro horizontal, y no difiere esencialmente en nada


de cualquier tiro oblicuo. En la prxima figura 7.17 mostramos un tiro oblicuo cualquiera. La
196

figura 7.16 simplemente representa la parte de la figura 7.17 que contina desde el instante en
que el proyectil pasa por la altura mxima.
y

Fig. 7.17: el movimiento parablico


de un proyectil ideal es la exacta
superposicin de un movimiento de
subida y bajada vertical, rectilneo,
con aceleracin constante, y un
movimiento horizontal uniforme.

Tambin hay otra forma de descomponer el movimiento oblicuo de cualquier proyectil ideal.
La presentamos en la prxima figura 7.18.
C

movimiento rectilneo
uniforme del proyectil si
no actuase la gravedad
9s

Fig. 7.18: las esferas negras muestran posicio-


en

nes cada 1s de un proyectil ideal arrojado obli-


cuamente desde A. Se muestra cmo hubiera
libre

cada libre continuado uniformemente de no existir grave-


en 4s dad (Inercia). Adems se muestra cmo, si B es
el punto hasta el cual lo hubiese llevado este
cada

desplazamiento ideal en 4 segundos, entonces


B
una cada libre de 4s desde B, comenzando all
en reposo, lo restituye al exacto lugar B por el
cual el proyectil realmente pasa en t=4s. Lo
mismo se ilustra para el punto C , al cual llega el
B proyectil luego de 9s . Est claro que esto es
C vlido para cualquier instante: el viaje del pro-
yectil sometido a la accin de la gravedad
puede pensarse como la superposicin del
viaje sin gravedad, ms la accin de la grave-
dad sobre un proyectil que hubiera partido
con velocidad inicial nula.
A
Vemos en esta figura que sera lcito decir que, con respecto al movimiento rectilneo
uniforme que tendra el proyectil en ausencia de gravedad (segn principio de inercia), se
halla en cada libre desde el mismo instante en que parte. Tanto mientras sube como mientras
baja, el proyectil cae libremente con respecto a la trayectoria rectilnea mencionada.
Esto ocurre (con las modificaciones debidas a la presencia del aire) en todos los proyectiles. A
veces se tiene la idea errnea de que los proyectiles disparados por armas de fuego recorren un
trecho en lnea recta y luego comienzan a caer. Eso claramente no es as. Es muy difcil ver
estos proyectiles, y en general el observador se imagina que viajan en lnea recta; pero en
realidad mientras viajan caen como cualquier piedra. Sucede que llegan tan rpidamente a
197

cualquier punto cercano que no tienen tiempo de caer mucho, y viajan en una trayectoria de
curvatura muy suave, casi imperceptible. No obstante, estn cayendo desde el instante en que
abandonan el can del arma.
Veamos nmeros en un ejemplo: una bala disparada a 500m/s recorre 100m en 0,2s. Una
piedra cae gt2/2 0,2 m en ese lapso; y la bala tambin! La trayectoria de la bala parece
recta porque nadie percibe 0,2 m en 100 m; pero el encargado de calibrar la mira del arma, si
es un arma de precisin, s tiene en cuenta la cada de 0,2 m.
Cuando se arroja una piedra sta a lo mejor recorre 4 o 5 m en un lapso de 0,2 s, y fcilmente
se percibe la curvatura de esa trayectoria.

3.- Vector velocidad en los distintos puntos


En la siguiente figura se muestra el vector velocidad, tangente a la trayectoria, tiene una
componente horizontal constante, y una vertical variable. Un proyectil lanzado verticalmente
con velocidad inicial vAy, por ejemplo, llegara hasta la misma altura ymx porque vy es la
velocidad del movimiento proyectado sobre y, de subida y bajada en lnea recta vertical. Por
otra parte, vx es la velocidad del movimiento uniforme que se proyecta sobre el eje x.
y B=0
ymx B vB pA

C vCx
C IAB
vAy vA
vCy vC pB
IBC
En cada instante: pC
v v 2x v 2y Diagrama de cantidades
A vAx de movimiento

Fig. 7.19 : vector velocidad en tres puntos arbitrarios de la trayectoria ideal de un proyectil.
Los tres tienen exactamente la misma componente x, y la componente vertical que
corresponda para ser tangentes a la trayectoria. La velocidad en el punto ms alto, B, es la
menor de todo el trayecto, y vale lo mismo que la componente x de la velocidad inicial: v mn
= vB = vAx= vA cosA . El diagrama de cantidades de movimiento confirma las mismas cosas.

4.- Anlisis normal-tangencial


Lo que hasta aqu hallamos acerca de cmo vara la velocidad, tambin se obtiene
cualitativamente a partir descomponer la nica fuerza actuante, el peso, en sus componentes
normal y tangencial.
Como se ve en la figura siguiente, es claro que la fuerza tangencial frena al movimiento hasta
el punto ms alto, y lo acelera luego. Por otra parte, la fuerza normal curva la trayectoria.
Ambas componentes varan continuamente.
198

y
B vB

vA P C
FNC FTC
P vC

A FNA

FTA
P x

Fig. 7.20 : componentes normales y tangenciales del peso en distintos puntos de la


trayectoria de un proyectil. En el punto ms alto, B, el peso slo tiene componente
normal, que se confunde con l mismo.

Ejemplo desarrollado
Un inventor propone combatir incendios forestales soltando tanques llenos de agua desde aviones.
Los tanques se reventaran al chocar contra el suelo desparramando el agua en toda una zona, y se
decide poner a prueba la efectividad del mtodo. Para ello un avin que vuela horizontalmente a 600
km/h a 400 m de altura suelta un tanque que se desea que caiga en un punto A.
Despreciando la influencia del aire, calcule cuntos metros antes de pasar sobre A debe soltarse el
tanque, y qu velocidad tiene ste cuando est por tocar tierra.
Desarrollo
Aplicamos la independencia de los movimientos verticales y horizontales. El tiempo que demora el
tanque en llegar al piso se calcula a partir de: 400 m = g t 2/2 t = (2 d / g) 9,0 s.
En 9 s el avin avanza lo mismo que el tanque: x v9 s 1500 m el tanque debe soltarse 1500
m antes de pasar sobre el punto A (la velocidad del avin en m/s es: 6105m/3600 s 166,7 m/s).
La velocidad del tanque tendr una componente x que ser la misma velocidad del avin, y una

componente y que ser (en valor absoluto) vy = g t 88,3 m/s: v (166,7 m/s ; - 88,3 m/s). El mdulo
es: v 188,6 m/s.

Movimiento circular uniforme


La trayectoria con forma de circunferencia se logra aplicando al mvil una fuerza normal
adecuada; para que sea recorrida uniformemente se requiere la ausencia total de fuerza
resultante tangencial, es decir fuerza resultante estrictamente normal. Ahora bien, una vez
establecido que la rapidez del movimiento se mantiene constante, es claro que para obtener
una circunferencia lo nico que se requiere es mantener constante el valor de esta fuerza
normal (que es la resultante), pues ello curva uniformemente la trayectoria.
199

vectores p consecutivos

y vectores I = p py
consecutivos

(t) x px

R
Diagrama vectorial de
impulsos y cantidades
de movimiento
Fig. 7.21: a la izquierda se muestra la trayectoria indicando varias posiciones consecutivas del
mvil con los correspondientes vectores cantidad de movimiento y fuerza exterior aplicada. Los

mismos vectores p se indican en el diagrama de la derecha. Se ha sombreado el ngulo entre

dos posiciones consecutivas y entre los correspondientes vectores p consecutivos para destacar
que son exactamente iguales.
El movimiento es uniforme si el ngulo central vara uniformemente, es decir si (t) es un
funcin lineal. El cociente entre lo que aumenta el ngulo central y el tiempo transcurrido es
la velocidad angular, para la cual utilizaremos la letra griega (omega):

(7.16)
t
Notar que esta velocidad angular necesariamente es la misma con la cual cambia de direccin

el vector p , que es la que nos interesa para calcular la fuerza normal con (7.3) o (7.3).
Para relacionar la velocidad angular con el radio de la curva, y la velocidad con que se la
recorre simplemente reemplacemos la definicin de ngulo en radianes, ngulo = arco/radio
en (7.16) (usaremos en general l para el arco, es decir para la distancia recorrida a lo largo de
la trayectoria):
l
R 1 l
t R t (7.17)
v

R
Ahora bien, ya sabemos que la fuerza debe orientarse hacia el centro de la circunferencia en
todo instante, y que debe mantener constante el mdulo dado por (7.3), F = m v.
Sustituyendo en esta expresin a partir de (7.17), podemos tener las expresiones que ms se
usan:
v2
F mv m
v
R
R (7.18)
m v m2 R
R
200

Y de stas se puede obtener la expresin para el radio de la curva que seguir un mvil que
viaja con cierta velocidad mientras se le aplique determinada fuerza transversal:
mv2
R (7.19)
F

Pueden obtenerse otras expresiones, tiles para determinados casos particulares.

De manera que como conclusin podemos decir que para mantener un movimiento circular
uniforme es necesaria la aplicacin de fuerzas cuya resultante est dirigida hacia el centro, y
tenga el valor dado por (7.18). Esta fuerza se denomina centrpeta, queriendo significar
hacia el centro.
Por accin-reaccin, la persona o elemento que aplica esa fuerza al mvil sufre la accin de
una fuerza hacia afuera que le aplica el mvil a ella, llamada fuerza centrfuga, la cual, puesto
que accin y reaccin tienen siempre idntico valor, se calcula con las mismas expresiones
(7.18).
Es errnea la sensacin de que sobre el mvil acta la fuerza centrfuga: el mvil es desviado
de la lnea recta por accin de una fuerza exterior dirigida hacia donde es la desviacin, es
decir hacia el centro de la curva.
Tambin es errnea la afirmacin de que centrfuga y centrpeta se equilibran: cuando las
fuerzas se equilibran el movimiento es rectilneo adems de uniforme. Este es un claro
ejemplo de no equilibrio de fuerzas: sobre el cuerpo acta una fuerza neta, la centrpeta, que lo
desva continuamente. Si en un instante dado se suspende la aplicacin de la fuerza sobre el
cuerpo (la centrpeta), desaparece en el mismo instante la fuerza centrfuga, y el mvil, libre

ya de fuerzas, contina con el p que tena en ese instante, constante a partir de all: es decir
contina por la tangente (figura 7.22).
Los pasajeros de un mnibus que toma una curva tienden a irse hacia la pared del lado externo
porque tienden a continuar en lnea recta, y no porque algo los empuje, a pesar de lo que a
ellos les parezca. La pared del mnibus del lado externo de la curva los empuja hacia adentro,
hacindolos viajar en una lnea curva, y esa fuerza de interaccin pared contra pasajero /
pasajero contra pared da origen a los apretujones, molestias y moretones que resultan de las
tensiones mecnicas que se desarrollan.

se corta el hilo p constante a


partir de que se
corta el hilo

fuerzas centrfugas
que aplica el hilo
tirando del centro vectores p consecutivos

fuerzas centrpetas
que aplica el hilo fuerzas centrfugas que
tirando del cuerpo aplica el cuerpo tirando del hilo

Fig. 7.22: ilustracin de las fuerzas actuantes en el movimiento circular uniforme de un cuerpo
que gira sujeta por un hilo, en los instantes previos y posteriores a la ruptura del hilo. No se
considera la existencia de la gravedad.
201

Fig. 7.23: esquema de las fuerzas transversales aplicadas por el pavimento a los neumticos de
un automvil mientras viaja por una curva. La resultante de stas es la fuerza centrpeta.

Ejemplo desarrollado 1
Considere el ejemplo desarrollado al comienzo de este captulo: un automvil de 1000 kg (incluida la
masa de los ocupantes) viaja por una carretera rectilnea horizontal a razn de 20 m/s. En el punto A la
carretera se curva suavemente hacia la izquierda, de manera que 10 m ms adelante, en el punto B,
contina en lnea recta en una direccin que forma 5 con la anterior. En todo el proceso el conductor
regula la aplicacin de fuerza motriz de manera de mantener constante el valor de la velocidad en 20
m/s (el valor de la fuerza de rozamiento, de 1,5 kN es equilibrado por la fuerza impulsora, mantenida
por el conductor tambin en ese valor).
a) Verifique que obtiene los mismos resultados aplicando las formulas correspondientes al movimiento
circular uniforme.
b) Explique qu debera hacer el conductor, y cmo seran las fuerzas actuantes si la carretera, luego
del punto B, en vez de enderezarse, continuara curvada uniformemente de la misma manera
constituyendo una circunferencia.
Desarrollo
a) Valen las mismas consideraciones hechas antes sobre la fuerza vertical (el peso, de 9800 N, est
equilibrado con la reaccin normal del piso en todo momento), y sobre la tangencial (hay equilibrio
entre rozamiento y fuerza impulsora, ambas de 1,5 kN, en todos los tramos).
Ahora tambin decimos que en la curva debe haber una resultante no nula, que debe ser hacia el
centro de la curva (lo cual coincide con lo anteriormente hallado: normal a la trayectoria, horizontal,
hacia la izquierda), y el mdulo debe estar dado por (7.18): F R = m v2 / R .
Debemos averiguar R, el radio de la curva que tiene la carretera. Para ello utilizamos que 5 0,0873
rad, y dado que el ngulo en radianes arco / R, y que el arco AB es de 10 m de largo, obtenemos que
R 114 m.

5 10 m
= 0,0873 R 114 m
R
R

A B 5 = 0,0873 rad

10 m

As resulta FR 1000 kg(20 m/s)2/114 m 3,50103 N = 3,50 kN, lo cual confirma el resultado
anterior.
b) En este caso la trayectoria despus de A sera una circunferencia de 114 m de radio, que el mvil
recorrera uniformemente a razn de 20 m/s. Para lograr esto el conductor simplemente debera
202

mantener la presin sobre el acelerador para que se mantenga constante la velocidad (con esto
continuara actuando la fuerza impulsora de 1,5 kN equilibrando el rozamiento y anulando la resultante
tangencial), y mantener el volante girado adecuadamente para que el auto se mantenga sobre la
curva, con lo cual las ruedas recibiran constantemente del piso la fuerza hacia el centro de 3,50 kN.

Ejemplo desarrollado
Una persona revolea una piedra de 0,8 kg sujeta al extremo de un cordel de 2 m de longitud, de
manera que describe circunferencias horizontales uniformemente a razn de 0,8 vueltas por segundo.
Dibuje la situacin. Calcule y dibuje las fuerzas actuantes sobre la piedra.

Desarrollo
Sobre la piedra slo actan la gravedad y el hilo, y dado que el movimiento es circular uniforme
horizontal, la resultante debe ser horizontal, constante, y apuntada hacia el centro del crculo. Esto
significa que el hilo aplica una fuerza vertical que equilibra al peso, y como se ve en la figura, una
horizontal que es la resultante.
S
L
Fh

A FR
O
circunferencia en plano horizontal
P

De manera que el hilo no puede estar horizontal, y el centro de la circunferencia, O, no es el punto de


suspensin, S. El hilo debe formar cierto ngulo con la vertical, tal que Fh cos = P = m g, y se
mueve describiendo un cono, por lo cual esto se conoce como pndulo cnico. Adems la fuerza
resultante debe ser m 2 R, y tambin debe ser Fh sen.
De manera que deben cumplirse las ecuaciones:
R = OA = L sen ; m g = Fh cos ; Fh sen = m 2 R
Sustituyendo R en la tercera se tiene Fh = m 2 L (como si el hilo girase en un plano horizontal!), y
luego sustituyendo Fh en la segunda: m g = m 2 L cos, de manera que cos = g/(2 L) 9,80
(N/kg) / {[5,03 (1/s)]22 m} 0,194 78,82 ( en donde se utiliz = 20,8 (1/s) 5,03 (1/s)
= 5,03 rad/s).
A partir de estos valores lo dems es inmediato:
P = 0,89,8 = 7,84 N ; Fh = 7,84/0,194 = 40,4 N ; R = 2sen78,82 = 1,96 m ; FR = 40,4sen78,82
= 39,6 N (que se verifica que vale lo mismo que m 2 R).
Como reflexin final es interesante notar que a medida que aumenta la velocidad angular, se tiene que
cos 0, con lo cual 90. A determinada velocidad, a simple vista puede parecer que el hilo
llega a la horizontal, pero en realidad los 90 exactos no se alcanzan nunca, y el hilo siempre describe
un cono.
203

Pndulos y planos inclinados.


Un pndulo ideal consiste en un cuerpo pequeo, asimilable a una partcula puntual de masa
m, suspendido de un hilo de longitud L. Sobre el cuerpo actan la fuerza gravitatoria, o peso,

vertical, hacia abajo, de valor constante P = m g , y la fuerza del hilo, Fh orientada a lo largo
del hilo, de un valor por ahora desconocido.


m Fhilo sentido del
movimiento

Fig. 7.24: se muestran los vectores fuerza actuantes sobre el cuerpo en varias
posiciones sucesivas de un pndulo, en una pasada de izquierda a derecha. Ntese

como P es un vector totalmente constante, mientras que Fh vara en todas sus
caractersticas, pero manteniendo la orientacin del hilo. Ntese tambin, una vez
ms, que no hay ninguna fuerza hacia adelante aplicada sobre m ms que las que
resultan de proyectar tangencialmente el peso.

Accin de las fuerzas tangenciales


La nica de las dos fuerzas actuantes que tiene componente tangencial es el peso, de manera
que PT es la nica que necesitamos considerar para decidir acerca de la rapidez de este
movimiento.
As es que aplicamos (7.1), con FT = m g sen, y decimos: m g sen t = m v. Es decir que
la variacin de la velocidad de este cuerpo en cada intervalo t es absolutamente
independiente de la masa del mismo.
En la prxima figura se muestran los elementos geomtricos que debemos tener en cuenta.


direccin vertical



PT

P
Fig. 7.25: fuerza tangencial en un pndulo. Obsrvese que el ngulo que forma el hilo con
la vertical es el mismo que forma el movimiento (es decir la direccin tangencial) en cada
instante, con la horizontal.
204

El cociente v /t, que es la aceleracin tangencial del movimiento, resulta totalmente


independiente de la masa, depende solamente de la intensidad del campo gravitatorio y de la
inclinacin de la trayectoria en cada instante:
v
g sen (7.20)
t
Asimismo resultan independientes de la masa en este caso, tanto la velocidad que adquiere el
mvil luego de cierto intervalo cualquiera de tiempo, como la funcin (t), o cualquiera de las
funciones que indican la posicin con respecto a cada eje.
Antes de seguir con el pndulo es importante advertir que estas conclusiones son vlidas para
cualquier cuerpo que se desliza sin rozamiento, y sin ms acciones motrices que el peso
por una pista de cualquier forma.
Dado que el rozamiento es la fuerza tangencial que aparece en la superficie de contacto
entre el mvil y la pista, la ausencia del mismo es sinnimo de que la pista no aplica fuerza
tangencial. Por lo tanto, en los casos en que la nica otra accin sobre el cuerpo se deba a la
gravedad, la nica fuerza tangencial actuante ser la componente tangencial del peso, y para
estudiarla o describirla deberemos hacer siempre el mismo esquema geomtrico, y siempre
obtendremos:
Fuerza tangencial resultante = m g sen
v
g sen
t
Y en todos los casos llegaremos a la conclusin de que cualquiera de las funciones que sirve
para describir la posicin del mvil a travs del tiempo es independiente de la masa.
Por ejemplo en una calle con pendiente, todos los vehculos en buen estado que partan del
reposo o con igual velocidad, sin freno y sin motor, tardarn aproximadamente lo mismo en
recorrer la misma distancia, tanto si es pendiente abajo como si es pendiente arriba. Y llegarn
con la misma velocidad a los mismos lugares, independientemente de que sean bicicletas,
motocicletas, automviles, o camiones cargados o descargados (para que estas afirmaciones
sean estrictamente ciertas hay que considerar algunos detalles pero, de todos modos, la idea
esencial es que la masa no influye directamente).
Otro ejemplo sera considerar una montaa rusa de los parques de diversiones y preguntarse si
es mejor que vaya cargada o descargada para que sea ms rpida, o ms lenta, o para que haya
seguridad de que no va a detenerse sin poder trepar alguna cuesta. Si el artefacto est en buen
estado, carece en absoluto de importancia que vaya lleno o descargado, cargado con flacos o
con gordos; siempre completar su circuito en el mismo tiempo, y trepar con la misma
seguridad por todas las cuestas.
Las figuras siguientes ilustran para algunos casos tpicos.

P sen


Fig. 7.26: nica fuerza tangencial actuante
P
sobre el carrito de una montaa rusa ideal.
205

P sen = constante

Fig. 7.27: fuerza tangencial en el plano in-


clinado.
En este caso se tiene un movimiento rectil-
neo uniformemente variado, con aceleracin

v/t = g sen , independiente de m.

centro
de curvatura

P sen Fig. 7.28: deslizamiento ideal en una pista con


forma de arco de circunferencia de radio R
ubicado en un plano vertical. Este movimiento,
adems de realizarse con velocidad indepen-
diente de la masa, es idntico al movimiento

de un pndulo de longitud igual a R.

Por ltimo, si se desea aplicar una fuerza externa a un mvil en algn caso similar a stos, los
razonamientos acerca de la condicin de equilibrio o cosas similares, en la direccin
tangencial deben centrarse en la necesidad de equilibrar, o superar, a esta fuerza de valor
Psen que estamos tratando aqu.

Ejemplo desarrollado.
Un ciclista de 70 kg est con su bicicleta de 15 kg en reposo en A en lo alto de una colina, y tiene dos
caminos para llegar al punto B, en una calle horizontal 20 m ms abajo. El camino 1 es una recta
inclinada de 100 m de longitud que desemboca justo en B; el 2 tiene una bajada recta, de 50 m de
longitud hasta el punto C de la calle horizontal, seguido del trayecto horizontal restante CB, de 80 m de
longitud, como se muestra (visto desde arriba ACB no es rectilneo, por eso es un poco ms largo).
Suponiendo que el ciclista desciende sin pedalear y sin aplicar los frenos (y que no hay rozamientos
apreciables), averige, calculando lo que sea necesario, en cul de los dos caminos adquiere mayor
velocidad, y en cul llega antes al punto B.
A

20 m
C B

Desarrollo
Sabemos que para el descenso en lnea recta, sin rozamiento y sin motor, la sola accin de la
componente tangencial del peso hace que cualquier cuerpo independientemente de su masa, adquiera
una aceleracin a = g sen (en este caso puede ser o ). Al cabo de una distancia d partiendo del
reposo, con aceleracin a, adquiere velocidad tal que v 2 = 2 a d = 2 g sen d = 2 g h (donde h es el
desnivel total, ya que h/d = sen).
206

Esto ya responde a una pregunta: la velocidad que adquiere la bicicleta es la misma en AB que en AC,
porque slo depende del desnivel: v = (29,820) = 19,80 m/s, (y como veremos en el prximo
captulo esto es muy general y tiene que ver con la conservacin de la energa).
Ahora bien, el tiempo demorado en cada plano inclinado se puede calcular como t = d / vm, y dado
que vm = 19,8 / 2 = 9,90 m/s tanto para AB como para AC, tendremos: tAB = 10,10 s, y tAC = 5,05 s.
Ahora bien, la bicicleta que baja por AC, luego debe recorrer CB a la velocidad mxima, demorando
all: 80 / 19,80 = 4,04 s, con lo cual, para el tramo ACB tarda 9,09 s, alrededor de 1 s menos que por
AB.

Acerca de las fuerzas normales


Las fuerzas normales tienen que ver con la curvatura de la trayectoria.
En el plano inclinado, la trayectoria es recta y no hay fuerza normal resultante. En ese caso
podemos prever que la reaccin normal del plano equilibra exactamente a la componente
normal del peso, la cual vale Pcos .
reacciones del piso. Su resultante
equilibra a PNormal = P cos

P cos
Fig. 7.29: fuerzas normales en el plano
P inclinado.

Cuando la trayectoria es curva, como en el caso del pndulo, debe haber adems una
resultante con componente normal, centrpeta, de valor mv2/R. Como las componentes

normales existentes en este caso son: Fhilo , la cual acta puramente en esta direccin, y la
componente normal del peso, de valor Pcos, entonces la resultante normal en general se
expresa como Fhilo Pcos (ver figura 7.29), y no es cero, sino que como estamos diciendo,
vale m v2/ L.
Es decir:
mv 2
Fhilo = Pcos + (7.21)
L

Fh 1

mx
Fh 2
Fh 1
Fh 2
Fig. 7.30: fuerzas normales en el pndulo. Se indican
PN1 tambin las fuerzas con las que el hilo tira del punto de

v2 suspensin. El subndice 1 se refiere a la posicin de
P reposo instantneo, en la mxima elongacin.

P
En la posicin de reposo instantneo, en las mximas elongaciones, tenemos v1 = 0 y por lo
tanto: Fhilo 1 = P cos1 .
207

Cuando la masa pasa con mxima velocidad por la posicin ms baja, = 0 , tenemos la
mxima tensin del hilo. La fuerza resultante all es hacia arriba y responsable de la curva de
la trayectoria segn: Fhilo P = m v 2/L.

NOTA PRCTICA.
En la vida prctica a veces es ms fcil imaginar la fuerza centrfuga que la
centrpeta. Eso puede ser fuente de algunos errores, y hay que tener cuidado
para evitarlos.
Pero es interesante pensar, por ejemplo, que en el punto ms bajo el hilo tiene la
mxima tensin porque all tira de l la mxima fuerza centrfuga, adems
del peso, segn la expresin (7.21) : Fhilo 2 = P + m v22/L.

Ejemplo desarrollado
Un automvil de 1000 kg (incluida la masa de los ocupantes) viaja por una carretera rectilnea
horizontal a razn de 20 m/s. En el punto A la carretera se curva suavemente hacia arriba, de manera
que 10 m ms adelante, en el punto B, contina en lnea recta con pendiente positiva, formando 5 con
la horizontal.
Calcule la reaccin normal del piso mientras el automvil viaja horizontalmente, inmediatamente
despus de pasar por A, y despus de B, mientras viaja en lnea recta por la pendiente.
Qu cambiara si la pendiente (y la curva AB) fuera hacia abajo? A qu velocidad las ruedas
dejaran de pisar el pavimento? Qu sentiran los pasajeros?

v0 B
A 5
10 m

Desarrollo
Este ejemplo ya fue desarrollado para una pista que se curva horizontalmente. Ahora podremos utilizar
los nmeros de aquellos ejemplos, adaptndolos a esta situacin.
Sabemos que la fuerza tangencial resultante debe ser nula siempre, y que la resultante normal (ahora
en el plano vertical) debe ser de FR = 3,49 kN. Sabemos tambin que este valor puede tanto obtenerse
del dato de que la inclinacin cambia 5 en 10 m, como de que el radio de la curva es
aproximadamente 114 m.
De manera que inmediatamente despus de pasar A, la reaccin normal del piso debe valer: RN = PN +
FR 9,80 + 3,49 13,3 kN.
Ahora bien, en este caso en que la inclinacin cambia poco, tendremos que a lo largo del tramo AB
puede despreciarse tanto la variacin de la componente normal del peso, como la diferencia entre
proyectar sobre la direccin normal, o la direccin vertical, ya que PN = 9,80cos5 9,76 kN (dado
que la pendiente es pequea, la disminucin resulta casi imperceptible, del 0,4 %).
Esto significa que podemos decir que la reaccin normal del piso debe mantener aproximadamente
constante el valor aumentado de 13,3 kN a lo largo de todo el tramo, y durante los 0,5 s que dura, los
pasajeros sentirn un aumento (proporcional a la masa de cada uno) en la fuerza con que el asiento
los sostiene. Dado que esa fuerza es reaccin a la que cada uno ejerce contra el asiento, los pasajeros
tendrn la sensacin de un aumento (temporal) de peso.
208

En cambio en la direccin tangencial habr una componente tangencial del peso, PT = 9,80sen5 =
0,854 kN, que debern ser compensados con un aumento de 0,854 kN en la fuerza motriz, que deber
pasar a valer 1,50+0,85 = 2,35 kN, para seguir manteniendo nula la resultante tangencial.
13,3 kN 9,76 kN
13,3 kN
9,80 kN
2,35 kN
1,50 kN
1,50 kN 1,50 kN
1,50 kN 1,50 kN

A B 9,80 kN
9,80 kN
9,80 kN 9,80 kN

Para el tramo posterior a B, el conductor deber mantener este valor aumentado de la fuerza motriz
para que se pueda conservar la velocidad en 20 m/s.
Veamos ahora cuando la carretera se curva hacia abajo.
Ahora la fuerza resultante normal, entre A y B debe ser de 3,50 kN pero hacia abajo, para lo cual, debe
disminuir la reaccin normal del piso: RN = PN FR 9,80 3,50 6,30 kN. Por otra parte, en la
direccin tangencial la fuerza motriz debe disminuir en una cantidad igual a la componente tangencial
del peso (0,854 kN), ahora hacia delante despus de B, o sea que debe pasar de 1,50 kN antes de A,
a 0,65 kN despus de B.
Veamos ahora a qu velocidad las ruedas pierden contacto con el piso: debemos plantear que R N = 0,
y eso significa FR = 9,80 kN = m v2 / R v = (R g) 33,4 m/s.
9,80 kN
33,4 m/s
9,76 kN
33,4 m/s
1,50 kN 1,50 kN
1,50 kN
A 0,65 kN
B
9,80 kN 9,80 kN
9,80 kN
9,80 kN

A esa velocidad las ruedas pierden contacto con el piso, y el automvil est en cada libre durante un
breve lapso. Mientras dure, ni el piso sostiene al vehculo, ni los asientos sostienen a los pasajeros,
quienes comienzan a sentir ingravidez. A ms velocidad esto podr durar cada vez ms hasta que el
automvil retome el contacto, lo cual podr ocurrir cada vez ms lejos.

Comentarios sobre las preguntas referidas al movimiento oscilatorio.


a) Un cuerpo no tiene fuerza de avance. Eso no existe. Fuerza es lo que el resorte le aplica,
en este caso tirando de l para frenarlo. Por Accin y Reaccin el cuerpo tira del resorte hacia
adelante con una fuerza de exactamente la misma intensidad en todo momento.
b) Una fuerza nunca puede igualar a una velocidad, ya que son cosas de diferente naturaleza y
dimensin.
c) Vale el mismo comentario a).
d) Obviamente hay que aplicar la Ley del Impulso: m v2 = m v1 + Ix(t1t2) e igualar esta
expresin a cero.
209

ANEXO 7:

Movimiento oscilatorio armnico.


En el tratamiento del movimiento circular hemos recurrido a una manipulacin esencialmente
geomtrica de los vectores: hemos mostrado el vector velocidad tangente a la circunferencia
en cada lugar, y el vector fuerza apuntando hacia un costado perpendicularmente para lograr
que el mvil se desve continuamente de la lnea recta. Para averiguar el mdulo de este
vector fuerza hemos aplicado la Ley del Impulso, recurriendo para ello al dibujo de un
tringulo formado por los vectores correspondientes.
Ahora bien, las operaciones vectoriales que indica la Ley del Impulso, tambin pueden
hacerse analticamente, con cada componente por separado, o sea, proyectando todos los
vectores sobre el eje x, y tambin, independientemente, sobre el y.
En ese caso se obtienen expresiones para cada eje, que son vlidas independientemente de lo
que suceda en el otro eje. As, un movimiento circular uniforme proyectado sobre un eje es un
movimiento oscilatorio, y podra no haber razones a priori para que estas oscilaciones estn
emparentadas con las oscilaciones elsticas, pero veremos que s lo estn, y su descripcin nos
ser de gran ayuda.

Descripcin cartesiana del movimiento circular uniforme


Proyectaremos un movimiento circular uniforme sobre ejes cartesianos (x,y) con origen en el
centro de la circunferencia:
y
v vy v
vy

y(t) vx
vx Fx
Fy F Fy

F
=t
x R y
vx Fx
x(t) x0 = R
x

Fig. A7.1: proyeccin de un movimiento circular uniforme sobre los ejes cartesianos. Se muestran

los vectores F y v en una posicin cualquiera sobre la trayectoria, con sus respectivas
componentes. Las mismas componentes se muestran tambin sobre cada eje, asociadas con
movimientos oscilatorios que seran proyeccin del movimiento circular. Separadamente se
muestran los tringulos necesarios para los detalles de la proyeccin sobre el eje x.

Analicemos en detalle para el eje x. A partir de los tringulos destacados en esta figura po-
dramos obtener:
210

Fx vx x
cos(t ) ; sen (t ) ; cos(t ) (A7.1)
F v R
De la ltima expresin, podemos obtener la expresin para x(t):

x(t) = R cos( t) (A7.2)

Ahora bien, si efectuamos el cociente entre las expresiones correspondientes, encontramos


que aunque Fx y x, dependen del tiempo, su cociente no lo hace:
Fx F cos(t ) F
con stant e negativa = k
x R cos(t ) R
Donde k es el nombre que le damos a una constante positiva cuyo valor es k = F/R.
Vemos que el movimiento proyectado sobre el eje x se realiza bajo la accin de una fuerza
hacia el origen, de valor directamente proporcional a la distancia al mismo, que es
precisamente la ley de fuerza que da lugar a las oscilaciones elsticas.
Esto nos permite utilizar los elementos del movimiento circular para obtener propiedades de
las oscilaciones elsticas, sin recurrir a elementos de matemtica ms complejos.
Ahora decimos que esta constante de proporcionalidad es la constante k del resorte sujeto al
extremo del cual este cuerpo podra oscilar con la misma frecuencia de este movimiento
circular:
F m 2R
k m 2 (A7.3)
R R
De donde, razonando al revs, decimos:
Si tenemos un cuerpo de masa m oscilando a lo largo del eje x en el extremo de un resorte de
constante elstica k, al estar sometido a una fuerza Fx = k x, deber realizar las mismas
oscilaciones que corresponden a proyectar sobre el eje x un movimiento circular uniforme de
radio R igual a la amplitud de la oscilacin, y velocidad angular dada por:
k
(A7.4)
m
Obviamente, este movimiento circular es imaginario, no existe ms que como una ayuda para
visualizar su proyeccin coincidiendo con las oscilaciones que tratamos de explicar, en el
extremo del resorte.
De manera que la expresin de x(t) para la oscilacin elstica, efectivamente debe ser la
funcin armnica (A7.2). El perodo de la oscilacin, lgicamente, debe coincidir con el
tiempo demorado por el movimiento circular en completar una vuelta:
m 1 k
T 2 ; f (A7.5)
k 2 m
Es importante notar que la frecuencia (o el perodo) slo dependen de k y de m: el perodo
aumenta con m, que representa la inercia, y disminuye si aumenta k, que representa la dureza,
o rigidez, del resorte. Si a un cuerpo determinando, oscilando sujeto a un resorte determinado,
se le cambia la amplitud o la velocidad inicial, no cambiar el perodo.
Adems, este movimiento circular imaginario tambin sirve para obtener vx(t) como
proyeccin de la velocidad tangencial sobre la circunferencia. Esto permite obtener fcilmente
relaciones como:
211

vmx = xmx (A7.6)


que se justifica fcilmente si se tiene en cuenta que la velocidad con la que el cuerpo pasa por
la posicin de equilibrio en la oscilacin rectilnea, vmx, es simplemente la velocidad lineal
del mvil en el movimiento circular, ya que se proyecta en su verdadera magnitud en los
instantes que corresponden. En la figura A7.1, esos son los instantes en los que el mvil cruza
el eje vertical.

NOTA: LAS FUNCIONES SENO Y COSENO.


Por ltimo vale aclarar que si proyectamos el MCU sobre el eje y, tambin
encontraremos un movimiento rectilneo oscilatorio de exactamente las
mismas caractersticas (para l tambin se cumplirn (A7.3, 4, 5, y 6)).
En cambio no se cumplira (A7.2), ya que en el tringulo en el que x es el cateto
adyacente al ngulo , y es el cateto opuesto, y cumple con y/R = sen(), de
manera que:
y(t) = R sen(t) (A7.7)
Vemos que si la oscilacin sobre un eje queda descripta por la funcin cos(t),
la oscilacin sobre el otro queda descripta por sen(t). Es decir, ambas
oscilaciones tienen las mismas caractersticas fsicas, ambas estn descriptas
por funciones armnicas, y slo difieren en los instantes en los que pasan por
los distintos lugares de su trayectoria: el movimiento proyectado sobre y pasa
por su mxima elongacin cuando el proyectado sobre x pasa por el origen, y
viceversa.
Como conclusin general podemos decir que la elongacin en una oscilacin
elstica en funcin del tiempo, no importa cmo se llame, x(t), y(t), o de otra
forma, siempre estar dada por una funcin armnica, que indistintamente
puede ser ctesen(t), o ctecos(t) - y tambin podra ser una combinacin de
ambas, si hubisemos proyectado sobre cualquier dimetro oblicuo, pero no
entraremos en tanto detalle aqu.
Por ltimo revisemos las caractersticas de las funciones sen(), y cos().

1
cos() sen()

2 (rad)
90 180 360 (grados)
-1

Fig. A7.2: Representacin grfica de las funciones armnicas:


sen(), y cos().

Segn se muestra en la figura A7.2, ambas son similares, oscilan entre 1 y 1,


con la nica diferencia de que la grfica del seno es la del coseno corrida de
perodo (90) hacia la derecha: la del coseno pasa por el mximo positivo en
=0, mientras que la del seno pasa por cero en =0, y pasa por su mximo
positivo en =90.
212

Ejemplo desarrollado
Un cuerpo de masa 5kg est sujeto al extremo de un resorte de constante k=500N/m, sobre una
superficie horizontal sin rozamiento, etc. El cuerpo es desplazado de manera de estirar el resorte 4 cm,
y en t=0 se lo suelta.
Calcule el perodo, y la frecuencia de las oscilaciones que tienen lugar, escriba la funcin x(t) y
dibjela. Calcule tambin la velocidad mxima.

Desarrollo

kg.m
m 1
500 N / m s 2 1 10 s 1
100 10 f 1,59 1,59Hz
5kg kg s 2 2 s
T = 1/f = 1/1,59 = 0,63s
En cuanto a la funcin x(t) , en una situacin simplificada como es sta slo debemos elegir entre la
funcin seno y la funcin coseno. Para ello lo ms fcil es mirar la condicin inicial: sta debe ser una
oscilacin que comienza desde una mxima elongacin positiva (x0=4cm), en t=0.
Y analizando cada funcin vemos que:
La funcin seno comienza en cero (sen(0) = 0), y va aumentando durante el primer cuarto de
perodo. Por lo tanto es claro que no puede corresponder a nuestro caso.
La funcin coseno comienza en uno (cos(0) = 1), y va disminuyendo durante el primer cuarto de
perodo. Es decir comienza en el mximo valor positivo, y se es precisamiente el caso.
Por lo tanto corresponde elegir coseno.
x (cm) x(t) = 4cm cos (10 (1/s) t )
4

1 t
0,1 0,5 (s)

4 T = 0,63s

Y por ltimo, si se desea conocer la velocidad mxima, que es la que siempre tendr el mvil al pasar
por la posicin de equilibrio, se puede aplicar (A6.6): vmx = xmx = 10 [1/s] 4cm = 40cm/s .

Aplicacin: Clculo del perodo de un pndulo ideal


Consideremos una oscilacin de muy pequea amplitud. El movimiento limitado a un arco tan
pequeo no se distinguir
esencialmente de un movimiento
rectilneo. Si ubicamos un eje
cartesiano horizontal fijo, x, con
origen en la parte ms baja de la
trayectoria, las componentes
tangenciales de todos los vectores L Fig. A7.3: oscilacin de un pndulo
involucrados coincidirn con las en condicin de pequea amplitud.
correspondientes componentes x . A los fines del anlisis tangencial,
puede considerarse que el movimien-
Elegimos x positivo hacia la x to ocurre en el eje x .
derecha. La fuerza tangencial, que -xmx xmx
x=0 x(t)
213

ahora es Fx, vale P sen = m g sen , y se orienta hacia x = 0; si escribimos sen = x/L,
entonces obtenemos que la fuerza obedece a la misma ley que en el caso del resorte:
x mg
Fx mg x(t) (A7.8)
L L
ctek

Vemos que en esta aproximacin la masa oscila a lo largo del eje x exactamente como si
hubiese un resorte de constante elstica k = mg/L.
A travs de esta conclusin, podemos trasladar cualquiera de las relaciones referidas al
resorte, simplemente haciendo corresponder los elementos de los dos movimientos. Por
ejemplo, si reemplazamos k = mg/L en la expresin (A7.5) para el perodo de las
oscilaciones elsticas obtenemos:

m L
T 2 2 (A7.9)
mg g
L
era
Esta expresin es la que se conoce como 3 ley del pndulo, e indica que el perodo de estas
oscilaciones es independiente de la masa, tal cual deba ser, segn hemos dicho al estudiar el
pndulo.
Debemos decir dos cosas importantes de esta expresin:
Si bien la expresin es vlida estrictamente para oscilaciones de pequea amplitud, en la
prctica sirve muy bien para oscilaciones de cualquier amplitud. Las diferencias slo
pueden registrarse midiendo con muy buena precisin.
Esta expresin nos provee de un mtodo cmodo y preciso para determinar el valor de la
intensidad del campo gravitatorio. La medicin del perodo se puede hacer con gran
precisin simplemente dejando transcurrir muchos perodos (slo hay que contar bien). La
longitud debe medirse con cuidado.
214

CAPTULO 8:
Movimientos con conservacin de la
energa mecnica

8.1.- Sistemas con fuerzas conservativas.


Ya hemos hablado de la energa cintica, que es la forma de almacenar energa en el
movimiento de un sistema.
Ahora hablaremos de la energa potencial, que es aquella que el sistema puede tener an
estando en reposo, en funcin de las posiciones o deformaciones de las partes2.
La idea bsica de un sistema que almacena energa mecnica en forma potencial significa que
su estructura o naturaleza es tal que, cuando un agente externo realiza sobre l un trabajo
positivo W llevando sus partes desde una posicin A hasta otra B, si luego el sistema puede
retornar de B a A, lo hace realizando esa misma cantidad de trabajo W (positivo) sobre el
agente externo (le devuelve la energa antes suministrada). Es decir que estamos pensando en
un sistema en el cual, por algn mecanismo, necesariamente acta alguna fuerza interior
adems de la que le aplica el agente exterior, y esta fuerza entre las partes del sistema es la
que las hace retornar a la posicin o configuracin inicial (haciendo trabajo positivo para
ello).
Un ejemplo :
Para subir un cuerpo hasta una cierta posicin ms elevada, un agente deber hacer trabajo positivo,
ya que el cuerpo no subir espontneamente porque la fuerza peso siempre se opondr a ello. Pero
una vez que el cuerpo est en la posicin alta, la fuerza peso tender a hacerlo retornar a la posicin
baja, y si eso ocurre, har trabajo positivo en el descenso.
Ahora bien, el cuerpo no tendra la capacidad de hacer trabajo bajando si no estuviese en el campo
gravitatorio terrestre. De manera que el conjunto {cuerpo, planeta Tierra}, o el conjunto {cuerpo, campo
gravitatorio de la Tierra}, es un sistema que almacena energa potencial gravitatoria cuando el
cuerpo sube, y puede devolverla al bajar. La energa potencial gravitatoria debe ser una funcin de la
altura del cuerpo.

2
La denominacin potencial hace referencia a la posibilidad de llegar a ser algo, y no tiene que ver con la potencia; una
energa potencial, como veremos, debe interpretarse como una capacidad latente, en el sistema, algo que puede llegar a
manifestarse si se dan determinadas condiciones.
215

Otro ejemplo:
Para estirar o comprimir un resorte, o deformar cualquier cuerpo elstico, un agente hace trabajo
positivo. Pero luego el cuerpo elstico tiende a recuperar su forma inicial, pudiendo l hacer trabajo
positivo en ese proceso. Decimos entonces que almacena energa potencial elstica al ser
deformado, y puede devolverla al recuperar su forma inicial. La energa potencial elstica debe ser una
funcin de la deformacin.
Si el cuerpo no es elstico puede quedar deformado sin hacer ningn trabajo luego de ello; ese es un
ejemplo de cuerpo que no almacena energa potencial elstica.

Fuerza conservativa y energa potencial.

Ya hemos visto que hacer un trabajo W sobre un sistema es transferirle mecnicamente esa
cantidad de energa, de manera que si un sistema est interactuando slo con un agente
externo que hace sobre l la cantidad de trabajo Wext(AB), mientras el sistema pasa de la
posicin o configuracin A a la B, tenemos:
Energa transferida mecnicamente al sistema = Wext(AB)
Si adems estamos considerando procesos en los cuales no hay otro tipo de transferencia de
energa, entonces la variacin de la energa del sistema estar dada toda por la energa
transferida de este modo:
E = EB EA = Wext (AB)
Existe un Principio de Conservacin de la Energa que nos garantiza que el sistema ha
almacenado, en la diferencia EB EA , toda la energa suministrada por el agente externo. Pero
no nos garantiza que el sistema pueda realizar con ella la misma cantidad de trabajo sobre el
agente exterior, al retornar desde la configuracin o posicin B hasta la A.
Ya sabemos que en los sistemas macroscpicos eso difcilmente ocurre. Cuando ocurre, es
decir cuando el sistema tiene propiedades tales que se garantiza que puede realizar, al retornar
de B a A (para simplificar supongamos el caso en que nada de esta energa se almacena como
cintica) la misma cantidad de trabajo sobre el agente exterior que antes ste hizo sobre l,
entonces decimos que las fuerzas interiores que actan son conservativas.
Esto significa que consideramos que la energa suministrada durante el proceso AB es
conservada por el sistema, puesto que est garantizado que luego puede ser devuelta. Dado
que explcitamente hemos excluido la posibilidad de almacenar la energa en forma cintica,
decimos que la ha almacenado en forma de energa potencial: E = Ep.
Es decir, para esta evolucin AB, con EcA = EcB ,
Wext (AB) = Ep = EpB EpA (8.1)
Ahora bien, estamos hablando de sistemas especiales en los que, adems del agente externo,
acta algn agente o mecanismo interior, capaz de este proceso de almacenar energa que
estamos estudiando. Tendremos que Wresultante = Wext + Wint , y este trabajo resultante debe ser
nulo para el caso propuesto de Ec = 0, ya que (segn el teorema del trabajo y la energa
cintica) es igual a la variacin de la energa cintica total.
De manera que podemos reescribir (8.1) en trminos del trabajo de las fuerzas interiores que
intervienen en un proceso, para todos los casos en que stas son conservativas:
Wint (AB) = Ep = EpA EpB (8.1)
216

NOTA: esta expresin es parecida a (8.1) pero su validez es mayor, porque slo
involucra a las fuerzas interiores. El agente exterior, tanto como la fuerza que
l aplica, pueden existir o no. Lo que es propio del sistema es la fuerza interior
que acta entre sus partes en funcin de sus posiciones relativas. Esta fuerza es
la que puede ser conservativa o no. Si lo es, lo sigue siendo aunque el agente
exterior no exista, o aunque ste realice alguna accin distinta de las que
hemos considerado.
De manera que (8.1) DEFINE la variacin de energa potencial, y lo hace para
cualquier forma de movimiento entre A y B, con variacin o no de la energa
cintica, haya o no haya agente exterior. Mientras que (8.1) vale slo para el
caso en que el movimiento ocurre con Ec = 0 (y obviamente involucra un
agente exterior que en general se requiere para ello).

Cualquiera de estas expresiones nos muestra que la energa potencial esencialmente queda
definida a travs de su variacin Ep.
Podemos atribuirle cualquier valor arbitrario en cualquier punto configuracin arbitraria
tal como el A, y luego la expresin (8.1) permitir calcular su valor en cualquier B, a partir
de:
Ep(B) = Ep(A) Wint (AB) (8.1)
Como vemos el aumento de Ep ocurre cuando Wint < 0, es decir mientras las fuerzas interiores
estn trabajando en contra, y da cuenta exactamente del trabajo que ellas podrn hacer luego al
retornar al sistema a la situacin anterior. Y viceversa.
A partir de ahora, denominaremos en general conservativas, a las fuerzas de este tipo, sean

o no interiores al sistema considerado y las simbolizaremos escribiendo FC (como veremos, la
denominacin interior, que hemos estado utilizando puede no ser adecuada para muchos
casos de estas fuerzas).
De manera que la existencia de una energa potencial Ep se refiere siempre a la existencia de
una fuerza conservativa de algn tipo en un sistema, y queda definida en cualquier proceso
AB por:

WFc(AB) = Ep (8.2)

Condicin necesaria para que una fuerza sea conservativa.


A partir de (8.2) obtenemos que una fuerza conservativa siempre debe cumplir WFc(AB) =
WFc(BA), para dos puntos A, B, cualesquiera.

Observando la prxima figura nos damos cuenta de que esto significa que FC debe estar
igualmente definida en cada posicin, independientemente de con qu rapidez o con qu
sentido se pase por all. En cada posicin debe actuar siempre de la misma manera, con igual
intensidad y sentido, independientemente de cmo sea el movimiento. As vemos que fuerzas
como la gravitatoria, siempre vertical hacia abajo y de mdulo constante, independientemente
de que el objeto sobre el que acta suba o baje, o la elstica, siempre hacia la posicin de
equilibrio del resorte, independientemente de que ste se est estirando o acortando, cumplen
con esta condicin y son tpicos ejemplos de fuerzas conservativas.
217

En cambio encontramos que la fuerza de rozamiento invierte su sentido cuando invertimos el


sentido en que recorremos un trayecto cualquiera AB, y esto la califica automticamente como
fuerza no conservativa: quita energa a la ida (WAB < 0) , y la vuelve a quitar a la vuelta
(WBA < 0).
FC B FNC B
vBA A vBA A

vAB vAB
FC FNC
Fig. 8.1: Esquema de cmo acta una fuerza conservativa (izquierda), y una no conservativa
(derecha), mientras un mvil pasa sucesivamente por los puntos A y B en un sentido y luego en el
otro.

Energa potencial
Para simplificar el tratamiento (aunque luego mostraremos que las conclusiones son mucho
ms generales), ahora consideremos
el movimiento de una nica partcula sobre la que acta
una fuerza conservativa FC .
Esta fuerza puede ser aplicada por un campo de fuerzas creado por otro cuerpo (por ejemplo
un campo elctrico, o gravitatorio, etc.), o por elementos mecnicos, como un resorte cuyo
otro extremo est fijo.
Asociada con una fuerza conservativa siempre existe una energa potencial definida por (8.2):
Ep
( B Ep
) ) WFc(AB)
(A
Ep( AB)

Como ya dijimos, podemos atribuir a la energa potencial cualquier valor arbitrario en


cualquier punto arbitrario tal como el A, y luego podremos calcular su valor en cualquier otro
punto B, a partir de:
Ep(B) = Ep(A) WFc(AB) (8.2)

De manera que para calcular o conocer cul es la funcin de las coordenadas que da la energa
potencial correspondiente a un problema dado, lo que hay que saber es calcular el trabajo
hecho por la fuerza en un desplazamiento desde un punto de referencia hasta cualquier punto
genrico.

Repasando la definicin de trabajo encontramos que en un desplazamiento infinitesimal r ,

cuyo mdulo es l, el trabajo hecho por FC vale:

WFc = FC r = FC cos l = FCT l

componente tangencial
Y en todo el camino desde A hasta B el trabajo se expresa mediante una integral:
B
A
B
WFc(AB) = FC dr =
A
FCT dl

En este momento no estamos interesados en calcular integrales, ni en conocer especficamente


funciones energa potencial, sino en aprender a manejar las ideas y procedimientos asociados
218

importantes (luego veremos cmo se calculan o definen las energas potenciales en algunos
casos de inters).
Lo que necesitamos para entender propiedades generales de la energa potencial ahora es
estudiar lo que sucede al considerar desplazamientos infinitesimales, de manera que volvamos

a la definicin de trabajo recin escrita para el desplazamiento r .
Para una notacin ms prctica ubiquemos el eje x a lo largo del desplazamiento (cualquiera
que sea su ubicacin en el espacio). En estas condiciones FCT = FCx , y l = x, de manera
que, reescribiendo con esta notacin la definicin (8.2), WFc = Ep , obtenemos las
siguientes expresiones fundamentales:

Ep = FCxx (8.3)

E p
FCx = (8.4)
x

NOTA: LA FUERZA APUNTA HACIA DONDE LA ENERGA POTENCIAL DISMINUYE.


Vemos en la expresin (8.3) que el aumento de Ep (Ep > 0) slo puede ocurrir si
FCx tiene el signo contrario que x. Y viceversa, Ep disminuye cuando FCx tiene el
mismo signo que x.
Esto se ilustra en los esquemas siguientes, para diferentes orientaciones posibles
de la fuerza y del eje x:

Ep mayor FCx > 0 Ep menor Ep menor FCx > 0 Ep mayor


x x

Ep menor FCx < 0 Ep mayor Ep mayor FCx < 0 Ep menor


x x

Es decir, independientemente de cmo se ubiquen el eje y el desplazamiento,


Las componentes de la fuerza siempre apuntan
hacia donde la energa potencial disminuye.
Esto es lo que ya fue dicho al comienzo de este captulo: si la partcula se
desplaza o es desplazada en contra de una fuerza conservativa la energa
potencial se almacena, es decir, aumenta, mientras que si la partcula viaja
llevada por la fuerza conservativa (en el mismo sentido de ella), en este
proceso se gasta, es decir disminuye, la energa potencial.

Energa potencial gravitatoria


Consideremos un cuerpo de masa m en la vecindad de la superficie de la Tierra, donde el
campo gravitatorio es constante. Si colocamos un eje x horizontal y un eje y vertical, positivo

hacia arriba, la fuerza peso ser: P = (0 ; m g).
Una forma til y simple de calcular el trabajo que hace esta fuerza en un trayecto cualquiera es
comenzar considerando un trayecto rectilneo desde un punto (xA ; yA), hasta otro (xB ; yB).
WP(AB) = P cos dAB
219

m
PT yB yA

A
P

Fig. 8.2: Elementos para definir la energa potencial gravitatoria.


Para mayor sencillez podemos considerar en esta figura el ngulo = 90, que es el
ngulo de AB con la horizontal. Tendremos cos = sen, y dAB cos = dAB sen = y.
Entonces:
WP(AB) = P y (8.5)

Ahora bien, (8.5) es vlida para trayectos AB de cualquier forma, y no slo rectilneos. Para
convencernos de eso imaginemos que AB est compuesto de varios desplazamientos
sucesivos de distinta direccin, AAA etc. B. Al sumar todas las contribuciones
Py para obtener el trabajo total, tendremos el factor comn P por la suma de todos los y
sucesivos, cuyo resultado ser P por la diferencia total yB yA .
De manera que, si Ep = WP, entonces Ep = P y = m g yB m g yA.
Esto significa que podemos definir:

Ep = P y = m g y (8.6)

Vemos aqu cmo se cumple que la fuerza del campo apunta hacia donde disminuye Ep: la
fuerza gravitatoria apunta hacia abajo, que es hacia donde disminuye y.
Ahora bien, y representa la altura con respecto a un nivel que ha sido definido arbitrariamente
como altura cero (por ejemplo, el nivel del piso, o el nivel del mar, o el punto ms bajo de
algo, etc.). Es claro que estas elecciones posibles son todas arbitrarias, y ahora nos
encontramos con que si cambiamos esta eleccin, cambia Ep.
Qu significa esto?
Esto es una consecuencia natural de la definicin misma de lo que es una energa potencial,
dada por (8.1) o (8.2): la energa potencial es una funcin de la posicin que se define por su
variacin.
El valor particular de Ep en un punto cualquiera no tiene significado fsico; slo lo tiene su
variacin entre dos puntos. De manera que si Ep es una funcin energa potencial correcta
para un sistema, Ep + cualquier constante, tambin lo es.
Para el caso especfico de la fuerza gravitatoria, podemos decir que la funcin energa
potencial ms general posible ser:
Ep = m g y + C (8.6)

Donde C es una constante que se elige arbitrariamente. En general se elige para que Ep sea
cero en algn lugar particular. Si no se escribe nada, Ep ser cero en y = 0 (que de todos
modos corresponder a una altura arbitraria).
220

Una vez que se eligen estos valores, ya no se cambian, y los resultados con significado fsico
no sern afectados por estas elecciones, como veremos en ejemplos concretos.

Energa potencial elstica

Para el caso de una fuerza elstica a lo largo del eje x, si definimos x = 0 en la posicin de
equilibrio, se tiene Fx = k x. De manera que la fuerza tangencial es hacia la posicin de
equilibrio, y hace un trabajo negativo tanto si el resorte es estirado como comprimido. Por lo
tanto, si partimos de x = 0, y estiramos o comprimimos el resorte hasta cualquier valor x,
segn (8.1) ser:
Ep(x) Ep(0) = Welstico = rea deF(x)
F (mdulo)
F(x)
rea = x F = k x2

x
x
Fig. 8.3: Elementos para definir la energa potencial elstica.
De manera que, finalmente:
k x2
Ep(elstica) = +C (8.7)
2

Donde C es el valor arbitrario de Ep en la posicin de equilibrio (generalmente se toma C =


0).
Vemos aqu cmo se cumple que la fuerza del campo apunta hacia donde disminuye Ep: la
fuerza elstica siempre apunta hacia la posicin de equilibrio del resorte, y en este caso esta
posicin es x = 0, en la cual la energa potencial tiene su mnimo valor.

Ejemplo desarrollado 1
a) Calcule la fuerza necesaria para estirar 20 cm un resorte de constante elstica k = 3000 N/m.
b) Calcule tambin, para el proceso de estirar el resorte: el trabajo que hace un agente externo, el
trabajo que hace el resorte sobre el agente, y la energa potencial que almacena el resorte.
Desarrollo
a) F = k x = 3000 (N/m) 0,2 m = 600 N.
b) Una fuerza de 600 N actuando a lo largo de 20 cm, hubiera hecho un trabajo de 120 J; pero la
fuerza no fue siempre de 600 N, sino que comenz en cero y fue aumentando a medida que el resorte
se estiraba, y lleg a 600 al final. De manera que tenemos una grfica similar a la de la figura 8.3, y
para averiguar el trabajo realizado lo que debemos calcular es el rea, que est dada por la expresin
de la energa potencial: Wext = k (x)2 = 3000 0,22/2 = 60 J.
Una vez obtenido este valor, interpretando los signos podemos decir: Wresorte = 60 J; Ep = 60 J.
221

Para justificar los signos de Wext y Wresorte se debera hacer un esquema mostrando que la fuerza que
aplica el agente externo acta en el sentido de la deformacin, y la que (por reaccin) le aplica el
resorte a l, acta en sentido opuesto.

Ejemplo desarrollado 2
Considere un resorte alineado con el eje x que tiene un extremo fijo en x = 0, tal que su energa
potencial est dada por la funcin Ep(x) = 600 (N/m) (x 0,20 m)2.
a) Encuentre la constante elstica y la posicin de equilibrio x 0 del extremo libre del resorte.
b) Encuentre la fuerza que debe aplicar un agente para estirar el extremo libre del resorte hasta x1 =
0,30 m. Calcule el trabajo que hace el agente para estirar el resorte y mustrelo en una grfica de la
funcin Ep(x).
c) Repita los puntos a) y b) si la funcin energa potencial hubiese sido Ep 1(x) = 600 (N/m) (x 0,20
m)2 + 10 J (es decir, la misma Ep(x) ms una constante igual a 10 J).
Desarrollo
a) Dado que el coeficiente de x2, en este caso 600 N/m, en la expresin de la energa potencial
elstica, debe ser k/2, podemos deducir que k = 1200 N/m. Por otra parte es claro que el mnimo de la
funcin Ep(x) del enunciado est en x = 0,20 m, ya que all vale 0, y en cualquier x mayor o menor (que
0,20 m), Ep tiene algn valor positivo. Por lo tanto sa debe ser la posicin de equilibrio, y la diferencia
(x 0,20) es la distancia o estiramiento desde esa posicin.
b) F = k x = 1200 (N/m) (0,30 - 0,20) m = 120 N.
Wext = Ep = 600 0,12 0 = 6 J.
c) El coeficiente de x2 sigue siendo 600 N/m, de manera que k = 1200 N/m.
La funcin Ep1(x) tiene otro valor en el mnimo, pero ste sigue estando en x = 0,20 m, de manera que
sa sigue siendo la posicin de equilibrio.
La fuerza para estirar el resorte est dada por la misma expresin (independiente de la constante que
se pueda agregar a la funcin Ep) F = k x = 120 N.
Ahora Ep1(0,30) = 6000,12 + 10 = 16; Ep1(0,2) = 10; Wext = Ep1 = 16 10 = 6 J.
Veamos en las siguientes grficas cmo juegan los distintos valores.
Ep (J) Ep (J) Ep1(x)
20 Ep(x) 20
16
10 10
6
x x
0,1 0,2 0,3 0,4 (m) 0,1 0,2 0,3 0,4 (m)

8.2.- Movimiento en presencia de fuerzas conservativas y


no conservativas.
Supongamos un sistema que consiste en una partcula sobre la que acta una fuerza

conservativa FC , y alguna otra fuerza no conservativa, FNC (que podra ser aplicada por algn
agente, o deberse al rozamiento, o a un motor, etc.).
222


La fuerza resultante sobre la partcula est dada por: FR FNC FC , y por lo tanto WR = WNC
+ WFc . Ahora bien, segn el teorema del trabajo y la energa cintica, WR = Ec , de manera
que:
WNC + WFc = Ec
Cuando acta una fuerza conservativa podemos definir una energa potencial asociada con ella
segn (8.2), WFc = Ep , entonces sustituyendo esto en la expresin anterior queda:
WNC Ep = Ec ,
Esta expresin nos invita a agrupar todas las energas en el miembro derecho:
WNC Ep + Ec (8.8)
Y llamando energa mecnica total, ET , a la suma de la cintica y la potencial, podemos
escribir:
ET = Ep + Ec (8.9)

WNC = ET (8.8)

Ejemplo desarrollado: Resorte empuja, frena el rozamiento.


Un cuerpo de 16 kg est en reposo en el punto A de una pista horizontal. Un agente lo mantiene en
esa posicin comprimiendo 20 cm un resorte de constante elstica k = 10 4 N/m. En un instante dado el
agente suelta el cuerpo, que es impulsado por el resorte hasta perder contacto con l en el punto B.
Entre el cuerpo y el piso acta una fuerza de rozamiento constante de 50 N, que lo detiene finalmente
en un punto C.

A 0,20 m B C
1) Dibujar diagramas de cuerpo libre mostrando todas las fuerzas actuantes en cada tramo (entre A y
B, y entre B y C).
2) Calcule la velocidad con que el cuerpo pasa por B.
3) Calcule la ubicacin del punto C en el que el cuerpo se detiene.

Desarrollo

1) Le llamamos F a la fuerza que aplica el resorte, P es el peso, R N es la reaccin normal

(perpendicular) del piso, y FNC es la fuerza de roce, que viene a ser la parte tangencial de la reaccin
del piso.
RN RN

F
FNC

A FNC B C
P P
223

2) vB se calcula a partir de la energa cintica en B, para calcular la cual aplicamos: W NC = ET. En


esta expresin WNC es el trabajo hecho por el rozamiento, que es la fuerza no conservativa que acta
desde el instante en que se libera el cuerpo. Entre A y B, WNC = FR dAB = 50N0,2m = 10 J.
Dado que B es la posicin de equilibrio del resorte, para la energa potencial podemos plantear: EpA =
10000 (N/m)(0,2m)2/2 = 200 J ; EpB = 0. Y por otra parte, EcA = 0, con lo cual ET = EpB + EcB (EpA
+ EcA) = EcB 200 J.
Con estos elementos podemos calcular la energa cintica en B aplicando:
WNC = - 10 J = ET = EcB 200 J EcB = 190 J.
Despejando la velocidad de la expresin de Ec, en general se obtiene: v = (2Ec/m) , por lo cual ahora,
vB = (2190/16) 4,87 m/s.
Es importante notar que EcB. tambin se hubiese podido calcular razonando de una manera menos
estructurada, diciendo: el resorte va a impulsar al mvil hasta B dndole una energa igual a la
potencial que tiene almacenada en A, es decir, 200 J; y mientras esto ocurre el rozamiento le va a
quitar 10 J, de manera que el mvil va a tener, al pasar por B, Ec B = 190 J.
Desde B en adelante el cuerpo se desprende del resorte, y la nica fuerza tangencial actuante es la
del rozamiento (que adems pasa a ser la resultante). De manera que planteamos WNC = FR dBC, =
EcC EcB = 0 EcB 50 N dBC = 190 J dBC = 190 / 50 = 3,80 m.
Debe notarse que tambin podra haberse calculado la ubicacin del punto C de un solo paso, sin
calcular vB, simplemente planteando WNC = ET, para el trayecto AC: tenemos EcA = EcC = 0, por lo
cual ET = Ep = - EpA = - 200 J = WNC = FR dAC dAC = 200 J / 50 N = 4 m.

La conservacin de la energa mecnica.


En los casos en los que en un sistema slo actan fuerzas conservativas, y no existen otras
fuerzas, o bien existen otras fuerzas pero no hacen trabajo, la aplicacin de (8.8) o (8.8)
automticamente nos permite plantear que el movimiento ocurrir manteniendo la energa
mecnica total constante:
Si WNC = 0 ET = 0
O equivalentemente:
Ep + Ec= valor constante denominado ET (8.9)

Lo cual tambin puede expresarse diciendo que para dos puntos cualquiera A y B del
movimiento, se cumplir:
EpA + EcA = EpB + EcB = ET

Cada energa puede variar,


pero la suma se conserva.
224

Ejemplo desarrollado: Cada libre


Un cuerpo de masa m = 2 kg est cayendo verticalmente. En un instante t 1 pasa por A, a una altura de
100 metros del nivel de referencia (piso) y el mdulo de su velocidad es de 20 m/s. Luego, en otro
instante t2, pasa por B, a 40 metros sobre el mismo nivel.
1) Diga dnde considera el cero de la energa potencial, y de acuerdo con eso calcule la energa
mecnica total inicial del sistema .
2) Despreciando la resistencia del aire, determine la energa mecnica total, cintica y potencial del
cuerpo cuando se encuentra a 40 metros de altura.
3) Determine la velocidad en ese punto, y la que tendr luego al llegar a tierra (un instante antes de
tocarla).
4) Indique los valores de las energas (mecnicas) potencial, cintica y total, en cada punto: A, B, y C,
(inmediatamente antes de tocar el piso), eligiendo:
4.1) energa potencial cero en C.
4.2) energa potencial cero en A.
4.3) energa total cero en C (explique en dnde, aproximadamente estara E p = 0) .
5) Determine el trabajo de la fuerza peso desde A hasta B, y desde B hasta C, y muestre qu tienen
que ver esos valores con los anteriores.
Desarrollo
1) Consideramos arbitrariamente Ep = 0, en el piso, en el cual tambin situamos arbitrariamente el
origen del eje y (vertical hacia arriba). De acuerdo con esto, Ep = m g y.
Entonces, EcA = 2 kg(20 m/s)2/2 = 400 J ; EpA 2 kg9,8 (N/kg)100 m = 19,6 N100 m = 1960 J ;
ETA 2360 J ;
2) Planteamos conservacin de la energa mecnica: E TB = 2360 J ; y dado que EpB = 19,6 N40 m =
784 J, la energa cintica debe ser EcB = 2360 784 = 1576 J.
3) Despejando la velocidad de la expresin de Ec, en general se obtiene: v = (2Ec/m) , por lo cual: vB
= (21576/2) = 39,7 m/s.
Para la llegada al suelo ser EpC = 0, con lo cual EcC = 2360 J , y vC = (22360/2) = 48,6 m/s.
4) Se registran los resultados de este punto en las siguientes tablas:
4.1 4.2 4.3
Ep Ec ET Ep Ec ET Ep Ec ET
A 1960 400 2360 A 0 400 400 A -400 400 0
B 784 1576 2360 B -1176 1576 400 B -1576 1576 0
C 0 2360 2360 C -1960 2360 400 C -2360 2360 0
Es muy instructivo mirar estas tablas porque all se advierte rpidamente:
a) Tanto la energa potencial como la total cambian con la eleccin arbitraria del cero de la potencial,
no as la columna de las energas cinticas, cuyo valor tiene sentido fsico en s mismo, ya que
determina la velocidad.
b) Una vez conocido el valor de la energa mecnica total en un punto, por la conservacin, vale para
todos los puntos.
225

c) La suma de las dos primeras columnas siempre debe dar el valor de la tercera.
d) La energa potencial (y tambin la total, aunque en este ejemplo no se vea) puede ser negativa, no
as la cintica.
e) Una vez que el cuerpo llega al nivel y = 0 (si no choca con algo), seguir aumentando su energa
cintica mientras aumenta negativamente su energa potencial en los valores negativos de y. Se
considera que Ep disminuye cuando aumenta negativamente: -1960 < -1176 < 0, etc.

Para determinar el punto (llammoslo D) en el cual la energa potencial sera cero en el caso 4.3),
escribo que EpD = m g yD + Cte, y lo igualo a cero. La expresin Ep = m g y + Cte es vlida para todos
los puntos, y vale cero para el punto particular buscado, D. Si la aplicamos al punto C, para el cual y =
0, encontramos que Cte = -2360 J (notar que el mismo valor para la constante obtendramos en
cualquier punto - por ejemplo en A, la expresin quedara m g 100 m + Cte = 400 J, y de ella
tambin se obtendra que Cte = -2360 J).
Con este valor de la constante, ahora tenemos que para cualquier punto Ep = m g y 2360 J, y por lo
tanto, para que EpD sea cero, yD debe ser 2360/19,6 = 120,4 m. Esta es la altura desde la cual habra
que haber soltado este cuerpo desde el reposo para que cayera de esta forma, ya que con E T = 0, en
ese punto, con EpD = 0, tambin tendramos EcD = 0.
5) WP(AB)= 19,6 N60 m = 1176 J; WP(BC)= 19,6 N40 m = 784 J (ambos positivos porque fuerza y
desplazamiento tienen igual sentido).
Con estos valores podemos corroborar el teorema del trabajo y la energa cintica: Ec B = EcA +
WP(AB), y tambin: EcC = EcB + WP(BC).
Tambin podemos corroborar que el peso hace trabajo a expensas de la Ep: EpB = EpA WP(AB), y
tambin: EpC = EpB WP(BC).

Movimientos con vnculos.


En muchos casos prcticos hay sistemas con uno o ms vnculos que determinan o
modifican la trayectoria de la partcula, como por ejemplo un sistema de rieles o guas fijos, o
el hilo de un pndulo. Si estos vnculos tienen ciertas caractersticas ideales (ausencia de
rozamiento, por ejemplo) tales que no disipan energa, permiten seguir planteando la
conservacin de la energa mecnica.
En estos casos los vnculos aplican fuerzas sobre la partcula, y stas actan determinando la
trayectoria; pero consideramos que tienen la caracterstica ideal de no realizar trabajo. Para
esto las fuerzas de vnculo no deben tener componentes tangenciales, es decir que deben ser
fuerzas normales a la trayectoria que ellas determinan.
En estos casos, se plantea la conservacin de la energa mecnica total, con la misma funcin
energa potencial, dependiente slo de la fuerza conservativa que est en juego, e
independiente de estos vnculos y sus fuerzas.

NOTA PRCTICA
Un caso importante es el de cuerpos que se deslizan por pistas ideales sin
rozamiento. Para estos casos la nica fuerza que trabaja es el peso, y la
conservacin de la energa mecnica se traduce en que, para dos puntos A y B
de la pista o trayectoria: m vA2 + m g yA = m vB2 + m g yB. Simplificando la
masa se obtiene lo visto en el captulo anterior, en la parte de pndulos y
226

planos inclinados: la velocidad de los vehculos que viajan por carreteras de


cualquier forma, en condiciones ideales de ausencia de rozamiento y sin otras
fuerzas motrices ms que el peso, es absolutamente independiente de la masa.
Ahora adems podemos calcular esta velocidad para una pista de cualquier
forma, a partir de la expresin anterior:
vB2 vA2 = 2 g (yA yB) (8.10)
O bien:
vB2 = vA2 2 g y (8.10)
Ntese la semejanza con las expresiones del MRUV y de la cada libre vertical, lo
cual corresponde porque la cada libre vertical es a la vez un caso de MRUV, y
de conservacin de la energa. Ahora podemos ver que estas expresiones valen
para cualquier caso de conservacin de la energa en un campo gravitatorio
uniforme, con trayectoria de cualquier forma, aunque no sea ni rectilnea, ni
uniformemente variada la velocidad.

Ejemplo desarrollado: Pndulo


Considere el pndulo de la figura, consistente en un cuerpo de m = 500 g suspendido de un hilo de
71 cm de longitud, que se suelta, a partir del reposo, en la posicin A, a 45 o de la vertical. No se
consideran rozamientos.
O
L = 71 cm
45 23
22

A D

C
B
a) Calcule la velocidad del cuerpo en B, en C, y en D.
b) Indique los valores de las energas (mecnicas) potencial, cintica y total, en cada punto: A, B, C, y
D, eligiendo:
b.1) energa potencial cero en B.
b.2) energa potencial cero en A.
b.3) energa potencial cero en el punto de suspensin.
c) Dibuje cualitativamente todas las fuerzas actuantes sobre m en los puntos B, C, y D. Explique el
efecto de cada fuerza sobre el movimiento en el instante correspondiente.
Desarrollo
a) El hilo es un vnculo que obliga al cuerpo a seguir la trayectoria con forma de arco de circunferencia,
y al hacerlo provoca la aparicin de una fuerza cuyo valor se va ajustando en cada instante segn la
ley fundamental (del impulso) para las fuerzas normales. Pero esta fuerza no tiene componente
tangencial, y por ello no hace trabajo, y podemos ignorarla para plantear la conservacin de la energa
mecnica, lo cual se hace simplemente con la fuerza peso. De manera que podemos calcular las
velocidades aplicando las expresiones (8.10) o (8.10), para lo que slo necesitamos las alturas de los
puntos.
Tenemos yA yB = 0,71 m 0,71 m cos45o 0,710 0,502 0,208 m; yC yB = 0,71 m 0,71 m
cos22o 0,710 0,658 0,052 m; yA yC = 0,208 m 0,052 m 0,156 m; yD = yA .
227

O
50,2 cm 71 cm
45 23 65,8 cm
22
A D
20,8 cm C
5,2 cm
B
Entonces vB (0 + 29,80,208) 2,02 m/s ; vC (0 + 29,80,156) 1,75 m/s (vc tambin puede
calcularse a partir de vB: vC (2,022 29,80,052) 1,75 m/s) ; vD = 0.
b) Elegimos EpB = 0 ; EpA 0,5kg9,8(N/kg)0,208m 1,02 J ; EpC 0,5kg9,8(N/kg)0,052m
0,25 J.
Por otra parte las energas cinticas resultan: EcA = 0 ; EcB 0,52,022/2 1,02 J ; EcC 0,51,752/2
0,77 J ; EcD = 0.
Efectuando todas las sumas se verifica la conservacin, ya que todas dan Ec + Ep 1,02 J.
Para las otras elecciones de energa potencial cero, simplemente debemos sumar a las columnas de
Ep y de ET lo que haga falta para que se cumpla lo pedido y quede inalterada la columna de la Ec. Los
resultados de estn en las siguientes tablas:
b.1 b.2 b.3
Ep Ec Etotal Ep Ec Etotal Ep Ec Etotal
A 1,02 0 1,02 A 0 0 0 A -2,46 0 -2,46
B 0 1,02 1,02 B -1,02 1,02 0 B -3,48 1,02 -2,46
C 0,25 0,77 1,02 C -0,77 0,77 0 C -3,23 0,77 -2,46
D 1,02 0 1,02 D 0 0 0 D -2,46 0 -2,46
c) Las nicas fuerzas actuantes son, el peso, siempre igual, y la fuerza del hilo, siempre alineada con
l, hacia el punto de suspensin. La fuerza del hilo tiene mdulo variable de tal manera que supera a la
componente normal del peso en la cantidad exacta necesaria para curvar la trayectoria: F N PN =
m v2 / 2.
As tenemos que en A (y lo mismo ocurrir en D), vA = 0, FN iguala a PN, y la resultante es
exactamente tangencial, como se requiere para que el movimiento se inicie en direccin tangencial.

FB FD
FC

Resultante tangencial
Resultante normal P
P P
En el punto ms bajo, B, hay una resultante vertical, hacia arriba, que curva la trayectoria, y no hay
fuerza tangencial. Por ello la velocidad, que ha estado aumentando hasta all, deja de hacerlo, y
comenzar a disminuir.
En cualquier punto como el C, la resultante tiene una componente tangencial que va frenando el
movimiento, y sigue teniendo una componente normal (variable) que curva la trayectoria.
228

En el punto D la componente tangencial ha logrado detener el movimiento, y lo har recomenzar


instantneamente. En ese instante exacto la resultante normal es nula, y por ello el movimiento
recomienza en la direccin de la resultante, que es tangencial.
En el punto ms bajo, B hay mxima fuerza del hilo, pues es mxima la velocidad, y por ello debe ser
mxima la fuerza resultante normal. Podra pensarse que al ser sta la posicin de equilibrio del
pndulo, la fuerza resultante all debera ser nula, pero en realidad, por ser posicin de equilibrio de
una oscilacin, la que tiene que ser nula es la resultante tangencial, cosa que se cumple. Ahora bien,
dado que la trayectoria es curva, all tiene que haber una resultante normal. No debe haber equilibrio
de las fuerzas en B (a menos que el cuerpo est en reposo all), porque si lo hubiese la trayectoria no
se curvara.

NOTA PRCTICA

Si hubiese varias fuerzas conservativas, FC1 , FC 2 , ... etc. actuando sobre la
partcula, nada cambiara en los razonamientos.
Para cada fuerza conservativa hay una energa potencial definida de la
manera que ya se explic:

FC1 Ep1 , tal que : Ep1 = WFc 1

FC 2 Ep2 , tal que : Ep2 = WFc2
Etc.
El trabajo total se puede expresar en dos trminos: uno que contiene el trabajo
de todas las fuerzas conservativas, y otro que contiene el trabajo de las dems,

a las que llamaremos FNC :
Wtotal = WFc + WNC
En el trmino WFc sumamos los trabajos que vamos a reemplazar con
variaciones de energas potenciales, y en el otro quedan los de las dems
fuerzas (WNC).
Si aplicamos Wtotal = Ec, y sustituimos WFc por las variaciones de las energas
potenciales correspondientes tenemos:
Wtotal = (Ep1 Ep2 Ep3 ...) + WNC = Ec
Al igual que en (8.8), si reunimos todas las variaciones de energa del lado
derecho llegamos a:
WNC = Ec + Ep1 + Ep2 + Ep3 ...
Vemos que siempre seguir siendo vlida la expresin WNC = ET , siendo la
energa mecnica total, ET , la suma de la cintica ms todas las potenciales:
m v2
ET = + Ep1 + Ep2 + . . .
2

nica Ec Eptotal

ET = Ec + Eptotal

Dicho con otras palabras, la suma (vectorial) de todas las fuerzas



conservativas sera una fuerza resultante FC , tambin conservativa, la cual
define una energa potencial total Eptotal = Epi , que funciona exactamente con
la mismas expresiones de cualquier Ep.
229

8.3.- Interpretacin de los movimientos que conservan la


energa mecnica a partir de la grfica de la funcin
energa potencial.
Vamos a hacer aqu una presentacin limitada al caso de movimientos unidimensionales,
aunque veremos que las ideas bsicas se generalizan fcilmente a ms dimensiones.
Un movimiento unidimensional es aqul en el cual la posicin se puede indicar con una nica
variable. El ms simple es el movimiento rectilneo, en el cual se puede indicar la posicin
con el valor de x, por ejemplo, pero un movimiento unidimensional tambin puede ser
curvilneo, como en el caso del pndulo o de cualquier movimiento circular, en los cuales la
posicin se puede indicar con el valor de un ngulo, o de la distancia recorrida a lo largo de la
curva.
Ahora por simplicidad consideremos una partcula de masa m que slo puede moverse en
lnea recta, e imaginemos un eje x a lo largo de la trayectoria.
v
x
O x
Fig. 8.4: la partcula para la que se presenta esta discusin. El eje x puede representar
cualquier lnea recta del espacio a lo largo de la cual tiene lugar el movimiento.

Supongamos para abreviar que sobre m actan solamente una fuerza conservativa que

denominamos Fc (la cual acta alineada con el eje x), y que de ella slo tenemos como dato la
funcin energa potencial Ep(x) correspondiente:

E (J) Ep(x)

x (m)

Fig. 8.5: La funcin Ep(x), dato del problema, es arbitraria en esta discusin. Corresponde a la
existencia de una fuerza conservativa hipottica, y no debe pensarse que se habla
especficamente de la gravedad ni de un resorte, sino de cualquier cosa arbitraria concebible,
cuya naturaleza real no interesa. Para la gravedad y para los resortes corresponden grficas muy
simples que veremos en los ejemplos.


Sabemos que la funcin Ep(x) vara solamente cuando Fc hace trabajo, y dado que en un
desplazamiento x este trabajo vale WFc = Fcx x, resulta que, aunque nuestro nico dato sea
la funcin Ep(x), a partir de esta funcin podemos averiguar el valor de la fuerza que acta en
cada lugar del eje x aplicando (8.4), de la cual obtenemos:
E p
Fcx = = pendiente de Ep(x)
x

Supongamos que en un instante dado la partcula est en un lugar x1 con velocidad v1 hacia la
derecha; la curva nos indica el correspondiente valor de la energa potencial, Ep1 = Ep(x1), en
230

ese momento. Si calculamos Ec1 = m v12, y se lo sumamos a Ep1 obtenemos la energa


mecnica total en ese punto ET1 = Ep1 + Ec1 :
E (J)

Ep1 + Ec1
Ec1 sumamos Ec1
Ep(x1)
x
x1 v1
Fig. 8.6: As incorporamos a nuestra grfica arbitraria una situacin inicial particular,
dada por la posicin x1 y la velocidad correspondiente vx1.

Ahora la idea principal es que la partcula continuar movindose de manera tal que la energa
mecnica total mantenga constantemente el valor calculado para nuestro punto dato x1. As,
cuando la partcula pase por otra nueva posicin x2 cualquiera, su energa potencial habr
adquirido el nuevo valor Ep2 dado por la curva (Ep2 = Ep(x2)), y la Ec cambiar de manera de
tener el valor Ec2 que hace cumplir la condicin de conservacin:
Ec2 + Ep2 = ET1
E (J)

ET2 = ET1
Ep2 Ec2
Ec1
Ep1 x
Epmin x0 x1 x2
Fig. 8.7: Mientras la partcula pasa de x1 a x2, su energa cintica debe variar (en
este caso disminuir) para mantener constante la energa total.

En el caso de nuestra figura particular vemos que la partcula pasa por x1 dirigindose hacia
donde Ep aumenta (movimiento en contra de la fuerza). Como la energa cintica est
indicada por la distancia (medida en el eje de ordenadas) desde la curva hasta la lnea
horizontal que indica el valor constante ET, vemos que en esta caso va disminuyendo (Ec2 <
Ec1).

NOTA:
Como la energa cintica siempre es positiva encontramos que en cualquier x,
ET = m v2 + Ep(x) Ep(x)
Vemos que a partir del punto x1 elegido arbitrariamente en el cual
comenzamos nuestro anlisis, sumaremos al valor Ep1 una cantidad Ec1 que
siempre ser algo positivo, de manera que la lnea indicativa de E T siempre
estar por encima de Ep1, y la partcula siempre tendr alguna cierta
distancia para desplazarse mientras la curva de la funcin Ep(x) se mantiene
por debajo de la lnea horizontal indicativa de la ET.
El menor valor posible para la energa cintica es cero (v = 0). Si ese hubiera
sido el caso en nuestro punto inicial x1 , el valor de ET hubiera sido Ep1, y es lo
ms bajo que la lnea indicativa de ET podra estar.
231

Es decir, si la partcula est en algn valor de x, la energa mecnica total ya


tiene por lo menos el valor Ep(x), y no tiene ningn sentido plantearse la
posibilidad de que la lnea de ET pase all por debajo de la curva de Ep(x).
Dicho con otras palabras,
Por el solo hecho de existir en algn lugar x ,
la partcula ya tiene ET Ep(x).

De manera que si la funcin Ep(x) tiene un mnimo absoluto como el que se


muestra en la grfica en el punto x0, la lnea indicativa de ET no podr pasar
por debajo del valor Epmn. El menor valor posible para la energa total es ET =
Epmn , y slo para el caso en que la partcula est detenida en x 0. Para
cualquier otro caso ET > Epmn .
Por otra parte, el hecho de que los valores de la energa potencial sean positivos
(como en las figuras que hemos mostrado), o que sean negativos en algunos
lugares o en todos, resulta de elecciones arbitrarias, y no tiene ningn
significado especial, como ya hemos visto en algunos ejemplos.

Volvamos ahora a la discusin central.


La recta horizontal que hemos trazado indica el valor que siempre tendr la energa mecnica
Ep(x) + m v2 en este movimiento. De manera que a medida que la partcula avanza, la
diferencia entre ET y Ep (es decir la distancia que ET est por encima de la curva, medida en
unidades del eje de ordenadas) nos indica el valor de la energa cintica en cada lugar.
Y mientras sea ET > Ep , ser Ec 0, y esto significar que la partcula an no se habr
detenido, es decir, seguir avanzando.
La partcula avanzar hasta x = x , abscisa de la interseccin de la grfica de Ep(x) con la
recta indicativa de ET.
E (J)

ET
Ec2
Ec1 Interseccin: en x se tiene Ep(x) = ET
Ec = 0 v = 0 la partcula se detiene
x
x1 x2 x
Fig. 8.8: La partcula se detiene cuando llega al punto x = x , en el cual su energa cintica se anula.

En x la partcula se detiene.
Esta detencin es por un instante. La partcula no podr permanecer detenida all, porque en

ese lugar acta FC .

Cmo sabemos que Fc no es cero en x?


Pues porque Fcx = Ep/x = pendiente, y vemos que la pendiente no es cero all (y en
general no podr serlo cuando la curva intersecte una lnea horizontal, salvo casos especiales
que veremos).
232


La pendiente positiva de la grfica en x indica que all FC acta en sentido negativo.
Decimos que:
x es un punto de retorno, para este valor de la energa mecnica total.

De manera que la partcula invertir el sentido de marcha (RECORDAR QUE LA


PARTCULA NO VIAJA POR LA LNEA CURVA REPRESENTATIVA DE LA
FUNCIN Ep(x), sino que lo hace EN LNEA RECTA POR EL EJE x ) y volver a pasar por
los puntos x2, x1, x0, etc. En esta parte del viaje la fuerza (Fcx <0) actuar a favor del
movimiento haciendo que la energa cintica aumente, y que vuelva a tener en cada lugar el
valor que tuvo antes, indicado por la distancia desde la curva que representa Ep(x) hasta la
recta horizontal que representa a ET. As la partcula pasar por x2, x1, con los valores de
energa cintica Ec2 , Ec1, que tuvo antes, y su velocidad aumentar hasta x = x0 , en donde la
Ec tomar el valor Ecmx .
E (J)

ET

Ecmx
Epmn x
vmx
x0
Fig. 8.9: El punto de energa potencial mnima, x0 , es el punto por el
cual la partcula pasa con mxima velocidad (en mdulo).

A partir de este punto la fuerza comienza a actuar en contra, ya que la partcula se encuentra

viajando hacia dnde Ep aumenta (y FC apunta hacia donde Ep disminuye). La partcula
continuar mientras su energa cintica disminuye como lo indica la distancia desde la curva
hasta la recta horizontal indicativa de ET, y llegar al punto de retorno x, en donde
nuevamente curva y recta se intersectan.
E (J)

ET
Ec
Ecmx
v x

x x x0
Fig. 8.10: El mdulo de la velocidad siempre disminuye cuando la partcula viaja
hacia donde aumenta la energa potencial. Y la partcula se detiene siempre que la
partcula llega a un punto en el cual Ep = ET, en este caso x.

Y ya la grfica nos permite inferir que la partcula, luego de detenerse por un instante en x ,
se acelerar en sentido positivo, aumentar su velocidad hasta x0, y luego se frenar
gradualmente hasta detenerse instantneamente en x, repitiendo indefinidamente este
movimiento oscilatorio.
233

NOTA:
El movimiento continuar indefinidamente recuperando siempre los mismos
valores de la velocidad al pasar por los mismos puntos mientras se cumpla la
hiptesis de partida, de que slo acta la fuerza considerada, estrictamente
conservativa, en virtud de la cual se mantiene inalterado el valor de E T = Ec +
Ep = Ec1 + Ep1 .
El hecho de que esto no ocurra exactamente as en la prctica es una
manifestacin de la presencia de fuerzas no conservativas (en general tenemos
el rozamiento, para el cual ET = Wext < 0).

Para el caso particular que estamos analizando, vemos que podemos delimitar una zona entre
x y x (donde Ep ET) en la cual ocurre el movimiento. El resto del eje x es prohibido para
la partcula con esta energa total.
E (J)

ET

x
x x
Fig. 8.11: Las zonas sombreadas, x < x , y x > x , son prohibidas para la partcula con el valor
indicado de ET. En este caso la partcula se mantiene oscilando indefinidamente entre x y x .

Cabe ahora preguntarnos cules seran las caractersticas de los movimientos posibles con
otros valores de la energa total, suponiendo esta misma partcula bajo la accin de la misma
fuerza conservativa (es decir dada esta misma funcin Ep(x) ).
A continuacin planteamos pasos para un modo general de analizar movimientos a partir de la
grfica de Ep(x) siguiendo estas ideas.

a) Posiciones de equilibrio.
Dado el caso que estamos analizando, hay posiciones de equilibrio, es decir posiciones tales
que si dejamos la partcula en reposo en una de ellas, permanece all?
Si las hay, deben ser lugares en los que Fcx = 0, o sea los puntos de la grfica con pendiente
nula:
E p
Fcx = =0 Posicin de equilibrio.
x
En nuestro ejemplo hay tres posiciones de stas: x0 , x1 , y x2 .
E (J)
Ep1
Ep2

Ep0 x (m)
x0 x1 x2

Fig. 8.12: Las posiciones de equilibrio posibles para el


caso de la Ep graficada son x0, x1, y x2.
234

Como la fuerza apunta hacia donde la energa potencial disminuye, es nula en cada uno de
estos puntos exactamente, pero no en sus cercanas. Dependiendo de que el punto corresponda
a un mnimo o un mximo de la curva, la fuerza en la zona prxima apunta hacia el punto o
hacia afuera de la zona. Analicemos ahora lo que puede suceder en cada uno de estos puntos.

Puntos de equilibrio estable (mnimos de la funcin Ep(x): x = x0 , y x = x2 .


La partcula en reposo en x0 tiene ET = Ep0 = Epmn , y slo puede hacer una cosa, que es
permanecer quieta en ese punto, como ya hemos comentado. Para apartarla levemente de esa
posicin (ya sea hacia la izquierda o hacia la derecha) un agente externo debe hacer trabajo
positivo pues la energa de la partcula deber aumentar necesariamente (aunque el proceso
sea tan lento que no aumente Ec, lo mismo deber aumentar Ep para salir del lugar de Epmn).
Cuando el agente externo suspenda su intervencin dejando a la partcula con una energa
levemente superior a Ep0 , sta quedar oscilando con pequea amplitud alrededor de x0
(estamos en las mismas condiciones del ejemplo analizado antes, con la salvedad de que ahora
la energa es apenas superior a la Epmn).
Reconocemos la condicin de equilibrio estable: la partcula puede permanecer en reposo
all porque la fuerza es nula, pero si es apartada levemente hacia cualquier lado, la fuerza en
los puntos prximos tiende a volverla, haciendo que luego sta quede oscilando (levemente)
alrededor de esa posicin de equilibrio.
Podemos repetir el mismo anlisis para el punto x2 , (y en general podremos hacerlo cada vez
que hallemos que la funcin Ep tiene un mnimo).
E (J)

Ep2 + E

Ep0 + E x (m)
x1
Oscilacin Oscilacin
en torno a x0 en torno a x2
Fig. 8.13: Si un agente externo aparta levemente la partcula de una posicin de equilibrio
estable, al cesar la accin externa la partcula oscilar (levemente) en torno de dicha posicin.

Puntos de equilibrio inestable (mximos de la funcin Ep(x): x = x1 .


La partcula puede estar en reposo en x1 , puesto que all no hay fuerza. Pero si es apartada
levsimamente de ese lugar ya sea hacia la izquierda o hacia la derecha (para lo cual no se
requiere suministrar energa), la fuerza ya no ser nula, y como apuntar hacia donde la
energa potencial disminuye, tender a continuar apartando ms la partcula de all.
E (J)
ET = Ep1

x (m)
x x
x1: equilibrio inestable
Fig. 8.14: x1 es un punto de equilibrio inestable en este ejemplo.
235

Reconocemos en esta descripcin la condicin de equilibrio inestable. Y vemos que


podremos repetir el mismo anlisis en cada lugar donde hallemos que la funcin Ep tiene un
mximo.
Para este caso particular hay que aadir que la partcula en reposo en x 1 tiene, como se dijo,
energa ET = Ep1 , y si es apartada levemente hacia cualquier lado se alejar hasta un punto de
retorno (x si parti hacia la izquierda, y x si lo hizo hacia la derecha) desde el cual volver
luego nuevamente a la posicin de equilibrio inestable.
En la siguiente figura se muestra una funcin Ep(x) tal que si la partcula es apartada de su
posicin de equilibrio inestable, nunca retornar a ella.
E (J)
ET = Ep1

x (m)
x1: equilibrio inestable
Fig. 8.15: Caso de un punto de equilibrio inestable al cual la partcula nunca retornar.

Cada vez que una partcula queda en equilibrio inestable es imposible predecir si se ir o no
de all, o cundo y hacia qu lado lo har.

NOTA:
Es posible hacer una ANALOGA, imaginando que la grfica Ep(x) es una pista
o gua del movimiento en un plano vertical, y que la partcula mvil se desliza
por ella sin rozamiento. Si REEMPLAZAMOS IMAGINARIAMENTE la trayectoria
por la grfica, y la fuerza conservativa del problema por el peso de la
partcula, podremos visualizar fcilmente dnde la velocidad aumenta o
disminuye. Claramente visualizaremos que los puntos ms altos de las lomas
sern posiciones de equilibrio inestable, y los fondos de las hondonadas sern
posiciones de equilibrio estable. Etc.
Pero debe quedar claro que eso no pasa de ser una analoga, que puede ser til
o no. La real situacin fsica que estamos analizando en estas pginas es la de
un movimiento rectilneo que ocurre en el eje x, para el que la TRAYECTORIA
NO ES LA GRFICA DE LA FUNCIN, la fuerza NO ES EL PESO, etc.

b) La fuerza.
Entre las posiciones de equilibrio (pendiente nula - mximos o mnimos de la funcin Ep(x)),
existen zonas en las cuales sobre la partcula necesariamente actuara la fuerza (pendiente no
nula de Ep(x)):
E p
Fcx = 0
x
Por ejemplo para la grfica de nuestro ejemplo, las fuerzas que actuaran se muestran
cualitativamente (con vectores huecos) en la figura siguiente (8.16). Esta figura complementa
las explicaciones anteriores acerca de dnde puede haber equilibrio estable y dnde inestable.
236

E (J)

Ep(x)

x (m)

Fcx > 0 Fcx < 0 Fcx > 0 Fcx < 0


Fig. 8.16: Las flechas huecas ilustran cualitativamente las fuerzas que actan en el
caso de la energa potencial hipottica correspondiente a la grfica Ep(x).

c) Las zonas de movimiento posible, y las zonas prohibidas.


Como ya se dijo, dada una determinada ET, el movimiento slo es posible donde Ep ET. En
las zonas con Ep > ET el movimiento no puede tener lugar con esa energa, y haciendo alusin
a eso las hemos denominado zonas prohibidas (son prohibidas con ese valor de ET).
Para la grfica que estamos analizando podemos distinguir tres posibilidades principales,
segn que ET sea:
caso c1: mayor que Epmn , y menor que Ep2 ,
caso c2: mayor que Ep2 pero menor que Ep1 , y
caso c3: mayor que Ep1.
No consideramos la posibilidad de que ET sea exactamente igual a Ep0, Ep1, o Ep2, porque
seran los casos de equilibrio que ya han sido discutidos.
Veamos cada caso en detalle.

Caso c1: Ep0 < ET < Ep2


E (J)

Ep2
ET
Ep0 x

x < A, zona prohibida x > B, zona prohibida


A B

Fig. 8.17: Las zonas sombreadas muestran las partes inaccesibles


del eje x, para el valor mostrado de energa total ET.
Vemos que hay una oscilacin entre A y B, que son los puntos de retorno. Las zonas rayadas
son inaccesibles con esta energa. Etc. Es un caso que ya hemos discutido.
237

Caso c2: Ep2 < ET < Ep1


E (J)
Ep1
ET
Ep2

A B C D

Fig. 8.18: Zonas de movimiento posible para el caso Ep2 < ET < Ep1 .

Lo rayado en la figura indica las zonas prohibidas para el valor dado de ET. La partcula puede
estar oscilando entre A y B, o entre C y D. Si est en una de esas zonas, nunca podr pasar a la
otra zona posible (sin intervencin de un agente externo que cambie el valor de ET). De
manera que para una partcula entre A y B (con esta energa), no slo es inaccesible lo rayado,
sino tambin el tramo CD; y viceversa, si la partcula con esta energa est entre C y D, nunca
saldr de all, y todo el resto, incluida la zona AB estar prohibida para ella.
Por otra parte, el movimiento entre A y B, o entre C y D, es una oscilacin que se analiza de la
misma manera que las que hemos discutido antes.

Caso c3: ET > Ep1


E (J) Ecmn1 Ecmx2
ET
Ep1
Ecmx0

Fuerzas de la barrera x
A x0 x1 x2 B

Fig. 8.19: Movimiento posible para el caso ET > Ep1 .


La partcula oscila entre A y B, con una zona intermedia de frenado al acercarse a x 1, como
resultado de las fuerzas que ya hemos explicado. Al pasar por x1 la partcula llega a tener un
valor mnimo de energa cintica, que luego aumenta hasta valores mximos (en x0 o en x2 ,
segn el sentido de marcha), para despus disminuir nuevamente hasta la detencin
instantnea en el punto de retorno que corresponda.
La zona cercana a x1 acta como una barrera repulsiva que dificulta la aproximacin de la
partcula, y luego devuelve la energa impulsndola cuando se aleja.
El efecto de esta barreara es muy pronunciado si la energa total supera en muy poco la altura
de dicha barrera: la partcula pasa muy lentamente por ella, llegando a casi detenerse, para
ganar considerablemente velocidad luego de atravesar la zona. Si en cambio la energa total
est muy por encima de Ep1 el efecto de la barrera puede ser poco perceptible, y el
movimiento puede ser ms o menos parecido a cualquier oscilacin entre A y B.

d) La constante arbitraria.
Si dada una funcin Ep(x) imaginamos otra que slo difiera de ella en una constante aditiva
C:
238

Ep(x) = Ep(x) + C ,
NADA CAMBIA EN LA PENDIENTE en cada lugar, es decir que NADA CAMBIA EN LA

FUERZA FC , ni, por lo tanto, en los movimientos posibles.

Para obtener un mismo movimiento particular con Ep(x) que con Ep(x), por supuesto, hay
que considerar la energa total ET= ET + C .
En la siguiente figura se muestran a modo de ejemplo, tres funciones energa potencial. Una
es la Ep(x) que hemos venido utilizando hasta ahora para las explicaciones, otra es Ep(x) =
Ep(x) C, y la otra es Ep(x) = Ep(x) C, con C elegida tal que, arbitrariamente, la
energa potencial valga cero en x1 (hubo que definir C = Ep1).
E (J) Ep(x)

E T
Ep(x)

ET
Ep(x)

A x0 B x2 x (m)
x1
ET

Fig. 8.20: Tres funciones energa potencial equivalentes pues slo difieren en un valor constante
Vemos que estas tres grficas definen las mismas caractersticas de la fuerza, tienen las
mismas posiciones de equilibrio, etc. Por ejemplo para un valor cualquiera de la energa total
ET referido a la grfica Ep(x), se obtienen iguales oscilaciones, con los mismos puntos de
retorno A y B, que si se considera la energa total ET= ET + C para la grfica Ep(x), o la
ET = Em C para la Ep(x).
Debe notarse que no tiene ninguna importancia en s mismo el hecho de que la energa
potencial sea positiva o negativa. Sumando o restando una constante adecuada podemos
transformar valores positivos en negativos y viceversa. Si se suma o resta la misma constante
arbitraria a la energa mecnica total, tendremos el mismo movimiento. As en el ejemplo de
la figura vemos que el movimiento con energa ET negativa, referido a la grfica Ep(x), es
el mismo que el descripto por la grfica Ep(x) con energa ET positiva. Lo que importa es que
en ambos casos en cada lugar es igual la diferencia ET Ep(x), que da la energa cintica de la
partcula.

e) Movimientos con vnculos.


Ya hemos comentado sobre los casos en los cuales hay uno o ms vnculos que modifican la
trayectoria de la partcula aplicndole las correspondientes fuerzas.
Si estos vnculos tienen la caracterstica ideal que ya hemos mencionado: no disipar energa,
entonces no alteran la ecuacin de conservacin de la energa mecnica, y podemos aplicar el
mismo tratamiento que hemos presentado hasta aqu.
En estos casos podremos tener una trayectoria que no sea rectilnea, y por ello tal vez puede
convenir elegir una coordenada ms adecuada que x para indicar la posicin: por ejemplo en
un pndulo el ngulo , o la altura del cuerpo oscilante con respecto a cierto nivel horizontal,
239

o para un mvil sobre una pista arbitraria, la distancia recorrida, medida a lo largo de la pista,
etc.
Todos estos movimientos para los cuales la posicin se puede indicar con una sola
coordenada, se denominan unidimensionales.
Independientemente de la coordenada que se elija en cada caso, cuando tengamos la funcin
Ep escrita en funcin de esa coordenada, siempre podremos hacer este mismo tratamiento:
para una energa total ET, la partcula slo podr estar en lugares para los cuales Ep ET, y en
cada lugar tendr Ec = ET Ep , o sea velocidad de mdulo v = 2 Ec / m , con alguno de
los dos sentidos posibles, etc.

Ejemplo desarrollado 1
El siguiente grfico representa aproximadamente la energa potencial de un cuerpo de 2 kg que se
puede mover a lo largo del eje x.
E (J)
2
Ep(x) = 500 [J/m2] x2

x (cm)
-4 -2 2 4 6

a) Diga cul es la posicin de equilibrio diga sin es estable o inestable.


b) Si la energa total es 0,80 J, explique cmo es el movimiento.
b.1) Encuentre los puntos de retorno.
b.2) Encuentre la velocidad con la cual este cuerpo pasa por la posicin de equilibrio.
c) Encuentre la energa que es necesario suministrar al sistema para que el cuerpo aumente la
amplitud de sus oscilaciones hasta 6 cm a cada lado de la posicin de equilibrio.
d) Indique si esta grfica podra corresponder al cuerpo sujeto en el extremo de un resorte. En ese
caso encuentre la constante elstica del resorte, y la frecuencia de las oscilaciones.

Desarrollo
a) La grfica muestra una nica posicin de equilibrio, en x = 0, que es de equilibrio estable, por ser un
mnimo.
b) b.1) Trazando un recta indicativa del valor de la energa total por el valor 0,8 J, encontramos que
intersecta a la grfica en x = 4 cm, que son los puntos de retorno (estos valores tambin pueden
encontrarse analticamente despejando de la expresin para Ep(x) igualada a 0,8 J).
b.2) Al pasar por la posicin de equilibrio el cuerpo tiene la energa cintica mxima, que es 0,80 J; por
lo tanto, en valor absoluto, vmx = 2 0,8 J / 2 kg 0,89 m/s.
c) Para que la amplitud de las oscilaciones aumente hasta 6 cm, la energa total debe aumentar hasta
1,8 J, y para ello Em = 1 J.
d) Efectivamente esta grfica podra corresponder a las oscilaciones del cuerpo sujeto a un resorte de
constante elstica tal que k = 500 J/m2, es decir k = 1000 N/m. La frecuencia de las oscilaciones
segn la frmula para el resorte, estara dada por f = (1/2) k m 3,56 1/s.
240

Ejemplo desarrollado 2
La siguiente grfica muestra la Ep(x) de una partcula de 20 g de masa que se mueve en lnea recta a
lo largo del eje x, pasando por el punto A con vA = 20 m/s, como lo indica la figura.
E (J)
20 m/s Ep(x)
3
x (m)
-4 -2 2 4
C
A B
-6

a) Calcule la energa total del movimiento. Indquela en la grfica, y utilice este esquema para describir
si la partcula pasa por B y por C. Describa las caractersticas principales de este movimiento. Indique
en qu lugares la partcula encuentra fuerzas, y cmo son stas.
b) Calcule la velocidad mnima con que debe venir la partcula desde la izquierda para poder pasar por
B.
c) Repita lo solicitado en el punto a), para el caso en que la partcula incida con la misma velocidad,
pero desde la derecha hacia la izquierda (vC = 20 m/s) .

Desarrollo
a) Al pasar por A tenemos la energa potencial indicada por la grfica, Ep(A) = 6 J, y un energa
cintica de 0,02 kg (20 m/s)2 / 2 = 4 J. De manera que ET = -6 + 4 = 2 J.
E (J) punto de retorno
20 m/s Ep(x)
3
x (m)

ET = -2 2 4
-2
Ec = 4 J
-6
Vemos que hay un punto de retorno aproximadamente en x = 1 m, de manera que es claro que la
partcula no llega a B ni a C. La partcula viaja libre de fuerzas por los x negativos hasta el origen (es
claro que su energa cintica es constante en este tramo). All aproximadamente comienza a actuar
una fuerza que la frena, logra detenerla por un instante en el punto de retorno mencionado, y la lanza
de vuelta hacia los x negativos, por los que continuar uniformemente, libre de fuerzas, para siempre.

b) Para que la partcula supere la barrera que hay en B, su energa total debe ser E T 3 J. Para ello,
mientras se acerca por los x < 0, debe hacerlo con Ec ET Ep = 9 J, o sea con velocidad vA 30
m/s. En estas condiciones la partcula pasar por B con una mnima velocidad, y luego, entre
aproximadamente x = 2 y x = 3 m sufrir la accin de una fuerza hacia la derecha que la acelerar
hasta que adquiera una energa cintica Ecfinal = ET, que se mantendr constante indefinidamente de
all en adelante.
E (J)
ET > 3 J
v> 30 m/s 3
Ecfinal
x (m)
-2 2 4
Ec > 9 J

-6
241

c) Si la partcula incide desde la derecha con velocidad vC = - 20 m/s, o sea con energa cintica Ec = 4
J, se mismo es el valor de la energa total, ya que en esta parte E p = 0. Entonces a la partcula le
sobra para pasar la barrera y continuar hacia los x negativos, alejndose en esta parte con E c = 10 J,
lo que significa v = 1000 m2 / s 2 31,6 m/s. La figura correspondiente al punto anterior
representa bastante bien esta situacin, excepto por el signo de la velocidad.

Ejemplo desarrollado 3
Los dos tomos de una molcula diatmica cualquiera (esquema (b)), se ejercen fuerzas tales que
resulta una energa potencial de interaccin Ep(x) como la mostrada en (a).
E
(J) Ep(x) y nube electrnica

x0 x x

(a) x (b)

a) Explique cmo es la fuerza neta actuante sobre cada ncleo en funcin de la distancia entre ambos,
x, segn la forma de la grfica. Explique tambin las razones fsicas para que la fuerza sea de
atraccin, o repulsin, segn los valores de x.
b) Si para un compuesto arbitrario AB se registra experimentalmente que al formarse se obtienen 500
kJ/mol, y que la distancia media entre el ncleo de A y el de B en cada molcula AB formada es 0,20
nm, indique los valores que corresponderan en la grfica.
c) Indique un valor negativo cualquiera de la energa total en el dibujo. Explique cmo es el movimiento
correspondiente. Interprete detalles importantes. Indique los puntos de retorno. Comparando con dos
masas en los extremos de un resorte, interprete cules son las masas que vibran, y cul es el resorte,
o medio elstico.
d) Para el estado vibracional elegido en el punto anterior, indique (cualitativamente) en la grfica,
cunta energa sera necesario suministrarle para disociarla, a partir del estado vibracional elegido.
cunta energa podra an quitarse a la molcula, y en qu estado quedara sta cuando se le
hubiese quitado toda la energa posible.

Desarrollo
a) Leyendo exclusivamente lo que dice la grfica vemos que hay una posicin de equilibrio en x 0. Para
x < x0 la fuerza neta es repulsiva, mientras que es atractiva para x > x 0 . Ahora bien, si un agente
externo acerca los ncleos, encuentra que a medida que x disminuye la fuerza repulsiva se hace cada
vez ms grande, y es imposible llegar a x = 0 (que significara que ambos ncleos se han juntado en el
mismo lugar), porque se requerira una energa infinita para ello. En cambio si el agente externo hace
aumentar la distancia internuclear ms all de un cierto valor, la grfica muestra que la fuerza atractiva
se debilita y puede llegar a desaparecer, con lo cual la molcula se habra descompuesto en dos
tomos independientes.
Fsicamente la ligadura entre los tomos se entiende porque ellos disponen de estados orbitales
ligantes para algunos electrones, los cuales corresponden a una distribucin de carga negativa entre
los ncleos capaz de equilibrar o superar la repulsin entre ellos.
La posicin de equilibrio es precisamente aqulla en la cual estn equilibradas las fuerzas repulsivas
con las atractivas.
242

Se entiende la fuerza repulsiva cuando x < x0 como resultado de que al aproximarse los ncleos,
resulta insuficiente la carga electrnica (negativa) que queda entre ambos.
Inversamente, la fuerza es atractiva cuando x > x0 porque la distribucin de la nube electrnica en esta
situacin es tal que aumenta la carga negativa entre los ncleos, y la atraccin de estos electrones
sobre ambos ncleos supera a la repulsin mutua entre ambos.
Pero cuando la distancia entre los ncleos aumenta demasiado, la nube electrnica no se estira
indefinidamente como si fuera un material flexible, sino que los electrones se distribuyen en estados
orbitales que se van definiendo cada vez ms como pertenecientes a un ncleo o al otro, pero no al
conjunto. As es que si un agente externo separa mucho los ncleos, finalmente tendr una de dos
situaciones diferentes siguiente:
O bien cada ncleo se habr quedado con sus electrones, y tendremos dos tomos neutros que
prcticamente ya no interactuarn ms, como en el caso de una molcula de tomos iguales,
por ejemplo N2 N + N,

O bien un ncleo se quedar con un electrn de ms que habr arrebatado al otro en virtud de
una mayor afinidad, y tendremos dos iones de signo opuesto, que an interactuarn, pero con
una fuerza que tambin se debilitar con la distancia (ley de Coulomb), como en el caso de
ClNa Cl- + Na+
En cualquiera de estas dos situaciones, al aumentar x la grfica Ep(x) tender a una lnea horizontal
(pendiente nula) indicativa de que la fuerza de interaccin va desapareciendo.

b) c) y d) Se indica arbitrariamente un nivel ET. Supongamos un valor numrico de -300 kJ/mol


( kJ/mol )
puntos de retorno
Para la energa total indicada
0,20
(arbitrariamente), el valor de x (que indica la
x (nm)
distancia entre los ncleos) oscilara entre los
valores correspondientes a los puntos de
energa necesaria
ET retorno indicados. Fsicamente esto
para disociar
representa que ambos ncleos oscilan
500
acercando y alejndose alternativamente, de
manera que el centro de masa permanece fijo.
Es como si fueran dos cuerpos unidos por un resorte: cada ncleo es uno de los cuerpos y la nube
electrnica es el resorte. Muchos ms detalles se analizan en el captulo de vibraciones moleculares.
Por otra parte, la energa cintica media de los ncleos en esta vibracin se relaciona directamente
con la temperatura del sistema, suponiendo que no hablamos de una molcula aislada, sino de un
sistema macroscpico compuesto de ellas.
Como se indica en la figura anterior, la energa que es necesario suministrar al sistema para disociarlo
es la diferencia entre el nivel cero (disociacin), y el valor E T del estado vibracional elegido, es decir
300 kJ/mol.
Por otra parte, la energa que es posible quitar al sistema es la que queda entre el nivel E T y el mnimo
posible. Este mnimo posible, desde un punto de vista clsico sera el fondo de la curva (-500 kJ/mol),
pero como se explica en los captulos correspondientes, segn la mecnica cuntica el mnimo est
levemente por encima de ese valor. Esto significa que, clsicamente, la molcula puede quedar con los
ncleos en reposo a la distancia de equilibrio, con energa total exactamente igual a -500 kJ/mol; pero
la Mecnica Cuntica no admite eso, sino que indica que con la mnima energa an habr cierta
vibracin fundamental que no puede desaparecer. Ese estado fundamental de energa mnima tendr
cierta energa E0 levemente superior al mnimo de -500 KJ/mol, por ejemplo, sin pretender que sean
nmeros vlidos, digamos E0 = -490 kJ/mol. Eso significa que lo que podra quitarse a la molcula con
energa ET es ET E0 = 190 kJ/mol.
243

Anexo 8.1:

Conservacin de la energa y Primer Principio de la


Termodinmica
La situacin en la cual la energa mecnica de un sistema se conserva, es una situacin
absolutamente ideal, que en el mundo macroscpico slo se da en grados de aproximacin
mayor o menor.
Es claro que no existe un principio de conservacin de la energa mecnica, desde que la
expresin (8.8), WNC = ET , prev la posibilidad de la variacin de la misma en un sistema
que puede estar totalmente aislado. En esta afirmacin es importante recordar que el miembro
izquierdo de (8.8) representa trabajo de fuerzas que a veces se denominan errneamente
exteriores, pero que pueden ser perfectamente interiores. El ejemplo ms simple es el de un
reloj de cuerda (de los antiguos relojes de cuerda), que funciona a expensas de cierta energa
elstica acumulada en la cuerda, que es un resorte espiral. Cuando un agente externo carga
esta cuerda de energa, el reloj funciona durante un tiempo, hasta que esa energa se agota. Lo
que disipa su energa mecnica son los rozamientos, total y completamente interiores.
Esto sucede en cualquier sistema mecnico aislado.
Sin embargo, una de las propiedades definitorias de la energa es su conservacin. Realmente,
s existe un Principio General de Conservacin de la Energa, que se cumple en todas las
situaciones concebibles.
Este Principio no dice que se conservan las formas mecnicas de la energa, sino que se
conserva la energa en general, pudiendo desaparecer de una forma y continuar existiendo en
otra/s. La forma tpica para hacer cualquier balance de energa en la vida prctica tiene que ver
con los aspectos trmicos, por eso este principio general de conservacin se denomina
Primer Principio de la Termodinmica. Este nombre obedece al proceso en el que se
desarrollaron las ideas de energa (la Revolucin Industrial), en el que se uni lo trmico con
la dinmica, en la lucha por entender y mejorar las mquinas trmicas.
Este principio tiene el siguiente enunciado muy simple, que por razones obvias slo considera
formas trmicas y mecnicas de suministrar energa:

Primer Principio de la Termodinmica


Q + Wext = E (A8.1.1)
En donde:
Q es la cantidad de energa suministrada como calor, es decir por simple contacto o
proximidad con cuerpos a diferente temperatura. Q positivo significa calor transferido al
sistema por un cuerpo ms caliente, y a la inversa, Q negativo significa calor transferido
por el sistema a cuerpos ms fros.
244

Wext es el trabajo mecnico hecho por las fuerzas exteriores actuantes sobre el sistema.
Wext positivo significa energa suministrada al sistema mecnicamente, y viceversa, Wext
negativo significa energa que el sistema transfiere mecnicamente a otros sistemas.
Finalmente E es la energa del sistema, de cualquier tipo que sea. Una cantidad que se
compone de contribuciones mecnicas, trmicas, y de todo tipo, que depende del estado del
sistema. No es posible dar una expresin general para la energa, sino expresiones
particulares para cada tipo o forma de energa. Hemos visto, en pginas anteriores,
expresiones para casos tpicos simples de energas mecnicas. Hay una variedad inagotable
de posibilidades de la energa para distintos sistemas y fenmenos.
Se le pueden agregar a (A8.1.1) otros trminos que contemplen el ingreso o salida de energa
del sistema en procesos de otro tipo, adems de los mecnicos y trmicos, con tal de respetar
la idea fundamental de la conservacin de E. Teniendo en cuenta esas posibilidades, no hay
excepciones a este principio: es un Principio.
Es muy importante entender que la energa que ingresa de una forma, no tiene por qu
almacenarse de esa forma. As por ejemplo, en cualquier situacin de movimiento con
friccin se eleva la temperatura de un sistema haciendo trabajo sobre l, sin ponerlo en
contacto con algo ms caliente, es decir:
W > 0 , pero Q = 0 , y el nico efecto es que se eleva la temperatura. Es decir, se suministra
energa mecnicamente, pero se almacena trmicamente
Lo mismo sucede con cualquier sistema calefactor elctrico, al sistema entra energa elctrica,
pero lo nico que cambia en el sistema es que eleva su temperatura (es claro que no almacena
energa elctrica).

Unidades.
Desde el momento en que se aceptan (A8.1.1) o sus variantes como referente mximo para el
tratamiento de cualquier energa, y una vez aceptada la definicin de trabajo, Q y E tienen que
tener la misma unidad SI que el trabajo, o sea, J = Nm.
Ahora bien, dado que el proceso de definicin de la energa oblig a juntar ideas de mecnica,
con ideas sobre el calor, que se haban desarrollado independientemente, result natural que la
ciencia del calor tuviera ya sus unidades, entonces hubo que determinar equivalencias entre
las mismas.
La unidad natural para cantidad de calor, la calora, cal, que se define como la cantidad de
calor para elevar en 1C la temperatura de 1 g de agua (especficamente de 15,5 a 16,5 C), es
una unidad (no SI) de energa muy aceptada.
La equivalencia con el joule es:
1 cal 4,185 J
1 J 0,239 cal
245

Ejemplo desarrollado 1
Un automvil cuya masa (incluyendo ocupantes) es de 900 kg, viaja 50 km por una ruta horizontal a
razn de 80 km/h, sufriendo una fuerza de rozamiento (esencialmente por parte del aire) de 700 N.
a) Calcule la potencia mecnica efectiva que aplica el vehculo al piso en estas condiciones.
b) Si se supone que el motor aprovecha el 40 % del poder calorfico del combustible para producir
trabajo, y que el 20 % de ese trabajo se disipa en los rozamientos internos de los mecanismos del
vehculo sin transferirse al piso, calcule la energa total del combustible que se consume en el trayecto.
c) Si el combustible utilizado es nafta, con un poder calorfico de aproximadamente 9000 kcal/litro,
calcular la cantidad de nafta consumida.
Desarrollo.
a) El trabajo mecnico para vencer el rozamiento de 700 N, a lo largo de 50 km, es F d 3500104 J =
35 MJ. Como el tiempo demorado es t = 50/80 = 0,625 h = 2250 s, la potencia mecnica resulta P
35 MJ / 2250 s 15,5 kW (que en HP, sera aproximadamente 21 HP).
Es importante advertir que la energa cintica es irrelevante aqu, ya que si pensamos en que
apagamos el motor y dejamos que el vehculo avance a expensas de la energa almacenada, ella slo
le alcanzara para recorrer una muy pequea fraccin de los 50 km. Podemos corroborar esto
calculando: 80 km/h 22,2 m/s; Ec 90022,22/2 = 222 kJ 0,2 MJ, fraccin despreciable de los 35
MJ que estn en juego.
b) Si E es la energa total del combustible consumido, el enunciado dice que: 35 MJ (1 0,2)Wmotor
, o sea que Wmotor 35 / 0,8 = 43,75 MJ, y que esto es 0,4Ecombustible, o sea Ecombustible = 43,75/0,4
109 MJ 26103 kcal.

c) Esta cantidad de energa es la que se obtendra a partir de 26103 / 9000 = 2,9 litros de nafta.

Ejemplo desarrollado 2.
Un calefactor elctrico de inmersin es capaz de elevar la temperatura de 2 litros de agua desde los 20
hasta los 90 C en 5 minutos.
a) Calcular la energa empleada en este proceso, y la potencia del calefactor.
b) Calcular la cantidad de kWh que aporta esta operacin al consumo elctrico del hogar.

Desarrollo
a) En este proceso el calefactor toma una cierta cantidad de energa elctrica de la red domiciliaria y la
suministra como calor al agua. Sabiendo que se requiere 1 cal para elevar la temperatura de cada
gramo de agua en 1 C, podemos calcular que para este caso se necesitarn: Q = 1(cal/gC) m
T = 1(cal/gC) 2000 g 70 C = 140103 cal = 140 kcal.
Para expresar esa energa en J utilizamos la conversin 1 cal = 4,185 J, y resulta Q = E = 140
4,185 586 kJ.
La potencia es P = E / t = 1,95 kW.
b) El kWh es unidad de energa: 1kWh = 1 kW 3600 s = 3600 kJ; E = 586/3600 = 0,163 kWh.
Tambin se podra calcular multiplicando la potencia en kW por el tiempo en horas: 1,95(5/60) =
0,163 kWh.
246

ANEXO 8.2:

El trabajo de las fuerzas interiores.


El teorema del trabajo y la energa cintica relaciona el trabajo de la fuerza resultante con la
energa cintica de un cuerpo que se puede asimilar a una partcula puntual. As tenemos un
caso muy simple, que sirve para guiar muchos razonamientos, pero que en su simplicidad
puede generar algunas ideas engaosas.
Para superarlas es necesario revisar algunos detalles de casos ms generales.
Pensemos entonces en un sistema formado por varias partes rgidas, sobre las cuales actan
tanto fuerzas exteriores como interiores (algunas partes pueden estar unidas o vinculadas entre
s de diferentes maneras, algunas uniones pueden ser elsticas, y tambin puede haber
rozamientos).

Tenemos FR = Fint +
Fext , y por lo que ya hemos dicho al estudiar las leyes de la


Dinmica sobre que Fint = 0, resulta que FR =
Fext , y en el movimiento del centro de
masa (CM) slo intervienen las fuerzas exteriores:

mtotal vCM = FR t = F ext t

A partir de este resultado, un anlisis superficial puede sugerir, errneamente, que las fuerzas
interiores tampoco deberan modificar la energa cintica de cualquier sistema, cuando en
realidad lo nico que no pueden modificar es el movimiento del centro de masa.
Efectivamente, si tratamos de calcular el trabajo realizado por todas las fuerzas, encontramos
una diferencia muy grande con el procedimiento para calcular el impulso: no es lo mismo el
trabajo de la fuerza resultante, que la suma de todos los trabajos de todas las fuerzas!
Para ver esto supongamos cada fuerza aplicada en un lugar particular, y calculemos el trabajo
total en un cierto movimiento:
Wtotal = Fext,i di cosi + Fint,j dj cosj
Donde di o dj son las distancias recorridas por cada uno de los puntos sobre los que est
aplicada cada fuerza, y i , j, los correspondientes ngulos (entre cada fuerza y el
desplazamiento del punto sobre el que acta).
En un caso general, los desplazamientos son todos distintos. Son necesariamente iguales
solamente en dos casos: en el de una partcula puntual, o en el del movimiento de traslacin
pura de un cuerpo rgido.
En estos casos, efectivamente todos los d son iguales y salen factor comn de la sumatoria,
quedando el resultado ms simple:
Wtotal = ( Fext,i cosi + Fint,j cosj ) d = FR(tangencial) d
247

Segn este resultado, la suma de los trabajos de todas las fuerzas es igual al trabajo de la
fuerza resultante, y adems es igual al trabajo de las fuerzas exteriores actuantes sobre el
sistema. Todo esto suena tan bien, que es muy fcil equivocarse y creer que tiene validez
general, pero slo la tiene para estos dos casos citados, y para casi ninguna otra situacin.
En general, la suma de todos los trabajos de las fuerzas aplicadas no es igual al trabajo de la
fuerza resultante: los trabajos de todas las fuerzas aplicadas se pueden calcular cuando se
conocen todos los movimientos de las partes donde estn aplicadas, mientras el trabajo de la
fuerza resultante ni siquiera puede ser calculado porque la fuerza resultante es una
abstraccin que no est aplicada realmente en algn lugar definido.
Lo que se hace es definir (arbitrariamente y porque permite obtener un resultado interesante)
el trabajo de la fuerza resultante considerada aplicada en el CM. A ese trabajo lo

denominamos WFR, y no es igual al trabajo de las fuerzas exteriores (ya que FR es la resultante
de ellas, pero ellas estn aplicadas en otros puntos), ni es igual al trabajo de todas las fuerzas

(ya que las fuerzas interiores son totalmente independientes de FR ).
Definiendo de esta manera el trabajo de la resultante, atribuyndole el desplazamiento del
CM, recuperamos el resultado de la partcula puntual:
WFR = EcT
Donde EcT = mtotal vCM2, es la energa cintica de traslacin. Ntese que para la partcula
puntual, que consta de un nico punto (el cual a la vez es su CM), no tiene sentido la rotacin
sobre s misma, por lo que toda la energa cintica es de traslacin.
Por otra parte, para un cuerpo rgido la energa cintica total puede escribirse como la suma de
la de traslacin ms la de rotacin, y teniendo eso en cuenta, las expresiones simples ms
tiles en general, adems de la anterior son:
Wtotal = Ectotal
WNC = Em
Donde:
Wtotal es el trabajo de todas las fuerzas exteriores e interiores.
WNC es el trabajo de todas las fuerzas no conservativas exteriores e interiores
Ectotal es la suma de todas las energas cinticas de traslacin y de rotacin de todas las partes
del sistema.
Em es la energa mecnica total: cintica de traslacin ms rotacin, ms potencial, de todas
las partes del sistema.
248

CAPTULO 9:
Dinmica de las rotaciones

El siguiente paso en complejidad luego del movimiento de la partcula puntual, que


constituye la mxima simplificacin posible, consiste en estudiar el movimiento general
de un cuerpo rgido, que siempre se puede considerar como la superposicin de un
movimiento de traslacin con uno de rotacin.
Las leyes para el movimiento de la partcula puntual, como ya hemos dicho, tambin
describen el movimiento del centro de masa de cualquier cuerpo, y con un simple
razonamiento se extienden para abarcar el movimiento de traslacin pura de los cuerpos
rgidos.
Y en este captulo veremos cmo obtener, siempre a partir de estas leyes de la partcula
puntual, las que corresponden al movimiento de rotacin de los cuerpos rgidos.
Con estos elementos tambin abarcaremos la descripcin de los movimientos de
cualquier aparato o mecanismo que se pueda descomponer en partes rgidas, en cuyo
caso podrn aplicarse estos conceptos a cada una de esas partes.

9.1.- Generalidades sobre el movimiento de rotacin


El movimiento de rotacin es un movimiento de los cuerpos rgidos en el cual hay una recta,
denominada eje de rotacin, cuyos puntos permanecen fijos. Por la rigidez del cuerpo, los
dems puntos describen movimientos circulares manteniendo todas sus distancias o
posiciones relativas invariables.
Este movimiento, adems de poder ser uniforme o variado de diversas maneras, puede
combinarse a su vez con otros movimientos, como el de traslacin, o con otras rotaciones
alrededor de otros ejes, pudiendo obtenerse una gran variedad de situaciones posibles.
El caso ms simple posible se denomina rotacin pura, y se da cuando el eje permanece fijo.
Si adems el eje contiene al centro de masa del cuerpo, se dice que la rotacin es intrnseca:
en un leguaje coloquial, cuando un cuerpo ejecuta una rotacin intrnseca, se dice que el
cuerpo rota sobre s mismo, ya que mantiene fijo el centro de masa (la palabra inglesa spin,
que significa retorcer, o girar algo sobre s mismo, se utiliza para este tipo de rotacin a
nivel de partculas atmicas).
Por otra parte, si el eje se mueve tenemos una rotacin que no es pura; este movimiento puede
ser simple, pero tambin puede llegar a ser muy complicado.
249

Un caso ms bien simple es el de las ruedas de los vehculos cuando viajan en lnea recta: la
rueda gira alrededor de su eje, fijo respecto del vehculo, mientras ste se traslada. El
resultado es la combinacin de rotacin con traslacin. En la figura 9.1 se ilustran ste y
algunos otros casos.

motor

Volante de mquinas rotatorias fijas:


ROTACIN PURA, e INTRNSECA

Pndulo:
ROTACIN PURA,
NO INTRNSECA,
Trompo: OSCILATORIA
ROTACIN + ROTACIN del EJE
Ruedas de la bicicleta: (PRECESIN)
ROTACIN + TRASLACIN

Fig. 9.1: varios casos diferentes de rotacin.

En este captulo desarrollaremos, esencialmente, los conceptos que tienen que ver con la
rotacin pura.

Rotacin pura.
Consideremos un cuerpo rgido girando alrededor de un eje fijo. El cuerpo se considera
integrado por partculas de masa mi, y cada una ellas describe una circunferencia de radio i
en un plano que permanece fijo, perpendicular al eje, como se muestra en la figura 9.2.

i
mi j

mj
eje

Fig. 9.2: cuando un cuerpo describe una rotacin pura los puntos del
eje permanecen inmviles, mientras los otros describen
circunferencias en planos perpendiculares al eje. Como se ilustra, el
cuerpo en rotacin no necesita tener simetra ni forma determinada.

Es importante notar que cada punto material describe su propia circunferencia de centro Ci y
radio i. El centro de cada circunferencia es el punto interseccin del eje con el plano de
movimiento de la partcula considerada, y en general no es el origen de las coordenadas.
El origen de las coordenadas O se fija arbitrariamente. Por ejemplo, en el caso de la figura
9.3, se fija en algn punto del eje, que podra ser tambin el centro de masa, aunque eso no es
250


importante. El vector posicin de la partcula i es ri , cuyo mdulo en general no es igual al
radio de la circunferencia descripta por ella, ya que ste es i, que es la distancia de la
partcula al eje (tomada perpendicularmente).

Slo para los puntos que giran en el mismo plano que contiene al origen de coordenadas O, se
cumple que ri = i.

mi vi mi
ri i
i

Ci O Ci

Vista en el plano de
movimiento de mi

Fig. 9.3: Cada partcula i describe, en un plano perpendicular al eje, una circunferencia
cuyo centro Ci es la interseccin de dicho plano con el eje. El radio de esta

circunferencia es i , que resulta ser la proyeccin del vector posicin ri sobre el
plano del movimiento, y que indica tambin la distancia desde la partcula hasta el eje.

La condicin de que el cuerpo sea rgido implica que, aunque cada punto material recorre su
propia trayectoria circular con su propia velocidad lineal vi , todos tienen la misma velocidad
angular , porque todos giran el mismo ngulo en el mismo tiempo. Si recordamos que para el
movimiento circular = v / radio, para este caso tenemos:
vi
: igual para todos los puntos del cuerpo (9.5)
i

9.2.- Momento de una fuerza con respecto a un eje


Efecto de las fuerzas sobre la rotacin.
Una descripcin dinmica de la rotacin implica poder establecer cmo vara la velocidad de
rotacin en funcin de las fuerzas exteriores aplicadas al cuerpo.
Para esto, lo primero que hay que tener en cuenta es que: una fuerza slo influye sobre un
movimiento de rotacin si se la aplica de manera de tener componente en la direccin en la
cual el eje permite el movimiento del punto sobre el cual acta.
As es que, una fuerza aplicada sobre un punto A de un cuerpo en rotacin pura, en el mismo
sentido en que se mueve el punto, har un trabajo positivo y aumentar la velocidad de la
rotacin, mientras que aplicada en sentido contrario har trabajo negativo, y har disminuir
dicha velocidad.
De las consideraciones sobre el trabajo que la fuerza puede hacer se deduce que, dada una

fuerza exterior cualquiera FA que se aplique en un punto A, fuera del eje, para el efecto sobre
la rotacin slo interesa la componente en la direccin tangencial a la circunferencia
descripta por A; las otras dos componentes, Faxial , paralela al eje, perpendicular al plano de la
251

circunferencia descripta por A, y Fradial, en dicho plano, en la direccin de la recta que pasa por
el centro de dicha circunferencia, no hacen trabajo y no tienen efecto sobre la rotacin (figura
9.4).
Fradial
Fradial
FA

Faxial A
A Ftang
Ftang A
eje
CA CA
Circunferencia
descripta por A

Vista en el plano de
movimiento de A

Fig. 9.4: Se muestra el vector hueco FA , indicativo de una fuerza aplicada en A, y con flechas
llenas, sus componentes axial, radial y tangencial. En lnea de trazos tambin se muestra la

proyeccin de FA sobre el plano del movimiento del punto A, vector cuyas componentes en ese

plano tambin son Fradial y Ftan g . Para considerar efectos sobre la rotacin slo interesa Ftan g . Se
ilustra un cuerpo en rotacin que no tiene simetra ni forma determinada.

Para el caso especial de una fuerza aplicada exactamente en algn punto del eje, queda claro
que no puede influir sobre la rotacin, ya que estos puntos no se mueven y, por lo tanto, la
fuerza no hace trabajo (para estos puntos no hay direccin tangencial).
Es decir, en general la fuerza aplicada puede tener las tres componentes,

FA Faxial Fradial Ftan g , pero la nica componente con posibilidades de influir sobre la
rotacin es la Ftang.

Momento de una fuerza respecto de un eje.


Ahora tratemos usar estas ideas para establecer una expresin para lo que denominaremos
momento de la fuerza con respecto a un eje, concepto que representa el poder de la fuerza
para modificar (producir, detener, etc.) la rotacin de un cuerpo alrededor del eje. A veces,
tambin le diremos poder de rotacin de la fuerza con respecto al eje.
Para simplificar los razonamientos, consideremos una rotacin orientada con el eje
perpendicular al plano de la hoja, de manera que en nuestros esquemas veamos la rotacin
directamente hacia un lado u otro en el plano del papel. Para facilitar las ideas pensemos en un
disco o plato redondo de radio R que tiene absoluta libertad de rotacin alrededor del eje, que
ser el punto O en nuestros dibujos. Cualquier fuerza que apliquemos sobre un punto del disco
podr producir o no rotacin, pero no lograr cambiar la ubicacin del eje (ste est montado
sobre cojinetes que le permiten girar pero no desplazarse).

Ahora bien, es claro que si una fuerza FT se aplica en A tangencialmente a la circunferencia de
centro O, tiene el mismo poder de rotacin que si se aplica tangencialmente en cualquier otro
punto de esta circunferencia, como se sugiere en la figura 9.5(a).
252


Por otra parte, si en A se aplica F oblicuamente, para comparar su poder de rotacin con el

de FT slo se necesita ubicarla angularmente con respecto a dicha circunferencia, y
descomponerla segn las direcciones radial y tangencial. La componente radial indicar una
accin tendiente a desplazar el eje, que ser equilibrada por las reacciones en los soportes del
eje que impiden su desplazamiento, mientras que la componente tangencial expresar
estrictamente la accin de la fuerza tendiente a producir rotacin (figura 9.5(b)).

A
A
O
FT
O
FT F
A A
C B
(a) (b) Frad

Fig. 9.5: (a) estas fuerzas aplicadas tangencialmente a la misma circunferencia tienen el
mismo poder de rotacin con respecto a O a condicin de tener igual intensidad,
independientemente del punto particular de la circunferencia sobre el que actan.
(b) si una fuerza se aplica oblicuamente, su poder de rotacin est dado exclusivamente por
su componente tangencial, independientemente de la existencia de la componente radial.

Ahora bien, dada una fuerza aplicada en A nos interesa saber cunto debe valer una fuerza
aplicada en otro punto, para equilibrar el poder de rotacin de la primera, y para resolver eso

es suficiente con advertir, en la figura 9.5, que F podra ser equilibrada en todos sus efectos,

incluido su poder de hacer rotar, por F que se aplicara en cualquier punto B, C, etc, de la
misma recta de accin.

Ahora, tracemos una circunferencia con centro en O, tangente a la recta de la fuerza F en el

punto B (figura 9.6), e imaginemos la fuerza F aplicada en B (la denominamos FB ). Esta
fuerza acta tangencialmente a su circunferencia, y tiene el mismo poder de rotacin respecto
de O que cualquier fuerza de mdulo FB aplicada tangencialmente en cualquier punto de la

circunferencia de radio OB. Adems, ella puede equilibrar la accin de F aplicada en A, cuyo

poder de rotacin es el de FT (a la cual denominamos ahora FA ), aplicada tangencialmente en
A o en cualquier punto de la circunferencia de radio OA.

O
FA
FB F
B A

Fig. 9.6: Dada FB aplicada tangencialmente en B, para encontrar la fuerza que
pueda equilibrar su poder de rotacin aplicada tangencialmente en una
circunferencia de mayor radio, debe prolongarse la tangente en B hasta cortar a la
circunferencia mayor. En ese punto, denominado A, se proyecta tangencialmente

FB y se obtiene FA .
253

Ahora bien, observando la figura 9.6, vemos que FA F OB OA , de donde se deduce que la
relacin entre los mdulos de las fuerzas que hay que aplicar tangencialmente en puntos de
dos circunferencias de diferente radio para tener el mismo poder de rotacin est dada por:
FA OA = FB OB (9.6)
Podemos decir que esta expresin define precisamente el poder de rotacin de cada fuerza con
respecto a O, ya que expresa una cantidad proporcional a la fuerza aplicada, tal que si
comparsemos su valor para dos fuerzas cualesquiera orientadas de esta manera, aqulla para
la cual este producto sea mayor, superar en poder de rotacin a la otra.
En el lenguaje matemtico se acostumbra a denominar momento de la cantidad tal con
respecto a un eje o punto, al producto de esa cantidad por la distancia al eje o al punto, de
manera que segn la expresin que hemos hallado, al poder de rotacin le corresponde
precisamente la denominacin momento (que no debe interpretarse como algo que tiene que
ver con el tiempo o el instante).

De manera que, definimos el momento o poder de rotacin con respecto a O de una fuerza F
aplicada en A formando un ngulo con la lnea OA:

MF,O = F OA sen (9.7)


En donde, MF,O significa el momento de F con respecto a O, y se sobreentiende, en esta
expresin y en las otras similares, que OA indica la longitud del segmento OA.
Teniendo en cuenta que, como se ve en las figuras 9.5 y 9.6, F sen es FT, la componente de

F tangencial a la circunferencia que puede describir el punto A alrededor del eje, y que
OAsen es igual a la distancia OB entre el eje y la recta de accin de la fuerza, tenemos que
son definiciones equivalentes del momento:

MF,O = F OB
= FT OA (9.7)

La distancia OB entre el eje y la recta de accin de la fuerza se denomina brazo de palanca


de la fuerza respecto del eje, y utilizaremos para ella en general la letra b.

NOTA 1: ACERCA DEL BRAZO DE PALANCA


Cuando la fuerza se aplica en A de la manera ms efectiva, tangencialmente a
la circunferencia por la que se desplazar el punto A, resulta =90o, sen =1, y
el brazo de palanca coincide con OA. En este caso el momento vale FOA, que es
el mximo valor que puede tomar para las distintas orientaciones posibles de
la fuerza. El caso opuesto, de mnimo valor para la misma fuerza aplicada en
el mismo lugar A, es cuando la fuerza est alineada con el centro O. En ese
caso la distancia de O a la recta de accin de la fuerza es cero, ya que la recta
pasa por O.
254


O F O b=0
b = OA
90o MF,O = 0
MF,O = FOA A A
F
Momento mximo Momento nulo
Fig. 9.7: Caso de momento mximo y de momento nulo, para una fuerza
aplicada a distancia OA del punto eje O.

O O O
F b
b F
b F
A A A
Fig. 9.8: Cuando la fuerza se aplica oblicuamente, el brazo de palanca debe
buscarse como la distancia entre O y la recta de accin de la fuerza, tomada
perpendicularmente a la recta (la distancia de un punto a una recta slo tiene
sentido entendida de esta manera).

ACERCA DEL SENTIDO DE LA ROTACIN


La rotacin ocurre en el espacio, tiene orientacin, y se puede describir con
ayuda de ciertos vectores especiales (ver Anexo 9.1). Por ahora, en un planteo
simple, digamos que, observando de manera de ver de frente el plano de la
rotacin (en el cual tienen lugar las circunferencias descriptas por los puntos
que giran), es decir viendo de punta el eje, se acostumbra a asignar signo +
al sentido de rotacin antihorario, y signo al sentido horario.

Por supuesto que esta asignacin de signos es arbitraria, y puede ser


modificada si se lo desea: una rotacin horaria es vista como antihoraria
desde detrs del plano.

Ejemplo desarrollado
Se utiliza un torno para elevar baldes con mezcla de albailera. En los dos casos dibujados el balde
con la mezcla tienen una masa de 30 kg, el radio del tambor vale R 1 = 10 cm, y la manija tiene una
longitud de 50 cm.
Despreciando rozamientos calcule la fuerza necesaria para equilibrar el peso del balde cuando la
manija est en cada una de las tres posiciones indicadas, suponiendo que:
a) la fuerza se aplica perpendicularmente a la manija.
b) la fuerza se aplica siempre verticalmente.
60 60
Para los tres casos (b), descomponga la
30 30
fuerza aplicada segn las direcciones
tangencial y normal al desplazamiento posible,
y explique el efecto de cada una de ellas
(a) (b)
sobre el sistema.
255

Desarrollo
a) En todos los casos el peso del balde (P 29,4 N), actuando a lo recta de accin de P
largo de la soga, acta con un brazo de palanca (con respecto al brazo de palanca de P
eje), que es el radio del tambor, de manera que tiene un momento
MP = PR1 = 29,4 0,10 = 2,94 Nm, con sentido positivo
(antihorario).
De manera que la fuerza aplicada debe ser tal que equilibre el
momento MP, es decir debe aplicar un momento en sentido horario
(lo cual est garantizado por la forma en que estn dibujadas las P
fuerzas), de mdulo 2,94 Nm, en todos los casos.
Ahora bien, en los tres casos dibujados en a), la fuerza se aplica perpendicularmente a la manija, es
decir tangencialmente al movimiento posible, y eso significa que su brazo de palanca siempre es la
misma longitud de la manija, independientemente de los ngulos indicados.
Por lo tanto en cualquiera de estos tres casos el mdulo de la fuerza aplicada debe ser tal que:
F 0,50 m = 2,94 Nm
Es decir, F = 29,4 / 0,50 = 5,88 N.
b) Nuevamente debemos averiguar F para que aplique un momento igual a 2,94Nm, pero en cada
caso eso ocurrir con un valor distinto de F. Mostraremos dos formas equivalentes de calcular.
Una forma es buscando los brazos de palanca de cada fuerza.
F1T
F1N
F1
O B1 B3 B O
2

F2 F2
F3N
F3 F3T

Como se ve en la figura de la izquierda, para F1 el brazo de palanca es OB1 = 0,50 m cos 60 = 0,25
m. De manera que F1 = 2,94 / 0,25 = 11,76 N.

Para F2 el brazo de palanca OB2 es la misma palanca, y estamos en la misma condicin del caso a),
de manera que F2 = 5,88 N.

Para F3 el brazo de palanca es OB3 = 0,50 m cos 30 = 0,43 m. De manera que F3 = 2,94 / 0,43 =
6,79 N.
Otra forma de hacer el clculo es descomponiendo cada fuerza segn las direcciones tangencial y
normal al desplazamiento posible, como se muestra en la figura de la derecha. En este caso sabemos
que la nica que contribuye al momento es la componente tangencial, la cual, para producir el
momento buscado, debe valer 5,88 N en todos los casos.
De manera que, para tener esta componente,
F1 debe valer F1 = 5,88 / sen 30 = 11,76 N
F2 coincide con F2T, o sea que debe valer 5,88 N
F3 debe valer F3 = 5,88 / cos 30 = 6,79 N
Por otra parte, la componente normal acta a lo largo de la manija, comprimindola y empujando al eje
en el caso 1, y traccionando la manija y tirando del eje en el caso 3 (en el caso 2 no hay fuerza a lo
largo de la manija).
256

9.3.- Leyes de la dinmica de la rotacin pura


Inercia de la rotacin pura.
Ahora bien, para establecer leyes de una manera lo ms parecida posible a las que ya
conocemos para los movimientos lineales, comencemos considerando el caso ms simple, en
el cual no se aplique ninguna fuerza exterior sobre los puntos del cuerpo fuera del eje. En este
caso, una rotacin no podr iniciarse espontneamente, ni tampoco detenerse: si no se aplican
fuerzas con momento sobre un cuerpo rgido que est rotando, entonces, por inercia, su
rotacin continuar manteniendo constante el valor de la velocidad angular.

Vale aclarar que, en este caso, necesariamente actan fuerzas interiores sobre cada partcula
del cuerpo, ya que cada una describe un movimiento circular, que para mantenerse requiere de
la accin de una fuerza centrpeta. En el cuerpo rotante se desarrollan tensiones cuyo efecto es
aplicar una fuerza centrpeta neta sobre cada partcula. Estas fuerzas, tanto la accin sobre
cada partcula (centrpeta), como la reaccin (centrfuga) de ella sobre las vecinas, tienen la
direccin estrictamente radial, y por ello no tienen momento con respecto al eje, y no
contribuyen de ninguna manera a iniciar, ni a mantener, ni a detener la rotacin.

m3 Sc Sr

v3 F3
F1 v1
F2 m1
m2
v2 (a) (b)
Fig. 9.9: An en el caso en que un cuerpo rota libremente, sin ninguna fuerza exterior aplicada, hay
fuerzas sobre cada una de sus partculas constituyentes. Estas fuerzas son aplicadas sobre cada
partcula por las partculas vecinas, y tienen la direccin hacia el centro, como se ilustra en (a) para tres
partculas genricas con vectores huecos. En (b) se ilustran las tensiones normales, de traccin, que
aparecen tanto en cualquier superficie cilndrica, Sc, como en cualquier plano radial que contenga al eje,
Sr . El cuerpo podra romperse si estas tensiones se hicieran demasiado grandes, y a partir de ese
momento, cada parte continuara con un movimiento diferente.

Ley del Impulso para las rotaciones


Consideremos ahora un cuerpo rgido con un eje que es mantenido fijo por cojinetes ideales
sin rozamiento, que permiten al cuerpo ejecutar slo rotaciones puras. Sobre este cuerpo se
aplican varias fuerzas exteriores en distintos puntos, para lograr una rotacin de determinadas
caractersticas.
Si consideramos la masa mi de una partcula cualquiera del cuerpo, podemos aplicarle la Ley
del Impulso en la direccin tangencial, y decir que en un intervalo de tiempo t, su velocidad
(tangencial) vi sufrir una variacin vi tal que:
FTi t = mi vi
En esta expresin, Fi es la resultante de todas las fuerzas tangenciales sobre la partcula i.
Estas fuerzas son ejercidas por todas las partculas vecinas, ms los agentes exteriores que
acten all, si los hay (notar que los agentes exteriores, en general, actan slo sobre algunos
puntos particulares, mientras que todas las partculas vecinas se ejercen fuerzas mutuas unas
257

con otras); estas fuerzas son consideradas interiores para el cuerpo que rota, aunque sean
exteriores para una partcula determinada.

FT i t FT i t = mi vi
ejercidas ejercidas
por agentes por otras
exteriores partculas

Ahora bien, como el cuerpo es rgido, la velocidad tangencial slo puede cambiar al cambiar
la angular, de manera que el miembro derecho puede ser escrito como mi i . Como nos
interesa escribir este cambio en funcin de los momentos de las fuerzas actuantes, podemos
multiplicar toda la expresin por i :
t F Text ; i i t F T int ; i i = mi i i (9.8)

Ahora bien, en el primer trmino del miembro izquierdo est expresada la suma de los
momentos de las fuerzas exteriores aplicadas sobre cada partcula. El resultado de esa suma
es el momento total de las fuerzas exteriores que estn aplicadas sobre esa partcula, si las hay,
con respecto al eje. Si adems nos planteamos sumar ese resultado sobre todas las partculas,
simplemente tendremos el momento total de las fuerzas exteriores sobre el sistema completo
con respecto al eje.

FText ; i i Mtotal fuerzas exteriores (9.9)


i

En cambio, en el segundo trmino del miembro izquierdo est expresada una suma que
considera los momentos de las fuerzas sobre una partcula, debidas a sus vecinas. Estas
fuerzas, por accin y reaccin, son opuestas a las que ella ejerce sobre sus vecinas. De
manera que si se suman todos estos trminos sobre todas las partculas, el resultado, que es el
momento neto debido a todas las fuerzas interiores, debe ser nulo.
Momento neto de fuerzas interiores

FT int ; i i 0 (9.10)
i

De manera que, sumando ambos miembros sobre todas las partculas obtenemos lo que
podemos considerar como Ley del Impulso para una rotacin pura:

Mfuerzas exteriores t = mi i2 (9.11)


i

Revisemos los trminos de esta expresin, mientras la comparamos con la correspondiente al



movimiento lineal: F t (mv) m v .

Impulso angular.
Sabemos que el momento de las fuerzas es el concepto que juega, para las rotaciones, el papel
que las fuerzas juegan para los movimientos lineales. De manera que M t, es decir el
momento de la fuerza, por el tiempo que acta, juega el papel del impulso para las rotaciones,
al cual denominaremos impulso angular, con respecto al eje correspondiente.
258

Impulso angular = M t (9.12)


Vale decir que as como el impulso lineal es un vector con la orientacin de la fuerza aplicada,
el impulso angular tiene orientacin en el espacio, que es la misma orientacin del momento
aplicado (es decir, con respecto al eje ubicado de determinada manera, con signo ms para los
impulsos en sentido antihorario, y signo menos para los horarios). En el Anexo 9.1 se plantea
cmo representar las rotaciones por medio de vectores para entender, de manera ms sencilla,
el concepto de orientacin aplicado a rotaciones.

Momento de inercia
Si en el movimiento lineal, hacemos el cociente entre el mdulo del impulso aplicado, y el
cambio en la velocidad sufrido por el cuerpo, tenemos una constante caracterstica de la
inercia del cuerpo, que es m:
F t
m
v
Decimos que m caracteriza la inercia (para el movimiento lineal), y hasta a veces, para dar
nfasis, se dice que m es una medida de la inercia, porque a mayor masa, aplicando igual
impulso, se lograr un cambio menor en la velocidad, y viceversa.
Si tratamos de aplicar estos conceptos al caso de la rotacin, encontramos, a partir de (9.11),
que el cociente entre el impulso angular aplicado, y el cambio en la velocidad angular es la
siguiente expresin:
M t
mi i2 (9.13)

En el caso que nos ocupa, en el que el cuerpo es rgido, esta expresin da una constante para
cada cuerpo con un eje elegido: se calcula a partir de su masa y sus medidas (veremos
ejemplos), y no depende de que el cuerpo est en reposo o no, es decir de la velocidad
adquirida.
Esta constante, que debe expresar la inercia que ofrece el cuerpo para rotar alrededor del eje
considerado, recibe, por razones de usos matemticos la denominacin oficial de momento
de inercia, que simbolizaremos con I:

I m i
2
i (9.14)

El momento de inercia es proporcional a la masa, como corresponde a la idea de inercia, pero


adems depende de que la masa est distribuida cerca o lejos del eje. La diferencia esencial
entre la inercia para el movimiento de traslacin, y la inercia para el movimiento de rotacin
consiste en que, para dos cuerpos de la misma masa es necesario aplicar mayor impulso
angular para iniciar o frenar la rotacin del cuerpo que tiene la masa distribuida ms lejos del
eje, mientras que la distribucin de la masa no interesa en el movimiento de traslacin.
259

Cantidad de movimiento angular


Para el movimiento de rotacin, el concepto correspondiente a la cantidad de movimiento est
dado por la cantidad de movimiento angular, que designaremos con J.
Para este concepto vale aclarar que hay algunos aspectos complejos que no pretenderemos
abarcar, y que en este lugar nos vamos a limitar a los casos simples en los que el eje de
rotacin es a la vez eje de simetra del cuerpo (para otros casos habra que decir que lo que
vamos a definir sera slo una componente referida al eje, de un ente algo ms complejo).
Teniendo en cuenta esta aclaracin, para un cuerpo slido en rotacin pura definimos la
cantidad de movimiento angular como el momento de la cantidad de movimiento con respecto
al eje. Esto significa, para cada partcula del cuerpo, multiplicar el mdulo del vector cantidad
de movimiento, m vi , por su brazo de palanca, o, distancia al eje, i .
As, para todo el conjunto de partculas del cuerpo, tendremos:
J = mi vi i = mi i i = mi i2 (9.15)
Es decir, sacando factor comn de la sumatoria, y utilizando la definicin de momento de
inercia (9.14):

J=I (9.16)

El concepto de momento de inercia nos permite escribir para la rotacin, una expresin
totalmente similar a la del movimiento lineal. Con esta notacin, volviendo a la expresin
(9.11) de la Ley del Impulso para las rotaciones puras, encontramos que sta tambin adquiere
una forma totalmente similar a la de las traslaciones:

Ley del Impulso de las rotaciones puras

M t = J

= I (9.17)

Ejemplo desarrollado
Un volante cuyo momento de inercia vale I = 0,2 kgm, est en reposo montado sobre cojinetes sin
rozamiento. En un instante dado se le aplica un momento de M = 2 Nm en el sentido indicado, durante
5 segundos. Luego se suspende la aplicacin de M.
Calcule la cantidad de movimiento angular y la velocidad angular que adquiere el
volante al cabo de los 5 segundos de aplicacin del momento. Explique qu
sucede despus.
Desarrollo
Aplicando la Ley del Impulso obtenemos la cantidad de movimiento angular adquirida:
J = M t = 2 Nm 5 s = 10 joules
Dado que J = I , entonces la velocidad angular adquirida es:
= J / I = 10 Js / 0,2 kgm = 50 1/s (es decir 50 rad/s).
Al suspenderse la aplicacin del momento impulsor, dado que es una situacin ideal sin rozamiento, el
dispositivo continuar girando indefinidamente con esa velocidad angular inalterada.
260

Trabajo y energa cintica de rotacin.


Utilizando los conceptos desarrollados hasta aqu es posible escribir expresiones para el
teorema del trabajo y la energa cintica en las rotaciones, adaptadas a las variables angulares,
totalmente similares a las del movimiento lineal.
Trabajo en una rotacin pura
Ya sabemos que el trabajo hecho por una fuerza, en general, puede calcularse con la expresin
(8.1), W = FT d. Si ahora nos referimos a una fuerza que se aplica sobre un punto de un cuerpo
rgido que tiene un eje fijo, tenemos que cuando el punto haya recorrido una distancia d a lo
largo de una circunferencia de radio , podemos escribir:
d
ngulo girado (en radianes):

Momento aplicado: M = FT
Con lo cual:
d
W = FT d = FT

= M (9.18)
Esta ltima expresin no es imprescindible, ya que es totalmente equivalente a FT d, y
cualquiera de las dos puede aplicarse a una rotacin. Pero tiene el valor conceptual, y hasta
esttico, de permitir escribir, para las rotaciones, expresiones en variables angulares,
totalmente similares a las expresiones en variables lineales que hemos visto para el
movimiento de traslacin.
Energa cintica en una rotacin pura
Tenemos que la energa cintica de cualquier sistema, y eso incluye a un cuerpo que rota,
siempre puede ser expresada como:
mi vi2
Ec 2
Veamos cmo se puede reescribir esto mismo en trminos de las variables angulares.
Escribamos las velocidades en la sumatoria en funcin de la nica velocidad angular, ,
sustituyendo vi = i :
mi vi2 mi 2 i2
2
2
Si en esta expresin extraemos 2 como factor comn fuera de la sumatoria, llegamos a la
expresin buscada:


1
Ec 2 mi i2
2
I 2
= (9.19)
2
Teorema del trabajo y la energa cintica
Si repetimos con las variables angulares el mismo procedimiento hecho con las variables
lineales para demostrar este teorema, llegamos exactamente a la misma expresin W = Ec,
escrita ahora como:
261

M = ( I 2)
Donde M es el momento resultante de todos los momentos de las fuerzas exteriores con
respecto al eje del cuerpo, W = M es el trabajo resultante de todas las fuerzas exteriores, y
Ec = I 2, es la energa cintica de rotacin de este cuerpo.

Ejemplo desarrollado
Una partcula de masa m = 2 kg se mueve describiendo circunferencias de 3 m de radio, por estar
sujeta al extremo de un hilo ideal (sin masa) de esa longitud, cuyo otro extremo est fijo en el punto O.
Se supone un situacin ideal en la cual no acta la gravedad ni hay rozamientos.
Inicialmente la partcula tiene una velocidad (tangencial) de 9 m/s, y un
agente le aplica una fuerza tangencial de 5 N durante 8 segundos, que Fext
luego se suspende. v
R
a) Utilizando la dinmica del movimiento lineal, calcule la velocidad
O
final de la partcula, la distancia recorrida durante la aplicacin de la
fuerza mencionada, y corrobore que el trabajo realizado por el agente
coincide con el aumento de energa cintica.
b) Muestre que los mismos resultados se obtienen empleando los
conceptos y frmulas del movimiento de rotacin.
Desarrollo
a) A partir de la Ley del Impulso para la fuerza tangencial obtenemos la velocidad final vf = v0 + F t /
m = 9 + 40 / 2 = 29 m/s.
Dado que el movimiento tangencial resulta uniformemente acelerado (ya que la fuerza tangencial es
constante), podemos calcular la distancia recorrida a lo largo de la circunferencia durante los 10
segundos de aplicacin de la fuerza, aplicando d = vm t , siendo la velocidad media: vm = (9 + 29)/2 =
19 m/s. De esto resulta que la distancia recorrida es 198 = 152 m.
Con este valor de distancia el trabajo realizado resulta W = 5152 = 760 J; por otra parte, siendo la
energa cintica inicial 292 = 81 J, y la final 2292 = 841 J, se verifica perfectamente W = Ec.

b) Veamos ahora todo en trminos de variables angulares.


El momento de inercia de la partcula respecto de O es I = 2 kg (3 m)2 = 18 kgm2 (ver (9.20).
Inicialmente tiene una velocidad angular 0 = 9 (m/s) / 3 m = 3 rad/s, y correspondientemente, una
cantidad de movimiento angular inicial J0 = I 0 = 54 Js, y una energa cintica Ec = I 02 = 81 J.
La fuerza se aplica con un momento M = 53 = 15 Nm, y en 8 s aplica un impulso angular igual a
158 = 120 Js.
Aplicando la Ley del Impulso para las rotaciones podemos calcular la candidad de movimiento angular
final Jf = J0 + 120 Js = 174 Js. Esto significa que la velocidad angular final es f = 174 / 18 = 9,67 1/s,
y la energa cintica final 841 J.
Por otra parte, el trabajo es W = M , y el ngulo girado es = med t, siendo med = (3 +
9,67) = 6,33 1/s. De manera que = 50,67 rad (que es lo que se hubiera obtenido dividiendo el arco
de 152 m por el radio de 3m), y W = 15 50,67 = 760 J.
Casos de momentos de inercia
Para calcular el momento de inercia de un cuerpo a partir de sus medidas se debe aplicar la
relacin (9.14): I = mi i2. Excepto para el caso 1, los mtodos matemticos para evaluar esta
262

suma no estn a nuestro alcance aqu, pero podremos resumir algunas ideas bsicas
importantes, y algunas expresiones para cuerpos de formas tpicas.
Caso 1: Masa puntual en el extremo de una varilla.
Cuando hay un cuerpo girando sujeto en el extremo de una varilla o de un hilo de longitud L,
suponiendo que el cuerpo tiene un tamao despreciable frente a L, y que la masa de la varilla
o hilo es despreciable frente a la masa del cuerpo, tenemos un caso trivial en el cual, para la
sumatoria planteada en (9.14), todas las partculas que pueden considerarse tienen
aproximadamente la misma distancia L al eje. Por lo tanto, L2 resulta factor comn de la suma
de las masas, y queda:
I = m L2 (9.20)
La misma idea tambin se aplica a cuerpos cuya masa se mantiene a distancia R
(suficientemente grande) del eje, por medio de varillas o elementos cuya masa puede
despreciarse, como el pndulo, cualquier objeto no muy grande que gira en el extremo de una
cuerda, un anillo, una rueda de bicicleta y otros (figura 9.6).

m m M
L R R

O M=4m

I = m L2 I = M R2 I = M R2

Fig. 9.10: Para los casos en los cuales la masa del cuerpo puede considerarse toda a cierta
distancia fija del eje, el momento de inercia vale masa distancia2

Caso 2: Cilindro, o disco, con respecto a su eje.

R R
1
I MR2 (9.21)
2

Fig. 9.11: Para un cilindro o un disco el momento de inercia con respecto al eje vale f masa
R2, donde f es un factor menor que 1 (en este caso, ), ya que al efectuar la suma (9.14) todos
los elementos de masa, excepto los del borde, tienen distancias i al eje menores que R.

Caso 3: Esfera, con respecto a un eje que pasa por el centro.

R
I 2
M R 2 (9.22)
5

Fig. 9.12: Para una esfera sucede lo mismo dicho en la figura 9.11 para el cilindro, excepto que el
factor f es menor an (0,5 para el cilindro y 0,4 para la esfera), porque la esfera concentra ms
masa cerca del eje, a igualdad de otras condiciones.
263

Caso 4: Eje paralelo a otro con respecto al cual I es conocido.


Si se conoce el momento de inercia I0 con respecto al eje e0 que pasa por el centro de masa, el
momento I con respecto a otro eje e, paralelo al anterior, a distancia d del mismo, vale (esto se
conoce como teorema de Steiner):
m e0
CM
I = I 0 + m d2 (9.23)
d
e

Fig. 9.13: El teorema de Steiner ayuda en los casos en que el eje de rotacin est
fuera del cuerpo.

Ejemplo desarrollado 1
Aplicar (9.23) a una esfera de 2 cm de radio y 300 g de masa que gira sujeta al extremo de un hilo de
60 cm de largo, y determinar cunto error se hubiera introducido aplicando (9.20).
Desarrollo
Aplicamos (9.23), con d = 62 cm. Para calcular I0 aplicamos (9.22): I0 = 0,40,30kg(2cm)2 0,48
kgcm2. Ahora, aplicamos (9.23): I = I0 + 0,30kg(62cm)2 0,48 + 1153,2 kgcm2 1153,7 kgcm2.
Se ve claramente que el segundo sumando corresponde a la expresin (9.20), y que I0 0,48 kgcm2,
puede ser despreciado introduciendo un error < 0,05%.

Ejemplo desarrollado 2
Consideremos un volante de hierro (7850 kg/m3) de 30 cm de dimetro y 1,2 cm de espesor, montado
sobre un eje tambin cilndrico de hierro, de 2 cm de dimetro y 30 cm de longitud, montado sobre
cojinetes perfectos sin rozamiento. Solidario con este disco hay un pequeo tambor de 2 cm de radio
en el que se enrolla un hilo que se utiliza para hacer girar el sistema por medio de una fuerza que tira
de l.
R = 15 cm
F

r = 2 cm
F

a) Calcular el momento de inercia del disco. Explicar si es necesario considerar las contribuciones del
eje y del tambor, o si pueden despreciarse.
b) Calcular el momento y el impulso angular aplicado al disco si se tira del hilo con una fuerza de 20 N
durante 6 s.
c) Suponiendo que inicialmente el disco est en reposo, calcular la velocidad angular que adquiere en
este proceso, y a partir de ella obtener la longitud de hilo desenrollada bajo la accin de la fuerza.
d) A partir de la longitud de hilo desenrollada calcular el trabajo hecho por la fuerza, y verificar que
coincide con la energa cintica adquirida.
e) Explicar cmo es el movimiento luego de suspenderse la aplicacin de la fuerza.
264

Desarrollo
a) Tanto para el disco, como para el eje y el tambor se aplica la frmula (9.21) del cilindro. Es de
prever que las contribuciones del eje y del tambor sern despreciables (lo que mostraremos con unos
clculos aproximados).
Necesitamos las masas de cada parte.
Mdisco = densidad volumen = 7850 (kg/m3) (0,15 m)2 0,012 m 6,66 kg
Meje = 7850 (kg/m3) (0,01 m)2 0,30 m 0,74 kg
No tenemos elementos para calcular la masa del tambor, pero veremos que no es necesaria porque no
contribuir apreciablemente al momento de inercia.
Idisco 6,66 kg (0,15 m)2 / 2 0,075 kgm2.
Ieje 0,74 kg (0,01m)2 / 2 0,000037 kgm2, totalmente despreciable frente al anterior. Asimismo, se
puede estimar que el momento de inercia del tambor ser comparable al del eje, tal vez menor, y que,
por lo tanto, no ser necesario calcularlo.
b) El brazo de palanca de la fuerza aplicada es el radio del tambor, 2 cm, como se ve en la figura.
Entonces, el momento aplicado es: M = 20 N2 cm = 0,40 Nm.
El impulso angular con respecto al eje resulta: M t = 0,40 Nm 6 s = 2,4 Js.
c) A partir de la ley del impulso, sabemos que la cantidad de movimiento angular adquirida es igual al
impulso angular aplicado: J = 2,4 Js. Dado que inicialmente el disco estaba en reposo, sta es la
cantidad de movimiento angular final: J(6s) = J = 2,4 Js = I . De manera que = 2,4 Js / 0,075
kg.m2 = 32 1/s.
Dado que la velocidad angular vara uniformemente de 0 a 32 1/s, la velocidad del hilo, que es la del
contorno del tambor (v = r), pasa de 0 a 0,64 m/s, y su extremo recorre una distancia d = v m t =
0,326 = 1,92 m.
d) El trabajo hecho por la fuerza resulta W = 20 N 1,92 m = 38,4 J; por otra parte, la energa cintica
vale: Ec = 0,075 kgm2 (32 (1/s))2 / 2 = 38,4 J (hay total acuerdo).
e) Por inercia, luego de suspenderse la aplicacin de la fuerza, suponiendo que no hay rozamientos, la
rotacin continuar con velocidad angular constante de 32 radianes por segundo.

La conservacin de la cantidad de movimiento angular


A partir de la Ley del Impulso para las rotaciones (9.17), es elemental plantear que si el
momento resultante de las fuerzas exteriores es nulo, la cantidad de movimiento angular debe
ser constante:
Si M = 0, entonces J = constante.
En el movimiento lineal, si la fuerza resultante es nula, debe conservarse la cantidad de

movimiento p m v . Y dado que la masa es necesariamente constante, esto significa que

debe conservarse v .
En el movimiento angular hay una gran diferencia, porque J = I , pero I puede variar si el
cuerpo cambia de forma. Es decir, para cuerpos absolutamente rgidos que giran alrededor de
un eje fijo, la conservacin de J es lo mismo que la conservacin de . Estamos en un caso
absolutamente similar a los casos de movimiento lineal.
265

Pero es fcil tener casos de movimientos de cuerpos formados por partes que pueden acercarse
o alejarse, o acomodarse de diversas maneras, variando I. En estos casos, debe variar
automticamente de manera de mantener constante el producto I .

Un ejemplo analizado: Acrbatas y patinadores.


Cuando se tiene un cuerpo que puede deformarse, la ausencia de momento aplicado con
respecto al eje de rotacin, implica la conservacin de la cantidad I , y dado que I se puede
disminuir (aumentar) acercando (alejando) partes al eje, resulta que se puede variar la
velocidad angular, que es inversamente proporcional a I:
1 I 2

2 I1
As tenemos el caso de patinadores que suelen iniciar giros alrededor de un eje vertical con los
brazos abiertos, y luego logran aumentar enormemente la velocidad de rotacin acercando los
brazos al cuerpo para disminuir el momento de inercia.
Es notable en este caso (y en muchos otros similares), que si bien el sistema no est aislado de
manera absoluta, lo est respecto de acciones referidas al eje de rotacin, por ello se da este
caso de conservacin de J.
Pero no se conserva la energa cintica! Para verlo basta intentar con algunas cifras: por
ejemplo, supongamos que I2 = I1 / 3, de manera que 2 = 3 1. Si calculamos la energa
cintica final: Ec2 = I2 22 = (I1/3) (31)2 = 3 Ec1.
De dnde sali la energa cintica extra? Del trabajo de las fuerzas interiores: el patinador
debe aplicar una fuerza para vencer la fuerza centrfuga y acercar sus brazos o partes al eje.
Realiza un trabajo positivo (a expensas de su reserva interna de energa) que se traduce en el
aumento de energa cintica de rotacin.
Adems, es fcil calcular que la fuerza centrfuga aumenta mucho para las partes que se
acercan al eje, porque aumenta su velocidad lineal, y tambin disminuye el radio, de manera
que el acrbata debe aplicar una gran fuerza para acercar estas partes, y ello resulta en un gran
aumento de energa cintica. Todo puede terminar en un gran desastre si el que gira no est
muy bien entrenado para mantener cada parte en el lugar y con la alineacin exacta!

Actividad experimental ilustrativa


Conseguir un cordel fuerte de algo ms de un metro de longitud y sujetar de un extremo algn
cuerpo de unos 40 o 50 gramos de masa (puede ser un llavero con dos o tres llaves).
Pasar el cordel por dentro de un tubo (puede ser el cuerpo de una birome) que habr que
sostener verticalmente en alto con una mano, revolendolo suavemente de manera que el
cuerpo en el extremo describa un movimiento circular horizontal, como se muestra. Mientras
tanto el otro extremo del hilo debe ser mantenido fijo con la otra mano.
266

Cuando el movimiento tenga una velocidad suficiente como para mantenerse suavemente sin
necesidad de mover ms el tubito, apretando fuertemente a ste para mantenerlo lo ms
inmvil posible, se tira suave pero sostenidamente con la otra mano del extremo del hilo para
ir acortando el radio de la circunferencia.
Es claro que la fuerza aplicada por el hilo al cuerpo est alineada con el centro, por lo que no
aplica momento, y se debe plantear la conservacin de la cantidad de movimiento angular.
Esta conservacin, al disminuir el radio, se manifestar como un marcado aumento de la
velocidad del cuerpo. El movimiento podr llegar a transformase en un giro vertiginoso,
mientras el aumento de la fuerza centrfuga har imposible mantener inmvil el tubo, que se
sacudir violentamente por ms fuerza que se aplique para impedirlo.
El efecto es muy notable a pesar del rozamiento que se hace muy fuerte en el borde del tubo.
Para evitar accidentes se recomienda comenzar a experimentar con masas pequeas y
movimientos suaves.

9.4.- Rotacin ms traslacin


El movimiento ms general posible de un cuerpo slido, siempre se puede expresar como una
rotacin alrededor de un eje que pasa por el centro de masa, ms una traslacin pura con la
velocidad del centro de masa.
Segn la complejidad de cada situacin, puede ocurrir que el eje de rotacin se traslade
paralelamente a s mismo, o no; tambin la velocidad del centro de masa, tanto como la
velocidad de rotacin, pueden ser constantes, o variar de maneras complicadas.
En cualquier caso, siempre podremos tratar el movimiento aplicando las siguientes leyes:
a) Movimiento Lineal
Para cualquier movimiento de un cuerpo rgido, la parte del movimiento de traslacin siempre
se trata con la Ley del Impulso:

FR t m vCM
Donde

FR es la resultante de todas las fuerzas exteriores
m es la masa del cuerpo

vCM es la velocidad del centro de masa
En el movimiento de traslacin pura todas las partculas del cuerpo tienen la misma velocidad

que el centro de masa, vCM . Esto equivale a decir que tenemos una nica partcula con toda la
masa, concentrada en el centro de masa. Es lo que hicimos al desarrollar todos los temas de
dinmica de movimientos lineales, todo ello tiene vigencia para esto. Adems, sabemos que
esta ley, referida al movimiento del centro de masa, es vlida para cualquier sistema, sea
rgido o no.
267

b) Movimiento intrnseco
La parte del movimiento intrnseco de un cuerpo rgido siempre es una rotacin, y se trata con
la Ley del Impulso para las rotaciones:
MCM t = JCM
Donde
MCM es el momento total de las fuerzas exteriores con respecto al centro de masa
JCM es la cantidad de movimiento angular intrnseco, es decir con respecto al centro de
masa.
c) Energa cintica total
La energa cintica total (de un cuerpo slido) siempre puede escribirse como la suma de dos
trminos:
Ec = m vCM2 + ICM 2
= EcT + EcR
Donde:
EcT = m vCM2, es la energa cintica de traslacin, correspondiente a toda la masa m
desplazndose con la velocidad del centro de masa, y EcR = ICM 2 , es la energa cintica
de rotacin, siendo ICM el momento de inercia con respecto a un eje que pasa por el centro de
masa.

Ejemplo desarrollado
En un juego de bolos se arroja un bolo de 16 cm de dimetro y 2 kg de masa con una velocidad de 10
m/s. El bolo es arrojado horizontalmente, sin girar sobre s mismo, y rasante con el piso (de manera
que toma contacto inmediatamente con l, sin golpear). As que inicialmente el movimiento es de
traslacin pura, y en el punto de contacto, donde el bolo desliza, acta una fuerza de rozamiento de 2
N.
Describir todos los aspectos posibles del movimiento, con todos los clculos correspondientes.
Desarrollo

RN RN RN
0 = 0 f
v0
v vf

Fr situacin inicial: Fr intermedio: situacin final:


rodadura con rodadura sin
P traslacin pura P deslizamiento P deslizamiento

Inicialmente, desde que el bolo toma contacto con la pista, tenemos la accin de una fuerza de
rozamiento constante, que a su vez es la fuerza resultante, ya que en la direccin vertical el peso se
equilibra con la reaccin normal de la pista.
Dado que el bolo es un cuerpo rgido debemos considerar, por un lado el movimiento del centro de
masa (que es simplemente el centro geomtrico del bolo), y por otro lado la rotacin alrededor del
centro.
268

Para el movimiento del centro tenemos que considerar la accin de la fuerza resultante, que es el
rozamiento, constante, de 2 N hacia atrs. Resulta que tenemos un MRUV, con aceleracin a = 2N /
2kg = 1 m/s2, en sentido contrario al movimiento.
Por otra parte, para la rotacin tenemos que considerar que slo la fuerza de rozamiento tiene
momento con respecto al centro. Dicho momento es constante, vale 2 N 0,08 m = 0,16 Nm, y acta
en el sentido de hacer rodar el bolo hacia delante.
Esto nos dice que mientras el avance se va retardando uniformemente, la rotacin se va acelerando
uniformemente, de manera que el movimiento comienza siendo de traslacin pura, pero contina
siendo una superposicin de traslacin retardada, con rotacin acelerada, deslizando en el punto de
contacto. Descripto desde un sistema en el que el centro del bolo est en reposo, la parte inferior del
bolo pasa hacia atrs con velocidad R (cada vez mayor a medida que aumenta ), y el piso pasa
tambin hacia atrs a velocidad cada vez menor (a medida que el bolo se frena). En algn momento,
esas velocidades se igualarn, y en ese instante desaparecer el deslizamiento.
En ese instante tambin deber desaparecer la fuerza de rozamiento.
La fuerza de rozamiento desaparece buscamente, en un instante! Esto es muy notable, y para
deducirlo haremos un razonamiento que se denomina por el absurdo, y consiste en razonar
mostrando que si suponemos que lo que queremos demostrar no sucede, entonces se llega a una
conclusin absurda.
Es decir, para probar que la fuerza de rozamiento debe desaparecer en el instante en que la velocidad
del centro se iguala con R, supongamos que se llega a esa condicin, y la fuerza de rozamiento no
desaparece en ese mismo instante, sino que contina actuando (hacia atrs sobre el bolo) una cierta
cantidad de tiempo, aunque sea muy pequea.
Entonces, como la fuerza de rozamiento actuante sobre la superficie inferior del bolo es un vector que
apunta hacia atrs, concluimos que un instante despus el bolo debera rotar ms rpido, y avanzar
ms despacio (que lo que corresponde a la relacin v = R). Esto significa que su superficie inferior
debera haber comenzado a deslizar hacia atrs respecto de la pista.
Pero si la superficie del bolo deslizara hacia atrs respecto de la pista, sta debera aplicarle una
fuerza de rozamiento en sentido contrario, oponindose al deslizamiento, es decir hacia delante.
Como se ve, habramos llegado a una contradiccin, o absurdo, porque partimos de suponer que la
fuerza continuaba actuando hacia atrs.
A partir del instante en que desaparece el rozamiento, el bolo contina rodando y avanzando sin
deslizar, uniformemente.
Atencin: antes de seguir vale aclarar que el bolo rodando tambin sufre cierto tipo de fuerza de
frenado por parte del piso, pero que no es de rozamiento, dado que no hay deslizamiento de las
superficies. Esta fuerza puede ser extremadamente dbil si las superficies son suficientemente lisas y
duras, ya que tiene que ver con el pequesimo aplastamiento que se produce en el punto de contacto,
el cual ya hemos dicho (Captulo 3) que nunca puede reducirse a un punto geomtrico. No nos
ocuparemos de ella aqu.
Habiendo aclarado entonces que no consideraremos la fuerza de frenado por rodadura, la descripcin
que hemos hecho nos indica qu es lo que puede ser interesante calcular. Por ejemplo:
Cul es la velocidad final que luego se mantendr constante?,
cunto tiempo debe transcurrir hasta que se llegue a la condicin de rodadura sin deslizamiento?,
qu distancia se recorre en este proceso?,
cunta energa se pierde por rozamiento?
Comencemos tratando de calcular la velocidad final.
269

La condicin de rodadura sin deslizamiento es que la velocidad del centro sea igual a la velocidad del
contorno del bolo con relacin al centro, R (les coloco el subndice f porque estos sern los valores
finales):
vf = f R
Ahora bien, siendo t = 0 el instante del contacto inicial con el piso, aplicamos Ley del Impulso para la
velocidad del centro:
m v(t) = m v0 Fr t
Mientras tanto, aplicamos Ley del impulso de Rotacin para la velocidad de rotacin ( inicial es nula):
I (t) = M t = Fr R t
En el instante tf en que se alcanza la condicin de rodadura perfecta se deben cumplir:
vf = f R
m vf = m v 0 F t f
I f = F R tf
Eliminando tf y f, se obtiene:
1 1
vf = v0 = v0 = 7,14 m/s
I 1,4
1
m R2
2 kg 10 m / s 0,4
Con este valor se obtiene directamente f = 89,3 1/s, y tf = = 2,86 s.
2N 1,4
La distancia recorrida en ese tiempo es: df = (v0 + vf) tf = 24,5 m.
La energa cintica inicial es de traslacin pura: m v 02 = 100 J, mientras que la final es de traslacin
y rotacin: m vf2 + I f2 = 51,0 J + 20,4 J = 71,4 J , de manera que la energa cintica perdida es
28,6 J.
Ntese que si multiplicamos la fuerza de rozamiento por la distancia recorrida mientras ella acta
obtenemos: 2 N24,5 m = 49 J, que no es la energa perdida, sino slo la energa cintica de
traslacin perdida. Esto se entiende porque dado que la bola rodaba, el deslizamiento entre las
superficies nunca lleg a ser 24,5 m, y una parte del trabajo de la fuerza de rozamiento no se perdi en
friccin, sino que se emple en producir la rotacin.
270

ANEXO 9.1:

Vectores Axiales
Una rotacin queda determinada en el espacio si se indica el eje y el sentido de rotacin
alrededor del mismo.
La ubicacin del eje determina al mismo tiempo todos los planos, perpendiculares a l, en los
cuales se desplazan los puntos del cuerpo en rotacin, asi como los centros de todas las
trayectorias circulares de stos.
Esto significa que el problema de indicar una rotacin en el espacio es equivalente al
problema de indicar su eje, y uno de dos sentidos posibles asociados con l.
De manera que si, por medio de alguna convencin asociamos (arbitrariamente) un sentido de
circulacin alrededor del eje, con un sentido a lo largo del mismo, podremos utilizar un vector
para indicar variables de la rotacin, de la siguiente manera:
Con la direccin del vector indicamos exactamente la direccin del eje de rotacin;
Con una convencin arbitraria decimos que cada sentido del vector indica un sentido de
giro alrededor del mismo.
Hay dos convenciones posibles para asociar el sentido del vector con el sentido de la rotacin
que representa, y stas seran: o la convencin dextrgira (que significa del giro a
derechas, o de la mano derecha), o la convencin levgira, o siniestrgira (que
significa del giro a izquierdas, o de la mano izquierda).
Los vectores utilizados para representar variables angulares se denominan vectores axiales
(axial significa perteneciente o relativo a un eje). Utilizaremos siempre la convencin
dextrgira, que es la siguiente:
Si se toma el vector axial con la mano derecha, con el pulgar apuntando como el vector, los
dedos al cerrarse, quedan indicando el sentido de circulacin correspondiente alrededor del
mismo.

Sentido
de
circulacin

Mano derecha
Fig. A9.1.1: Convencin dextrgira.
271

Otra forma de definir esta misma convencin dextrgira es decir que observando el plano de
la rotacin desde el lado desde el cual la rotacin se ve con el sentido antihorario, se debe ver
el vector correspondiente salir hacia el observador. Se ilustra en en la figura siguiente.

Vector axial visto desde la punta Vista superior:


(saliente de la hoja hacia el observador) Vector axial saliente,
circulacin antihoraria
Vector axial

circulacin Situacin en el espacio

Vista inferior:
Vector axial visto desde la cola Vector axial entrante,
(entrante hacia la hoja desde el observador) circulacin horaria
Fig. A9.1.2: La circulacin es dextrgira respecto del sentido del vector axial, si
corresponde a alguna de estas figuras. Notar que la misma rotacin se puede ver
como horaria o antihoraria, segn desde qu extremo del eje se la observe.

Debe tenerse siempre presente que:


Un vector axial no indica en absoluto algo que ocurra a lo largo del mismo, hacia uno u
otro de sus extremos, sino que indica rotacin alrededor del mismo, en uno u otro sentido
de circulacin.
Una vez que elegimos una convencin, ya sea la dextrgira, o la levgira, para representar
una circulacin con un vector axial, decimos que este sentido de circulacin es dextrgiro o
levgiro con respecto al sentido del vector axial.

Con ayuda de los vectores axiales, entonces definimos:


Los ngulos se representan con vectores axiales:
Vector axial representativo de

Representacin tradicional del ngulo en el plano


Fig. A9.1.3: Representacin de un ngulo con un vector axial.

La velocidad angular, la cantidad de movimiento angular, se representan con vectores


axiales
La velocidad angular se define dividiendo el desplazamiento angular por el tiempo, de manera

que si el desplazamiento angular se expresa con un vector: 2 1 , entonces la

velocidad angular resulta el correspondiente vector axial . Si adems consideramos que la
cantidad de movimiento angular (en una rotacin pura, y para los casos simples en que el eje
272

es eje de simetra) se define como el producto del momento de inercia por la velocidad
angular, tambin tendremos naturalmente un vector axial para representar la cantidad de

movimiento angular J :

J


t


J I
Fig. A9.1.4: Representacin de velocidad y cantidad de
movimiento angulares con vectores axiales.

El momento de las fuerzas aplicadas se representa con un vector axial


Si se considera una fuerza actuando en el plano de una rotacin, podemos representar el
momento con un vector axial cuyo mdulo es M = F b, cuya direccin es la del eje, es decir
perpendicular al plano, y con el sentido dado por la convencin dextrgira:

M M
F b
F b

Fig. A9.1.5: Representacin del momento de una fuerza con vectores axiales.

Como aplicacin de estos conceptos podemos escribir la Ley del Impulso de las
rotaciones puras con ayuda de los vectores axiales correspondientes:

M t I J
Ejemplo desarrollado
Considere un volante de hierro montado sobre un eje cilndrico de hierro, con cojinetes perfectos sin
rozamiento. Solidario con este disco hay un pequeo tambor en el cual se enrolla un hilo de 1 m de
lonfitud, del que pende una pesa de masa m = 1 kg.

(a) (b)

a) Considere la situacin ilustrada en (a), cuando el hilo lleva desenrollados 60 cm, habiendo partido la
pesa desde una situacin inicial en reposo, con el hilo totalmente enrollado. Dibuje los vectores axiales
momento aplicado sobre el disco, velocidad angular y cantidad de movimiento angular del disco, en el
instante considerado.
273

b) Considere la situacin ilustrada en (b), luego de que el hilo se desenroll totalmente, y por estar
sujeto al tambor, ha comenzado a enrollarse en sentido contrario debido a que el disco, por inercia, ha
continuado girando. Dibuje los vectores axiales momento aplicado sobre el disco, velocidad angular y
cantidad de movimiento angular del disco, en el instante considerado.
Desarrollo
a) Dado que el disco parte del reposo, la velocidad y la cantidad de movimiento angulares que
adquiere, tienen el mismo sentido que el momento aplicado, y segn la regla de la mano derecha, son

tres vectores sobre el eje, hacia la derecha: ; J ; M
b) Ahora el momento aplicado cambia de sentido, tendiendo a frenar la rotacin, que contina con el

mismo sentido anterior. Por lo tanto los vectores son: ; J ; M
274

Anexo 9.2:

Equilibrio de momentos, cuplas, y fuerzas paralelas.

Equilibrio, y equilibrio de rotacin


En el captulo de Esttica hemos explorado las caractersticas de algunos sistemas sencillos de

fuerzas planteando la llamada Primer Condicin de Equilibrio ( Fi 0 ). Esta condicin
solamente tiene en cuenta las componentes de las fuerzas, y no el lugar en el que se aplican.
Ahora sabemos que la misma fuerza aplicada en distintos lugares en general tiene diferente
momento, y por lo tanto diferente capacidad para influir sobre posibles rotaciones de los
cuerpos.
As, por ejemplo, consideremos el caso, muy frecuente, en que se trata de hacer girar un
volante o cuerpo similar aplicndole en puntos diametralmente opuestos dos fuerzas opuestas
de igual mdulo (fig. A9.2.1).

Fig. A9.2.1 : Estas fuerzas, siendo vectores opuestos entre s, cancelan mutuamente sus
tendencias a producir traslacin del sistema hacia algn lado, pero suman sus efectos
rotatorios, ya que, con respecto al eje, ambas tienden a producir rotacin en el mismo sentido.
Este sistema de fuerzas tiene resultante nula, es decir que cumple con la llamada primer

condicin de equilibrio ( Fi 0 ), y sin embargo es claro que no est en equilibrio desde el
punto de vista de los momentos aplicados, que pueden impulsar la rotacin.
Por otra parte, tambin podemos imaginar que dada una fuerza aplicada sobre un cuerpo que
tiene un eje de rotacin, la rotacin puede ser impedida con otra fuerza de diferente mdulo y
direccin, si se la aplica con el brazo de palanca adecuado. De manera que dos fuerzas que no
cumplen con la primer condicin de equilibrio, s pueden equilibrar sus momentos, es decir
sus efectos sobre posibles rotaciones.
Todo esto nos dice que para que un sistema de fuerzas est en equilibrio no es suficiente con
que se cumpla la primer condicin que ya hemos visto en Esttica (resultante nula), sino que
adems tambin debe cumplirse una segunda condicin similar, referida a los momentos de
las fuerzas con respecto a algn eje.

Es decir, dado el sistema de fuerzas { Fi }, actuantes sobre un cuerpo, decimos que cumplen
con la segunda condicin de equilibrio, o Condicin de equilibrio de los momentos, si,
con respecto al punto (eje) O que se elija, el momento resultante es nulo:
275

M F ;O 0
i
(A9.2.1)

Si esta condicin se cumple se considera que el sistema de fuerzas tiene potencialidad nula
para influir sobre la posible rotacin del cuerpo alrededor del eje considerado.

Condiciones de Equilibrio de un sistema de fuerzas


Decimos que un sistema de fuerzas est en equilibrio si cumple con las dos condiciones de
equilibrio que hemos visto.

1er Condicin: F 0i

2da Condicin: M F ;O 0
i

Nota: Para resolver situaciones prcticas es importante conocer algunas consecuencias que se
deducen fcilmente de las condiciones y razonamientos presentados hasta aqu. Dos
afirmaciones tiles (que por razones de espacio no demostraremos) son las siguientes:

a) Si se cumple la primer condicin ( FR = 0), y los momentos estn equilibrados con respecto
a algn punto elegido como eje, entonces los momentos estn equilibrados con respecto a
cualquier otro punto que se elija.
Esto significa que, en la prctica, cuando se resuelve un problema de equilibrio, planteando la
primer condicin, luego hay libertad para elegir el punto que se considere ms cmodo como
eje para la segunda condicin, ya que cualquier punto ser equivalente para esto.

b) Con ciertos criterios sencillos es posible elegir puntos diferentes como eje, tales que si los
momentos estn equilibrados con respecto a dos de estos puntos, entonces necesariamente se
cumple la primer condicin.
Esto significa en la prctica, que los problemas de equilibrio se pueden resolver planteando la
segunda condicin con respecto a dos puntos diferentes, sin necesidad de plantear la primer
condicin, que se cumplir automticamente.

Ubicacin de la equilibrante y de la resultante de un sistema de fuerzas


Cuando un sistema de fuerzas no est en equilibrio se puede definir una fuerza equilibrante de
manera que, incluyndola en el sistema, lo deja en equilibrio.
Para ello, dado un sistema de fuerzas, lo primero y ms sencillo es encontrar las componentes
de la fuerza resultante, efectuando simplemente la suma vectorial:

FR F1 F2 ... FN
Luego, estas mismas componentes, cambiadas de signo, constituyen las de la equilibrante:

FE FR

Ahora bien, la ubicacin de FR se define de manera que tenga el brazo de palanca necesario
para que su momento con respecto a cualquier punto sea igual al momento resultante del
sistema con respecto a ese mismo punto. Como dada una determinada ubicacin de una
276

fuerza, su momento no cambia si se la desplaza a lo largo de su misma recta de accin (ya que
esto no cambia el brazo de palanca), entonces lo nico que es posible y necesario determinar

de la posicin de FR , en realidad es la ubicacin de su recta de accin.

Se define la ubicacin de la recta de accin de la resultante de un


sistema de fuerzas de tal manera que:
El momento de la resultante con respecto a cualquier eje es
igual a la suma de todos los momentos de las fuerzas con respecto
al mismo eje.


Consiguientemente, FE , al ser un vector exactamente opuesto al FR , slo puede equilibrar el
momento de sta si tiene el mismo brazo de palanca, es decir la misma recta de accin, que se

define indistintamente para FE y FR .

Fuerzas paralelas
Un caso de mucho inters prctico el de sistemas de fuerzas paralelas. Si, para comenzar la
discusin, consideramos dos fuerzas paralelas del mismo sentido, tendremos una situacin
como la de la figura A9.2.2.

A A' Fig.A9.2.2: Sobre un cuerpo se aplican dos


FA fuerzas paralelas, en los puntos A y B. La fuerza
C' resultante debe equivaler a ellas en mdulo,
direccin, sentido, y momento. Para ello tiene la
FR
misma direccin y sentido, un mdulo igual a la
suma de los mdulos, y se ubica de manera de
tener el mismo momento que las fuerzas
aplicadas. Para calcular los momentos con
FB B B' respecto al punto arbitrario O, se traza la recta
OBCA, perpendicular a las rectas de las fuerzas:
los brazos de palanca son los segmentos OA,
O OB, y OC.

En esta figura A9.2.2 vemos dos fuerzas paralelas del mismo sentido aplicadas en dos puntos
cualesquiera, A y B, de un cuerpo. En funcin de lo que hemos dicho, es lo mismo tratar de
encontrar la resultante de estas dos fuerzas, o considerar que este cuerpo est en equilibrio

gracias a la accin de una fuerza equilibrante FE , la cual, para ser encontrada, requiere de las
mismas ideas y procedimiento.

Comencemos buscando la resultante de este sistema: FR FA FB . Al componer FA y FB ,

vectores paralelos, se encuentra que FR ser un vector tambin paralelo a ellos, con el mismo
sentido, y con un mdulo igual a la suma de los mdulos.
Para ubicar la recta de accin de la resultante debemos pensar que ella debe aplicar el mismo
momento que las fuerzas del sistema respecto de cualquier eje, puesto que se pretende que
equivalga a ellas.
277

De manera que, si elegimos cualquier punto O como eje para tomar momentos, considerando
los brazos de palanca OA, OB, y OC (figura A9.2.2), la recta de accin de la resultante debe
ubicarse de manera que:
FA OA + FB OB = FR OC = (FA + FB) OC
De donde se deduce que:
FA OA FB OB
OC (A9.2.2)
FA FB
Esta expresin permite ubicar la recta de accin de la resultante respecto de cualquier punto O
elegido arbitrariamente.
Si ahora se desea aplicar una fuerza que equilibre al sistema, para ello bastar aplicar la fuerza

FE , opuesta a la FR calculada antes, en cualquier punto de esa misma recta. El sistema

formado por FA , FB , y FE (figura A9.2.3), queda en equilibrio.

A A'
FA FE
C'

FB B B'

O

Fig. A9.2.3: FE equilibra al sistema FA , FB . En estas condiciones, cualquiera de
estas fuerzas es equilibrante del sistema formado por las otras dos.

El punto O con respecto al cual se calculan los momentos es arbitrario. Si se lo elige en un


punto de la recta de alguna de las fuerzas, se anular el brazo de palanca de esa fuerza, y en la
condicin de equilibrio de los momentos slo intervendrn las otras dos fuerzas. Por ejemplo,

si tomamos momentos con respecto al punto B, el momento de FB ser nulo, y la condicin
de equilibrio de los momentos quedar:
FA AB - FE CB = 0 [A9.2.3]
Es decir, para tres fuerzas que estn en equilibrio, siempre se equilibran los momentos de dos
cualesquiera de ellas con respecto al punto en que se aplica la otra.
Esta ltima expresin tambin podra haberse planteado para encontrar la ubicacin de la recta

de accin de FE de una manera tal vez ms sencilla:
FA AB
BC
FE
278

Es interesante advertir que cualquier sistema de tres fuerzas paralelas en equilibrio tendr
siempre el aspecto mostrado en la figura A9.2.3: dos fuerzas apuntan en un determinado
sentido, y la otra, opuesta, que es mayor porque su mdulo iguala a la suma de los mdulos de
stas, queda situada entre ellas.
Para ver que siempre ser as, revisemos los razonamientos siguientes:

a) Si tenemos dos fuerzas paralelas que apuntan en el mismo sentido, como FA y FB , la
equilibrante, de sentido opuesto, con un mdulo FE = FA + FB , debe situarse en un punto con

respecto al cual FA y FB cancelen sus momentos. Para ello este punto debe estar entre ambas,
ya que de lo contrario los momentos tendrn igual signo, y ms cerca de la mayor, para que
sta tenga menos brazo de palanca.

b) Ahora bien, si consideramos dos fuerzas que apuntan en sentidos contrarios, como FA y FE

en la misma figura, entonces la equilibrante es la FB . Para que los momentos de FA y FE

puedan equilibrarse con respecto al punto B en el cual se aplica FB , la recta de accin de sta

ltima debe estar fuera del sistema, de modo que los momentos de FA y FE tengan signo

contrario, y del lado de la mayor, FE , para que sta tenga menos brazo de palanca que la otra.

Adems FB debe apuntar en el sentido de la menor, FA , para que se pueda cumplir la primer
condicin de equilibrio.
De manera que vemos que se repite la conclusin para los sistemas de tres fuerzas paralelas en

equilibrio (y lo mismo se obtendra partiendo del sistema FB y FE en la misma figura):

En un sistema de tres fuerzas paralelas en equilibrio siempre hay dos


fuerzas que tienen el mismo sentido, y la tercera, opuesta, debe tener
un mdulo igual a la suma de los mdulos de las otras dos, y estar
ubicada entre ellas, ms cerca de la mayor.

Esto se comprende bien en cualquier caso prctico, como el de la balanza romana que
presentamos en la figura A9.2.4.

FO

A O B

FB
FA

Fig. A9.2.4: esquema de fuerzas que actan sobre la varilla AOB de una balanza romana.

El funcionamiento de esta balanza se basa sobre la idea de regular el brazo de palanca de la


pesa mvil, hasta que el momento aplicado por su peso con respecto al punto de apoyo O,
279

equilibre al momento del peso del objeto a pesar. Ahora bien, el punto de apoyo O es el punto

en que acta la fuerza FO (reaccin del apoyo), que es la equilibrante de las otras dos (para
simplificar esta discusin estamos ignorando el peso del platillo, de la varilla, etc.; est claro
que todos deben ser tenidos en cuenta en un tratamiento exacto, pero que no se modifican por
eso las ideas que estamos planteando).

Pero si consideramos con respecto al punto A, FO tiende a hacer girar la varilla en sentido
antihorario alrededor de A, considerado fijo, con un momento que F0 OA, que es equilibrado

por el momento -FB AB, con el que FB tiende a hacerla girar en sentido opuesto.

Y algo similar puede decirse con respecto al punto B: FO tiende a hacer girar la varilla en
sentido horario alrededor de B, considerado fijo, con un momento que -F0 OB, que es

equilibrado por el momento FA AB, con el que FA tiende a hacerla girar en sentido
antihorario.
Debe quedar claro entonces que cada fuerza es la equilibrante de las otras dos.

Cupla o par de fuerzas



Si tenemos el caso de dos fuerzas paralelas de sentido opuesto, FA y FB , de mdulos
parecidos (supongamos que FB es levemente mayor que FA), tenemos que la equilibrante debe

tener un mdulo muy pequeo (FE = FB - FA, valor pequeo), estar del lado de la mayor ( FB ),
a una distancia EB suficientemente grande como para que su momento con respecto a B
equilibre al de FA.

FB

A B E

FE
FA

Fig. A9.2.5: Esquema de tres fuerzas paralelas en equilibrio.

Como se ilustra en la figura A9.2.5, si FE muy chico, entonces BE debe ser muy grande, ya
que para que haya equilibrio de momentos con respecto a B debe cumplirse: FA AB = FE BE .

Si el cuerpo sobre el que FA y FB estn aplicadas no es suficientemente extenso como para

abarcar el punto E, no habr forma de equilibrar al sistema con una nica fuerza FE .

Como caso lmite tenemos que si FA y FB tienden a hacerse iguales en mdulo, entonces FE
tiende a cero, mientras que el punto en el que habra que aplicarla se aleja infinitamente. Se
llega as a un sistema muy particular de dos fuerzas de igual intensidad y sentido opuesto,
actuando en rectas paralelas, denominado cupla, o par de fuerzas. Este sistema puede ser
equilibrado por otra cupla (opuesta), pero no por una fuerza equilibrante nica.

La cupla constituye un sistema de fuerzas muy interesante, porque tiene un momento cuyo
valor no depende de la distancia al eje. En efecto, consideremos dos fuerzas paralelas
cualesquiera como las mostradas en el siguiente esquema. Las dos rectas de accin de las
280

fuerzas definen el plano de la rotacin, y el punto O indica la ubicacin arbitraria de un eje


perpendicular a ese plano.
O

A'

OB= brazo de palanca de FB
B'
A FB OA= brazo de palanca de FA
FA
B

Dado que FA = FB = F, resulta que la fuerza resultante (y por lo tanto la equilibrante) es nula:

FA + FB = 0
Pero el momento total con respecto a O vale:
FOB FOA= F (OB OA) = F AB
Vemos que el momento resultante de una cupla depende de la distancia entre las rectas de
accin de las fuerzas, pero no de la ubicacin del eje o centro de momentos.
En funcin de esto definimos que la distancia entre las rectas, AB, es el brazo de palanca de
la cupla, y podemos enunciar:

Momento (resultante) de una cupla


El momento de una cupla, es igual al producto del mdulo de una de las fuerzas
por la distancia entre las rectas, a la cual llamamos brazo de palanca de la cupla.

Los siguientes esquemas ilustran varias posibilidades para el momento mayor o menor de una
cupla, en funcin de la ubicacin relativa de las fuerzas.

A A

A b = AB B A b = AB B b < AB b=0
B B

Momento mximo Momento mximo Momento intermedio, Momento nulo


positivo. negativo positivo

Explicacin de momentos en trminos de cuplas.


Al tener fuerza resultante nula, y tener un momento cuyo valor no depende de la distancia al
eje, la cupla puede ser considerada, en cierta forma, como un momento puro. Es ms, como
veremos a continuacin, siempre que existe una fuerza aplicando un momento con respecto a
un punto, puede interpretarse que hay una cupla responsable de ese momento.

En efecto, consideremos la fuerza FA aplicada en un punto A cualquiera de un cuerpo. El
punto O indica un eje perpendicular al plano del dibujo, con respecto al cual vamos a
281

considerar los momentos. Este eje puede ser una idealizacin matemtica, sin existencia real -

eso no es relevante en esta discusin. El momento de FA con respecto al eje vale FA b,

siendo b la distancia desde la recta de accin de FA hasta el eje.

FA
O
b A


Ahora bien, si suponemos que en O hay aplicadas dos fuerzas, una FO igual a la FA , y una

opuesta, FO , no ha cambiado nada, de manera que tener FA , es lo mismo que tener el

conjunto { FA , FO , y FO }.

FO
FA
O
-FO b A


Ahora bien, FA , y FO forman una cupla cuyo momento con respecto a O es FA b ,

exactamente el mismo que el de la FA sola. De manera que todo el conjunto puede ser

pensado como la fuerza FO (que es la FA pero aplicada en O), ms una cupla de momento
igual a FA b , que en realidad puede estar aplicada en cualquier lugar (ya que su momento es
el mismo con respecto a cualquier punto).
Es decir, este razonamiento nos ensea cmo se puede trasladar paralelamente una fuerza
desde un punto A a otro punto B:
La accin de una fuerza aplicada en un punto A, equivale a la de la misma fuerza
aplicada en B, ms una cupla cuyo momento debe ser el de la fuerza aplicada en
A, con respecto a B.

FA FA
FA
B B
Equivale a -FA
b A
b

Ejemplo desarrollado.
Considere un bolo que se arroja en las condiciones del ejemplo desarrollado al final del Captulo 9,
cuyo enunciado deca:
En un juego de bolos se arroja un bolo de 16 cm de dimetro y 2 kg de masa con una velocidad de 10
m/s. El bolo es arrojado horizontalmente, sin girar sobre s mismo, y rasante con el piso (de manera
282

que toma contacto inmediatamente con l, sin golpear). As que inicialmente el movimiento es de
traslacin pura, y en el punto de contacto, donde el bolo desliza, acta una fuerza de rozamiento de 2
N.
Explique cmo se facilita la explicacin del movimiento aplicando la idea de trasladar las fuerzas
exteriores al centro del bolo, especialmente en la parte del movimiento en que hay rodadura con
deslizamiento parcial respecto del piso.
Desarrollo
Al resolver este problema en aquella ocasin dijimos que, desde que el bolo toma contacto con la pista
se considera la accin de una fuerza de rozamiento constante, de 2 N hacia atrs, que a su vez es la
fuerza resultante, y se calcula cmo es el movimiento del centro del bolo, sin importar dnde est
aplicada dicha fuerza. De esta idea result que este movimiento es un MRUV, con aceleracin a = 2N /
2kg = 1 m/s2, en sentido contrario al movimiento.
Por otra parte, para la rotacin se considera el momento de la fuerza de rozamiento con respecto al
centro, que vale 2 N 0,08 m = 0,16 Nm, y acta en el sentido de hacer rotar el bolo en sentido
horario (rodar hacia delante).
Ahora simplemente agregamos una idea que hace ms claro el panorama: decimos que la fuerza de
rozamiento, actuando sobre el punto inferior del bolo (en el contacto con el piso) puede suponerse
actuando en el centro, a condicin de agregar una cupla de 0,16 Nm, actuando en sentido horario.
De este modo se entiende claramente que el movimiento del centro de masa se puede calcular por la
accin de la fuerza resultante, ya que ella se puede considerar actuando exactamente en ese punto. Y
por otra parte, la rotacin se entiende como resultado de la accin de una cupla neta sobre el sistema,
actuando para acelerar la rotacin hacia delante
Situacin real Equivale a

F
CM v v
Fr
F

Fr Fuerza en el CM para Cupla para estudiar


fuerza resultante estudiar cmo se cmo se acelera la
sobre el bolo frena la traslacin rotacin
283

CAPTULO 10:
Movimientos orbitales

Las Leyes de la Dinmica son necesarias y suficientes para establecer todas las caractersticas
de cualquier movimiento de cualquier cuerpo, y eso incluye todo los casos de rbitas posibles.
No obstante eso, algunos aspectos de movimientos orbitales muy conocidos pueden requerir de
un manejo algebraico bastante difcil para ser establecidos directamente desde las Leyes de la
Dinmica, y por ello en este captulo nos especializaremos en presentar y discutir los aspectos
ms relevantes de estos movimientos, con el mnimo andamiaje matemtico que sea posible.
Tambin aprovecharemos para introducir algunos elementos de la teora cuntica que son
necesarios para extender estas mismas conclusiones al mbito atmico.

10.1.- Movimiento en coordenadas polares


Adems de las coordenadas cartesianas existen otras formas de ubicar la posicin de puntos
en el espacio, como por ejemplo dar su distancia al origen, y los ngulos que ubican el vector
posicin con respecto a direcciones elegidas de referencia. Este tipo de coordenadas suelen
denominarse polares, o esfricas.
Nos limitaremos aqu a movimientos en el plano, porque as bastar con un solo ngulo para
ubicar un punto, y ser suficiente para nuestros fines.

Direccin
angular
v

v vr = v cos

rA vr
A v = v sen

A
Direccin arbitraria
O de referencia: = 0
Fig. 10.1: Elementos de las coordenadas polares en el plano.

Como vemos en la figura 10.1, para cada punto del espacio hay dos direcciones de referencia
sobre las cuales se proyectan los vectores. Una es la direccin radial, que es la direccin en la
284

cual no vara, con sentido positivo hacia donde aumenta r. La otra es la direccin angular,
tangente a la circunferencia en la cual r no vara, con sentido positivo hacia donde aumenta .

Cualquier vector de inters, como v en la figura, se proyecta en esas direcciones para definir
sus componentes vr y v.

Velocidad angular orbital


Consideremos entonces una partcula de masa m movindose en el plano de la hoja.
Perpendicularmente al plano del movimiento se elige arbitrariamente un eje, cuya interseccin
con la hoja es el punto O, origen de las coordenadas polares.
A medida que la partcula se mueve, la ubicamos con su distancia al origen, r, y el ngulo
con la direccin de referencia. Es claro que, como se muestra en la figura 10.2, aunque un
movimiento sea rectilneo, si no est alineado exactamente con el origen el ngulo ir
cambiando, de manera que visto desde O, el movimiento tiene una velocidad angular = /
t.

A
v A v
A A
v
rA

A
A A
Direccin = 0 Detalle ampliado de AA
O

Fig. 10.2: Izquierda. Se ilustra cmo va variando el ngulo que forman las visuales
dirigidas desde O a un punto mvil que viaja en lnea recta. Derecha: se muestra
ampliada la parte en la cual se ve que el desplazamiento AA proyectado sobre la
perpendicular a la visual desde O, es AA = AA sen.

Es claro que en este caso no esperamos que la velocidad angular sea constante o que tenga una
expresin simple. No estamos tratando de simplificar algo, sino de mostrar una forma de tratar
el tema.
Un movimiento rectilneo se complica bastante cuando es descripto en coordenadas polares,
pero el movimiento de traslacin de un planeta en rbita, en cambio, se analiza naturalmente
de esta forma, de manera mucho ms simple que en coordenadas cartesianas. Por ello es que,
cuando se habla del movimiento de traslacin de una partcula descripto con respecto a un
centro, se suele utilizar la denominacin movimiento orbital, an cuando no exista rbita.
As es que denominamos velocidad angular orbital a la que considera cmo cambia (por
unidad de tiempo) el ngulo con que se ubica la partcula vista desde el punto origen o eje
elegido, para distinguirla de la intrnseca, que se refiere al ngulo que giran las partculas del
cuerpo con respecto a su centro de masa.
Designaremos O a esta velocidad angular con respecto al punto O, y como se ilustra en la
parte derecha de la figura 10.2, el ngulo en radianes que barre la visual desde O para un
pequeo desplazamiento AA (para simplificar estamos utilizando AA tanto para designar el
segmento como su longitud) se puede calcular proyectando el segmento AA sobre la

direccin perpendicular a r , obtenindose: = AA/ r = AA sen / r .
285

De manera que, dividiendo por t, tenemos:


v sen v
O = (10.1)
r r
Donde es el ngulo que forma el vector velocidad con la direccin radial.
Vemos que la expresin (10.1) equivale a:
v
O (10.1)
r
Lo cual es natural pues indica que slo la componente v del vector velocidad contribuye al
movimiento angular respecto de O.
Si r es constante esta expresin se reduce naturalmente a la expresin habitual del movimiento
circular = v / r , ya que entonces = 90. En otros casos esta expresin puede ser muy
difcil de utilizar ya que puede variar de forma complicada, aunque eso no nos interesar
para lo que deseamos estudiar. S es importante notar, en funcin precisamente de que no es
constante, que el desplazamiento AA debe ser suficientemente pequeo como para que se
pueda considerar aproximadamente constante en todo el intervalo t. En ese caso, la
perpendicular a la visual OA, es la misma que a la visual OA, y los razonamientos se
entienden bien.
Por ltimo vale aclarar que aunque hemos comenzado mostrando un movimiento rectilneo,
todo lo que hemos dicho se aplica igualmente a movimientos lineales de cualquier forma,
rectilneos o curvilneos. Si se inspecciona la figura 10.2, puede advertirse que una vez que el
mvil ha pasado por A y por A, podra continuar por cualquier trayectoria, como en las
prximas figuras, y todo lo dicho seguira siendo vlido.

Cantidad de movimiento angular orbital


La cantidad de movimiento angular orbital con respecto a O, de una partcula de masa m y

velocidad v , es el momento de la cantidad de movimiento, esto es el producto del mdulo de
la cantidad de movimiento lineal m v, por el brazo de palanca b:

LO = m v b (10.2)

r
b
Fig. 10.3: Elementos para definir la cantidad de
v movimiento angular orbital. Los ngulos y son
m suplementarios, y son equivalentes para calcular LO,
ya que sen = sen.

NOTA:
Esta es la misma definicin que ya hemos utilizado de la cantidad de
movimiento angular, L = mi i2 , aplicada al caso de una nica partcula,
con respecto al punto O tomado como centro:
LO = m r2 O = m r v sen (10.2)
Donde r sen = r sen = b, siendo b la distancia desde el centro O hasta la recta

de accin de v (o de p que es lo mismo).
286

Velocidad Areal
Si se considera un pequeo desplazamiento AA = v t, de la partcula, se encuentra que AA
es la base del tringulo AAO, cuya altura es b, de manera que, para el rea de este tringulo,
tenemos
O

Area = AA b
r
= v b t b
v
A A

Y comparando con la expresin de LO, tenemos:


Area
LO = 2 m
t
= 2 m vareal
Donde vareal = rea / t , es la velocidad areal, es decir el rea barrida por unidad de tiempo
por la lnea desde la partcula al centro.
Vemos que, salvo un factor constante, la cantidad de movimiento angular orbital representa la
velocidad areal del movimiento.

Fuerzas centrales y conservacin de la cantidad de movimiento angular


Las fuerzas atractivas hacia un punto, o repulsivas desde l, por estar alineadas sobre la recta
que pasa por dicho punto, denominado centro de fuerza, O, no pueden aplicar momento con
respecto a l, ya que su brazo de palanca resulta nulo.
En consecuencia, aplicando la Ley del Impulso para Rotaciones, MO t = LO, obtenemos que
no puede variar LO.
Es decir:

Una partcula sometida a una fuerza central, se mueve conservando la


cantidad de movimiento angular orbital con respecto al centro de fuerzas.

Ntese que, eligiendo el centro de fuerzas como origen O del sistema de coordenadas, el

movimiento ocurrir necesariamente en el plano definido por r y v , ya que la fuerza, siempre

alineada con r , trivialmente nunca puede tener componente fuera del plano.

Ley de las reas


Segn lo que hemos dicho, entonces, la conservacin de LO implica que se mantiene
constante el producto v b, y tambin implica que se mantenga constante la velocidad areal.
La conservacin de la cantidad de movimiento angular en trminos de la velocidad areal, es la
Segunda Ley de KEPLER del movimiento planetario, conocida como la Ley de las reas, la
cual dice:
El segmento que une el Sol con un planeta barre reas iguales en tiempos iguales
287

Esta ley haba sido enunciada fenomenolgicamente por Johanes KEPLER (1571-1630), y
luego del desarrollo de la Dinmica se entendi que representaba la conservacin de la
cantidad de movimiento angular, y que era un consecuencia directa de que la fuerza actuante,
la gravedad, fuese central, es decir alineada con el centro.

bA S S
vA bA = vB bB = cte Area (AAS) = Area(BBS)
A FA A

bB A
FB
vB vB
B B B
Fig. 10.4: Se ilustra de dos maneras cmo se interpreta la conservacin de la cantidad de
movimiento angular orbital, en un caso de fuerza central. A la izquierda se muestra, para dos
lugares de la rbita, el brazo de momento de la cantidad de movimiento lineal, y a la derecha se
muestra, para los mismos dos lugares, el rea barrida en un mismo intervalo de tiempo.

10.2.- Movimiento bajo fuerzas coulombianas.


Carlos Agustn DE COULOMB (1631-1716), estudi las fuerzas electrostticas y determin que
su intensidad era inversamente proporcional al cuadrado de la distancia, al igual que en el caso
de la fuerza gravitatoria.
Salvo la diferencia esencial de que la fuerza gravitatoria depende de la masa de los cuerpos
que interactan, y slo puede ser atractiva, mientras que la electrosttica depende de la carga
elctrica, y puede ser atractiva (entre cargas de signo opuesto), tanto como repulsiva (entre
cargas de igual signo), ambas fuerzas son muy importantes en la naturaleza y, por ser ambas
centrales y depender de la distancia de la misma manera, dan lugar a los mismos tipos de
movimiento.
Por esto es que a las fuerzas centrales cuya intensidad disminuye con el cuadrado de la
distancia se las denomina coulombianas, y para ciertas caractersticas de los movimientos que
resultan no es necesario especificar si se habla de planetas o de electrones3.
Un detalle que valdr para todos, satlites, electrones, o lo que sea, porque es universal, es la
conservacin de la energa mecnica, ya que tanto la gravedad como la fuerza electrosttica
son conservativas.
Energa potencial coulombiana
Sabiendo que la energa potencial disminuye hacia dnde apunta la fuerza, es claro que para
una fuerza central la energa potencial slo puede ser funcin de la distancia al centro, r:
Ep = Ep(r)
Si adems recordamos que para un desplazamiento en cualquier direccin debe valer (11.4),
ahora tenemos, para la direccin radial:

3
Para el caso de los electrones en el tomo hay complicaciones que corresponden a la teora cuntica, que invalidan
muchas de las conclusiones que valen para los satlites. Pero an dentro de la teora cuntica es posible seleccionar
resultados de la teora clsica que son vlidos en el dominio atmico, como veremos oportunamente.
288

Ep (r )
Fr =
r
Esta nos dice que la energa potencial debe ser una funcin de Ep(r) tal que su derivada con
respecto a r, cambiada de signo, debe ser Fr, la cual es una funcin conocida simple (la fuerza
coulombiana):
cte
Fr =
r2
Concretamente, para el caso gravitatorio, si tenemos una fuerza atractiva hacia el origen, cuyo
mdulo es F = G M m / r2, donde M es la masa del astro central, muy grande, m es la masa del
cuerpo en rbita cuyo movimiento estudiamos, y G es la constante de gravitacin universal,

dado que el sentido de r es opuesto al de F , tendramos:
K
Fr = (10.3)
r2
Donde K es una constante positiva que vale:
K = G M m = 6,6710-11(N.m2/kg2)Mm (10.4)
Aplicando ahora que la derivada de r1 es r2, se puede inferir que, para este caso, la funcin
Ep(r) debe ser:
1
Ep = K (10.5)
r
Esta funcin admite que se le sume una constante arbitraria. Al no agregarle nada, estamos
eligiendo arbitrariamente que Ep 0 cuando r , lo cual es la costumbre ms difundida.

Podremos extender todo para el caso de un electrn de carga e atrado por un ncleo de Z
protones de carga +e (e 1,610-19C), cambiando solamente el valor de K, que deber ser:
K = kel Z e2 2,31028 (N. m2/C2) Z (10.6)
Donde kel es la constante de fuerza electrosttica que, al igual que G para la gravedad, indica
el valor experimental de la fuerza de atraccin (o repulsin) entre dos cargas de la unidad de
carga situadas a la unidad de distancia, y vale kel 9,0109 Nm2/C2.

Anlisis de casos en fuerza coulombiana atractiva.


Las tres leyes de KEPLER, que fueron enunciadas por ste para el movimiento planetario,
sirven en realidad para el movimiento de cualquier cuerpo en una fuerza coulombiana, cosa
que recin pudo ser demostrada por NEWTON. La segunda de estas leyes es la Ley de las reas
que ya hemos presentado, y excepto ella, las otras dos requieren un trabajo matemtico
demasiado elaborado para estas pginas, por lo cual utilizaremos sin demostrar el contenido
de la Primera Ley, adaptado al lenguaje que nos ser til.
Es decir, utilizaremos el siguiente conocimiento sin demostracin.
Una partcula sometida a una fuerza coulombiana, describe una cnica (elipse, parbola o
hiprbola) con el centro de fuerza en uno de sus focos.
289

A partir de este dato podemos proceder al anlisis utilizando nuestras herramientas habituales,
para el caso de un cuerpo o partcula de pequea masa m, movindose en el campo de una
fuerza central coulombiana.
No trataremos de hacer una descripcin en profundidad desde el momento inicial, sino de
comenzar discutiendo los aspectos de manera ms bien coloquial, para ir ganando en
profundidad al avanzar.
En el anlisis de un ejemplo el primer valor importante es el de la energa mecnica total. Para
comenzar elijamos un valor negativo para la energa mecnica total y representemos las
energas en una grfica en funcin de r. Dejaremos para ms adelante el caso de ET positiva.
E (J)
r b
r
ET
Ec
Ep(r)
K
Ep(r) = r

Fig. 10.5: Una situacin tpica en una fuerza coulombiana,


caracterizada por la energa total. La energa total debe ser negativa
para que la partcula est atrapada en rbita en una regin acotada.

Vemos que en r = b se intersectan la recta indicativa de ET con la grfica de Ep(r). De la


igualdad Ep = ET podemos despejar b = K / ET = K / ET.
Ahora bien, el valor r = b seala toda una zona del espacio, una superficie esfrica de radio b
centrada en el origen, en la cual Ep = ET. Todo el espacio exterior a esta superficie esfrica es
zona prohibida para el mvil con esta energa.
La regin dada por r b es el interior de la esfera, y es la zona en la cual es posible el
movimiento. La energa cintica que tendra la partcula en cualquier lugar dentro de esta
esfera se puede calcular a partir de la conservacin de la energa mecnica:
K
Ec = ET Ep(r) = ET
r
Ahora bien, dentro de esta regin el movimiento es posible de muchas maneras distintas,
siempre teniendo en cuenta que ya sabemos que ocurrir en un plano que pasa por el centro.
Una vez elegido el plano, bastar con r y (ngulo respecto de alguna direccin de
referencia), para describir todo.
Puede haber movimiento variando solamente r (movimiento puramente radial). Puede haber
movimiento circular, con r = cte, mientras solamente vara . Y puede haber infinidad de
combinaciones variando r y de distinta manera.
Para tener un panorama general digamos que, si la partcula estuviese en algn r < b,
alejndose de manera exactamente radial, entonces se alejara hasta r = b, en donde se
detendra. La detencin slo podra ser instantnea, porque inmediatamente la fuerza de
atraccin la hara volver en lnea recta aceleradamente hacia el centro.
290

E (J) v b
r
ET
Fig. 10.6: El punto r = b es
Ec
Ep(r) efectivamente el punto de retorno si la
partcula se aleja radialmente del centro.

Pero en general la partcula no se mueve exactamente a lo largo del radio, y entonces debe
describir una elipse. Cuando llega al punto ms lejano, R = rmx , este valor tiene que ser
menor que b, porque si hubiera llegado hasta r = b, se habra detenido, y a partir de all slo
podra caer radialmente hacia el centro.
De manera que si sigue por la elipse es porque en el punto ms alejado del centro no llega a
estar en reposo, sino que se mueve perpendicularmente a la direccin radial segn la curva
correspondiente, conservando as cierta cantidad de energa cintica mnima, que nunca llega
a ser nula.
Para ilustrar esto en la figura siguiente se muestran tres rbitas posibles para la misma energa.

(1)
(3)

(2) Fig. 10.7: Tres rbitas posibles. La rbita


elptica general es la (3), que en el caso
extremo de mxima excentricidad se
transforma en (1), y con excentricidad
cero es la circunferencia (2).

Revisemos los detalles de lo que es una elipse.


Dados dos puntos F y F, llamados focos, la elipse es la lnea que se forma con todos los
puntos P tales que la suma de las distancias de cada uno a ambos focos es una constante: FP +
FP = cte.
Esto significa que la elipse ser una curva cerrada simtrica con respecto a un eje que
contenga los focos, y tambin con respecto a un eje perpendicular al anterior que equidiste de
ambos focos. La interseccin de estos ejes es el centro O de la elipse, y no es el punto en el
cual se ubica el astro central en el caso de la fuerza coulombiana (segn la Primera Ley de
KEPLER, el astro central se ubica en un foco). En la figura 10.8 se muestran los elementos con
la notacin usual.
P : cualquier punto de la elipse
P F, F: focos
a b
O: centro
F c c
O F OF = OF = c
a a = semi-eje mayor
b = semi-eje menor
e = c/a : excentricidad

Fig. 10.8: Elementos de una elipse


291

Inspeccionando un poco los elementos dados, se advierte que la excentricidad es un nmero


entre 0 y 1.
El caso extremo de e = 0, implica que ambos focos estn confundidos en el origen, y tenemos
una circunferencia en la cual a = b = radio.
El otro caso extremo de e = 1, mxima excentricidad, achata la elipse hasta que se transforma
en un segmento de recta desde F hasta F, con b = 0, y a = c.

centro de la elipse
ba
F F F
F a
astro astro
e casi 1
e << 1

Fig. 10.9: Dos casos casi extremos de excentricidad. Notar que el astro que sera el
centro de fuerza, no est en el centro de la elipse. En el caso de las rbitas de los
planetas (derecha), stas son prcticamente circunferencias descentradas.

Veamos ahora los detalles de cada caso de movimiento.

Caso 1: Movimiento radial.


La partcula se aleja desde r 0, con Ec (luego discutimos si es posible), hasta r = b, en
donde se detiene para volver.
(J) b
r r
ET
Ec en un r cualquiera
Ep(r)
Ec cuando r 0

Fig. 10.10: Energas en el caso de movimiento radial

Claramente en este caso la partcula no tiene cantidad de movimiento angular orbital con
respecto a O (en adelante omitiremos el subndice, ya que siempre ser con respecto a O), ya
que su trayectoria no tiene brazo de palanca:

Movimiento radial: L = 0

Especulemos ahora un poco acerca de lo que sucede en la zona r 0.


Podemos saber qu sucede exactamente en r = 0? Tiene sentido esta pregunta?
No tiene sentido fsico plantear que una partcula es lanzada desde (o llega a) exactamente el
origen con velocidad infinita. Hay que pensar en partir de puntos muy cercanos al origen, con
velocidades muy grandes.
292

Por ejemplo pensemos en un cometa cayendo radialmente (o en una trayectoria prxima a


una recta radial) hacia el Sol. Este cometa vendra desde muy lejos, en un viaje de cientos o
miles de aos, desde un r b muy grande, casi infinito, en donde tena Ec 0, Ep 0, y ET
0.
Antes de llegar a r = 0 (centro del Sol), chocara con su superficie. Inmediatamente antes de
chocar tendra una enorme Ec debido a la enorme disminucin de Ep, que habra pasado de
Ep(b) 0, a Ep(RSol) = valor negativo muy grande (ver figura 10.11 - en valor absoluto es un
aumento, pero como aumenta negativamente, a los fines de la teora es disminucin).
(J)
RSol 0 b
r
ET 0
Fig. 10.11: Un caso prctico de movimiento casi radial.
b podra ser considerado infinito en relacin con el
radio del Sol. La energa cintica del cometa al llegar al
Sol podra ser considerada infinita en relacin con su
Ep Ecf Ep(RSol) energa en otras partes del trayecto, aunque no en
relacin con la energa que almacena el Sol.

Al chocar toda la Ec del cometa desaparece como energa mecnica (decimos que se
transforma en energa trmica - el Sol es muy grande y caliente y no se altera por ello).
Pero tambin podra ocurrir que el movimiento no fuese exactamente radial. El Sol es muy
pequeo, prcticamente un punto comparado con la rbita de cualquier cometa. El cometa
podra dar vuelta por detrs del Sol, muy cerca de la superficie, y visto desde lejos parecera
que vuelve prcticamente por la misma lnea radial por la que se acerc, como si hubiese
rebotado contra la superficie; y el proceso proseguira peridicamente. Se podran inventar
historias de ciencia ficcin con stas y otras posibilidades.

r=b
Sol
Fig. 10.12: La trayectoria radial es un caso lmite de elipse
degenerada en recta, que es una elipse de excentricidad = 1.
Cualquiera que sea la historia que se invente, este cometa sera muy rpido cerca del Sol y
muy lento lejos, es decir que pasara unos pocos meses o das en la vecindad del Sol (que es
cuando podemos avistarlo) y siglos en la parte lejana.

Caso 2: rbita circular.


La circunferencia es un caso particular de elipse sin excentricidad. En este caso el movimiento
es circular uniforme, y todo se mantiene constante en l, no slo ET.
Veamos relaciones que sern muy tiles. Si distinguimos con subndice 0 a todos los
valores que correspondan a este caso especial (r0, v0, Ec0, Ep0, etc.) tendremos:
K m v02 K
Fuerza normal: 2 simplificando r0 m v02 (10.7)
r0 r0 r0

Conservacin energa: Ep0 + Ec0 = ET Ec0 Ep0 = ET (10.8)


293

En (10.7) vemos que m v02, que es el doble de Ec0, es exactamente igual a la energa potencial
cambiada de signo, o sea, ya que sta es negativa, igual al valor absoluto de la misma: Ec0 =
Ep0.
Introduciendo esto en (10.8), queda algo muy simple e interesante:
K
Ec0 =ET = Ep0 ET =
2 r0
Pero siendo que b = K /ET, entonces: r0 = b

E
r0 b r
ET
Ec0 Fig. 10.13: El caso del movimiento circular
Ep0
uniforme con energa total ET. La partcula se
mantiene en r0 = b/2, con Ec0 igual al valor
absoluto de ET.

La cantidad de movimiento angular orbital para este caso vale: L0 = m v0 r0 , que como
veremos enseguida, es el mximo valor que puede tener el momento angular orbital para el
valor dado ET de energa total.

Caso 3: rbita elptica cualquiera.


Apoyndonos ahora en lo que sabemos del movimiento circular, pensemos en la siguiente
situacin.
Dado siempre el mismo valor de energa total negativo, ET, nos ubicamos en A, a la distancia
r0 necesaria para establecer una rbita circular (ya sabemos que es r0 = K/ET ), y desde
all nos proponemos lanzar la partcula con la velocidad v0 necesaria para el movimiento
circular (ya sabemos que vale v0 = 2 E T m , porque la energa cintica tiene que ser igual
a ET), pero en varias direcciones diferentes, como se ilustra en la figura 10.14.

(2)

v0
v2 Fig. 10.14: El cuerpo se lanza desde A (a distancia r0 del
v3
v1 origen) en varias direcciones distintas, con velocidad
(1)
A siempre de mdulo v0 (v1 = v2 = v3 = v0). En la direccin

radial, v1 , tendremos el movimiento rectilneo 1. En la

(3) direccin perpendicular a la radial, v 0 , tendremos un
r = r0 movimiento circular. En cualquier otra direccin tendremos
elipses de caractersticas bsicas similares, como (2) y (3).
r=b

De la forma que hemos procedido queda claro que se obtienen rbitas diferentes, pero todas
de la misma energa total ET, ya que sta se conserva, y en el punto inicial, A, todas comparten
el mismo valor de Ep y de Ec.
294

Sin embargo todas tienen diferente momento angular orbital, ya que ste tambin se conserva
a lo largo de cada rbita, pero en el momento inicial todos son distintos. Efectivamente, si
aplicamos (10.2): L = m r v sen, tendremos que:
El movimiento (1), radial, se inicia con = 0, y por tanto L1 = 0, como ya hemos dicho.
El movimiento circular se inicia con = 90 , y por tanto tiene L0 = m v0 r0, que es el mximo
valor posible de L para esta energa, porque sen90 = 1 = mximo valor posible del sen.
Cualquiera de los otros movimientos, con trayectorias elpticas, se inicia con sen < 1, por lo
cual L2 = m v0 r0 sen2 < L0, y L3 = m v0 r0 sen3 < L0.
Teniendo en cuenta esto, ahora inspeccionemos una rbita elptica cualquiera, como la de la
figura 10.15.
zona en la cual
puede hallarse
r=b (J) la partcula

rp rp ra b
r
ra va ET
Ecmx Ecmn

vp

Fig. 10.15: Aqu se muestran los elementos bsicos de una rbita elptica tpica. En el diagrama
de la derecha se entiende claramente que la interseccin de Ep(r) con el valor de E T ya no indica
un punto de retorno, como ocurrira en un movimiento unidimensional. La apariencia de este
diagrama es engaosa porque slo muestra una de las coordenadas del movimiento.

Vemos que la partcula no llega ni a r = 0, ni a r = b. Se mantiene orbitando entre los valores


rmx = ra y rmn = rp. Su energa cintica no alcanza a anularse nunca, sino que disminuye hasta
un valor mnimo en el punto ms alejado, y crece hasta un valor mximo en el punto ms
cercano al origen. En astronoma se designa apoastro al punto ms alejado del astro central
(afelio si es el Sol, y apogeo si es la Tierra), y periastro al ms cercano (perihelio si es el Sol,
y perigeo si es la Tierra).
Ahora bien, planteamos:
Conservacin energa mecnica:
K m v2
ET (10.9)
r 2
Conservacin momento angular:
L = m vp rp = m va ra va = vp (rp / ra) (10.10)
Si escribimos (10.9) para el punto ms cercano y el ms alejado, y recordamos que ET =
K/r0, queda:
2
K m vp K
(10.9a)
rp 2 2 r0
295

K m va2 K
(10.9b)
ra 2 2 r0
Ahora, si utilizamos (10.8), podemos eliminar va en (10.9b), luego, con (10.9a) eliminamos
tambin vp , obteniendo (luego de despejar tarea para el lector):

rp + ra = 2 r0 (10.11)

Pero vemos en la figura 10.15 (o en la 10.8), que rp + ra es la longitud el eje mayor de la


elipse, y segn (10.11), debe valer lo mismo en todas estas rbitas posibles.
Esto nos dice claramente que:

Todas las rbitas de la misma ET,


tienen el eje mayor de la misma longitud.

Notar que en el caso particular e = 0 la rbita es una circunferencia cuyo dimetro, que mide 2
r0, tambin es el eje mayor, y que en el caso e = 1, se tambin es el valor de b, que es el eje
mayor.
Adems (10.11) tambin nos dice que r0 = (rp + ra) = promedio entre distancia mxima y
mnima.
O sea, mirando la figura 10.14, ahora podramos decir que cuando lanzamos el cuerpo desde
A (con la velocidad v0) oblicuamente hacia fuera (o hacia dentro) con relacin a la
circunferencia de radio r0, se aleja tanto hacia fuera de la circunferencia, como despus se va a
alejar hacia dentro lo que le falte para llegar a la distancia mxima, r = b, es exactamente lo
que le va a faltar luego para llegar al centro.
Las rbitas de poco momento angular tienen gran variacin de distancia alrededor de r0, que
siempre es la distancia promedio, mientras que las de gran momento angular, se mantienen en
un anillo angosto cerca de la circunferencia de radio r0.

LAS LEYES DE LA DINMICA


Es importante recordar que siempre deberamos ser capaces de aplicar las Leyes
de la Dinmica a cualquier situacin, aunque no podamos desarrollar
detalles matemticos.
As por ejemplo, en un punto cualquiera de una rbita elptica debemos ser
capaces de dibujar la fuerza actuante, que es un vector hacia el centro de
fuerza, encontrar sus componentes normal y tangencial, por ejemplo, y
analizar el efecto de cada una.
En cada punto de la curva tendremos una fuerza tangencial FT que determina
que la velocidad deba aumentar en la parte en que la partcula se acerca al
centro de fuerza, y disminuir cuando se aleja.
Por otra parte, la fuerza normal, FN, acta curvando la trayectoria, la cual,
segn la Ley del Impulso para la fuerza normal debera tener un radio R =
m v2/FN.
Ahora bien, no estamos en un movimiento circular, pero eso no importa: en
cada instante, el arco que se recorre durante un pequeo intervalo t, se
confunde exactamente con una circunferencia del radio R debido. Este radio
296

se denomina radio instantneo de la curva, aunque no sea una


circunferencia.
eje N
eje N recta tangente
eje T A
FT A
v RA
FN RA RA
F
mejor
circunferencia
tangente en A
CA
CA
Como se ilustra en la figura para un punto cualquiera A (en una elipse
cualquiera), a cierta distancia sobre el eje normal habr un punto, C A , tal que
haciendo centro en l con un comps, se podra trazar la mejor circunferencia
tangente a la trayectoria en A (hay que encontrar el punto C A correcto para
ello puede caer dentro o fuera de la elipse- y aqu no profundizaremos ms).
En la vecindad de A, la trayectoria se confunde perfectamente con un pequeo
arco de esta circunferencia, y, mientras se lo est recorriendo, se est en un
movimiento circular de radio RA. Debe cumplirse: FN = m v2/RA.
CA y RA se denominan respectivamente centro instantneo y radio instantneo
de la curva en la vecindad de A. No pretendemos saber encontrarlos, pero s
saber que existen.

El caso de los electrones en el tomo


En el dominio atmico rige la Mecnica Cuntica, que tiene una estructura matemtica que no
trataremos de abarcar. No obstante podemos aqu establecer un pequeo resumen de muchos
de estos resultados clsicos que s podemos aplicar al tomo.
La teora cuntica nos dice que no debemos cometer el error de atribuir todas las propiedades
de una partcula clsica a un electrn. Por ejemplo no debemos pensar que existe una lnea
que sea la trayectoria de un electrn.
Pero s podemos pensar en estados orbitales con cierta distribucin media en el espacio, y
caracterizados por ciertos nmeros cunticos, que, como veremos, definen los valores
posibles de algunas variables, que son las nicas susceptibles de arrojar valores definidos en
una medicin. stas son en general todas aquellas que en la fsica clsica son constantes de
movimiento (energa, momento angular, etc.).
A saber, para el movimiento orbital de los electrones en el tomo tenemos cuantificadas:
ENERGA MECNICA TOTAL, En , cuantificada con el nmero cuntico principal, n,
MOMENTO ANGULAR ORBITAL de mdulo cuantificado con el nmero cuntico l,
componente segn un eje del VECTOR AXIAL correspondiente al MOMENTO
ANGULAR ORBITAL, cuantificada con el nmero cuntico m.
Cada una de estas variables slo puede adoptar los valores que corresponden a alguno de los
nmeros cunticos. Los valores intermedios no son permitidos. El cambio de un estado a otro
se entiende como un proceso brusco en el cual no se pasa de manera continua por los estados
intermedios los cuales no existen.
297

Veamos detalles.
Niveles de energa
El electrn puede estar en estados de energa total constante, pero que no pueden tener
cualquier valor, sino slo valores discretos dados por el nmero natural n, denominado
nmero cuntico principal.
Es decir, slo son permitidos los valores:
1
E1, E2, E3, En, dados por: En = E1 (10.12)
n2
Donde E1 es un valor negativo (as, todos los En son negativos, como corresponde) dado por
cierta expresin que se obtiene de la teora:
Z2 2 2 k el
2
me 4
E1 = Z2 2,18 1018 J
h2
Donde:
e 1,6 1019 C : carga elemental (del protn y del electrn, en valor absoluto)
kel 9,0 109 Nm2C2 : constante de fuerza electrosttica
m 9,1 1031 kg : masa del electrn
h 6,6 1034 J.s : constante de PLANCK
Z: nmero atmico (nmero de protones en el ncleo)

En la figura 10.16 se muestran los 3 primeros niveles para el caso ms simple de Z = 1, que es
el tomo de hidrgeno (recordando que Ep(r) = K/r, con K dado por (10.6): K 2,31028
(N. m2/C2) Z).
(1018J) b1 b2 b3
1 2 3 4 5 6 7 8 9 10 r() Fig. 10.16: Primeros niveles de energa en
E3 el tomo de H. Con la letra b indicamos el
E2 radio de la zona ms all de la cual sera
-1
imposible, clsicamente, hallar la partcula
E1 -2 de esa energa. Cunticamente decimos que
la probabilidad de que el electrn sea
-3 Ep(r) =
K
hallado ms all de b es tan baja, que
r
-4 prcticamente equivale a la imposibilidad.

La figura permite ver inmediatamente que, al ser valores negativos, mientras el valor absoluto
de En disminuye con n, En aumenta: si n > n, entonces En > En. Para n tendramos que
ET 0, el electrn se podra alejar infinitamente, y ya no se considerara ligado al tomo.
As E1 es el nivel ms bajo posible, denominado nivel fundamental. No es posible, segn la
teora cuntica, un estado de menor energa para un electrn en el tomo. Eso impide que los
tomos colapsen irradiando energa mientras los electrones se precipitan hacia r = 0.
Dado que bn , el radio de la zona accesible para un electrn con energa En , es inversamente
proporcional a En , resulta de estas frmulas una separacin muy marcada entre las zonas que
pueden ocupar los estados orbitales de los electrones con distintas energas, como se ve para
los primeros niveles en la figura 10.17. En la prctica se dice que los electrones se distribuyen
en capas de distinta energa:
298

Un electrn con energa E1 (nivel fundamental), se encuentra dentro de una regin de


radio b1, muy prxima al ncleo. Estos electrones son los ms ligados, porque para
arrancarlos del tomo hara falta mayor cantidad de energa que en cualquier otro nivel
(Wext E1). Se considera que el electrn ha sido arrancado cuando ha sido llevado a
un nivel con ET 0.
Los siguientes, con energa E2 (podran ser arrancados del tomo con Wext E2,
cuatro veces menor que la energa necesaria para arrancar un electrn del nivel 1),
pueden llegar hasta el radio b2 = 4 b1. Aunque algunos de estos electrones tambin
pueden acercarse al ncleo, la mayora de ellos tienen demasiado momento angular y
nunca entran (o, en el lenguaje cuntico, tienen bajsimas probabilidades de hacerlo) a la
zona de los electrones de la capa inferior.
As sucesivamente, los de la capa n3 estn 9 veces menos ligados y ms lejos que los del
nivel fundamental, etc.

Momento angular orbital


El mdulo de la cantidad de movimiento angular orbital est dado, para el nivel n, por el
nmero natural l, que va desde 0 hasta n1, segn la expresin:
h
L= l(l 1) (10.13)
2
Donde h es la constante de PLANCK (en general se utiliza el smbolo h 2 ), que tiene
unidades de momento angular, y se considera la unidad atmica de momento angular.
As vemos que:
En el nivel fundamental, n = 1, l slo puede valer 0, Y lo mismo L. O sea que slo hay
movimiento radial. Como en la teora cuntica se suele interpretar que el electrn es una
onda, stas seran ondas esfricas que oscilan entrando y saliendo radialmente. Se
denominan s, ondas s, o estados orbitales s.
En el nivel n = 2, l puede valer 0, y tambin 1. O sea que en este nivel hay estado s, sin
momento angular, pero tambin estados denominados p, con l = 1, o sea L1 = 2 .
En el nivel 3, tenemos estados s, estados p, y tambin estados d, con l = 2, o sea con
momento angular L2 = 6
Etc. En cada nivel habr estados con momento angular que va desde 0 hasta un valor
mximo dado por (10.13) con el valor l = n1, que es el mximo l para el nivel.

Repitiendo las caractersticas del caso clsico, se encuentra que los estados orbitales de poco
momento angular combinan ondas radiales (entrante-saliente), con ondas que circulan
alrededor del ncleo.
Los estados de mucho momento angular se distribuyen en una capa ms delgada alrededor del
radio medio de la zona (bn/2). En los estados de bajo momento angular los electrones tienen
probabilidad de ser encontrados en una capa ms gruesa, pudiendo alejarse ms del ncleo, y
tambin acercarse ms al mismo.
299

Los electrones en estado s, caso extremo, son los nicos que tienen cierta probabilidad de ser
hallados en el ncleo o muy cerca de l, y a la vez que son los que tienen probabilidad de ser
hallados ms lejos, que es la mayor probabilidad (por razones similares a los cometas, que
estn la mayor parte del tiempo lejos del Sol).
Tan cierto es que estos electrones llegan al ncleo, que algunos ncleos inestables pueden
ocasionalmente CAPTURAR un electrn de un estado s. Este proceso se denomina captura
, y slo ocurre con determinados ncleos inestables. La masa del ncleo no se altera mucho
por esto, pero su carga elctrica s: disminuye en una carga elemental (un protn se transforma
en un neutrn), y el elemento se TRANSMUTA en un istopo del elemento anterior de la
Tabla Peridica con Z Z1, y con el mismo nmero de masa.

Componente espacial del momento angular orbital



La componente del vector axial L respecto de alguna direccin del espacio (en general se
elige el eje z, y se lo denomina eje de cuantificacin), est cuantificada segn:
Lz = m (10.14)
Donde m es un nmero entero que va desde l hasta l.
Es decir, para los estados s, m slo puede valer cero. Hay un solo estado s, que es de simetra
esfrica y no puede tener diferentes orientaciones en el espacio.
Para los estados p, que tienen L1 = 2 , hay tres orientaciones posibles con L1z = + , 0 , y

. Como el vector L es perpendicular al plano del movimiento clsico, esto en la teora
clsica indicara tres orientaciones posibles de este plano, y en la teora cuntica,
similarmente, tres estados orbitales con igual estructura radial y las correspondientes
orientaciones diferentes de la rotacin.
Y as sucesivamente, los estados d son 5 estados con L2 = 6 , y con rotaciones dadas por
L2z = 2 , , 0 , , y 2 .

El lmite clsico
La constante de PLANCK es prcticamente el elemento clave de la Teora Cuntica (Max Karl
Ernst Ludwig PLANCK, 1858-1947, la introdujo en 1900). La pequeez de esta constante es lo
que hace que los fenmenos cunticos pasen desapercibidos en la vida prctica, pero sean
notables a nivel atmico.
Si considersemos cada vez ms pequea esta constante, el espaciamiento entre los valores
permitidos de cualquier variable se hara cada vez ms pequeo, y sta podra pasar de un
valor permitido a otro infinitamente prximo de manera casi continua habramos
recuperado as la fsica clsica. Ahora bien, si revisamos las frmulas, encontramos que para
tener valores tpicos de cualquier variable, con h muy pequeo, tendiendo a cero, deberamos
tomar nmeros cunticos muy grandes, tendiendo a infinito.
Y ste es el llamado lmite clsico: con h que tiende a cero, esperamos que para los grandes
valores de los nmeros cunticos, las frmulas cunticas tiendan a las clsicas.
As por ejemplo, por razones de la indeterminacin que debe reinar en el dominio de la
cuntica, el vector axial momento angular, de mdulo Ll = l(l 1) , no puede proyectar todo
300

su mdulo sobre un eje (porque eso lo ubicara exactamente en el eje, perdindose el grado de
indeterminacin debido). Por eso es que su mxima proyeccin, l , siempre es menor
que l(l 1) . Sin embargo, si h se pensara tendiendo a cero, con grandes valores de l
podramos compensar esta pequeez para obtener cualquier valor de L de algn problema
dado, y, siendo que para l muy grande l(l 1) l, el comportamiento del vector L sera
clsico: podra proyectarse en todo su mdulo sobre cualquier eje, es decir alinearse
exactamente con l.
Otro ejemplo es el caso del mximo momento angular posible del nivel de energa n.
Cunticamente tenemos que con lmx = n1, ser Lmx = (n 1)n , valor que para n muy
grande tender a n .

Despejando de la frmula de la energa obtenemos e2 k el m 2 E1 = K m 2 E1 , con


lo cual podemos escribir el momento angular mximo en funcin de la energa total En , para
grandes valores de n, que resulta:
m m
Lmx n K =K
2 E1 2 En

Ahora bien, esto es exactamente lo mismo que encontramos en el tratamiento clsico si


escribimos Lmx = m v0 r0, en trminos de la energa total del nivel (usando v0 = 2 E T m , y
r0 = b/2 = K/(2ET)).

Energas positivas y escape de la partcula


Hasta ahora hemos analizado casos con ET < 0. Veamos qu sucede si la energa total es
positiva. En la figura siguiente podemos notar que en esta caso la partcula puede alejarse
hasta el infinito: NO EST LIGADA al centro de fuerzas. La trayectoria es una curva que
segn la Primera Ley de KEPLER es una parbola (slo si ET = 0), o una hiprbola, siempre
con el centro de fuerza en un foco.

v m
E Ec(r)
ET
r F

astro central

Fig. 10.17: Un caso de trayectoria hiperblica. A la izquierda el esquema de energas


correspondiente. A grandes distancias la hiprbola se transforma prcticamente en una lnea
recta, como lo muestran las rectas dibujadas, denominadas asntotas de la hiprbola.
A medida que el cuerpo se aleja del centro de fuerza va disminuyendo su energa cintica,
pero la fuerza atractiva que le va quitando esa energa tambin va disminuyendo. El resultado
es que muy lejos ya la Ep(r) se mantiene casi constante, y eso indica que la fuerza casi es nula
301

y puede ser ignorada, pero a la partcula le queda an una cierta cantidad de energa cintica.
En la grfica se ilustra que Ec() = ET.
Podemos terminar estos ejemplos diciendo que la condicin lmite, que separa los casos de
partcula LIGADA al centro de fuerzas, del caso de partcula NO LIGADA, es ET = 0.
Con ET = 0 la partcula puede alejarse hasta el infinito, pero pierde toda su energa cintica en
el proceso (Ec() = 0).
Para estos casos tambin es posible hacer diversas consideraciones sobre la relacin entre la
forma de la trayectoria y el momento angular orbital, pero no son de mucho inters por lo cual
no profundizaremos en el tema.

Velocidad de escape.
Supongamos que tenemos un cuerpo en la superficie de la Tierra (o de cualquier planeta o
astro similar), y nos preguntamos si es posible lanzarlo hacia arriba con una velocidad tal que
no retorne nunca (ignorando la resistencia del aire, as como la rotacin del planeta).
Es claro que no es vlida la aproximacin habitual de considerar el peso constante. Hay que
considerarlo como fuerza coulombiana que se va debilitando a medida que el cuerpo se aleja.
Si pensamos que en la superficie del planeta de radio R, el cuerpo est inicialmente en reposo
a distancia R del centro de fuerza, deberemos atribuirle energa mecnica total inicial E0 igual
a la potencial, la cual a su vez tiene el valor dado por la expresin coulombiana Ep(R) =
K/R.
E
interior R exterior r

Wext K
Ep(r) =
r
K
E0 E0 =
R
Esta funcin ya no es vlida
en el interior de la Tierra
Fig. 10.18: Esquema de la energa potencial
coulombiana cerca de la superficie terrestre.

Vemos en la figura que un agente externo debe realizar el trabajo capaz de dar al proyectil la
energa cintica que lleve la energa mecnica total desde su valor inicial E0 hasta su valor
final 0, con lo cual se podr alejar infinitamente.
Esto es la diferencia 0 E0 = E0= G M m / R, de manera que la energa cintica buscada es
v0 2 G M m
2
m R , y la velocidad necesaria resulta independiente de la masa del proyectil y
est dada por: v0 2GM R .
Esta velocidad se conoce con el nombre de velocidad de escape, y para el planeta Tierra
vale aproximadamente 11,6 km/s.
302

NOTA:
Es importante poder relacionar la energa potencial gravitatoria dada por
Ep = m g y, que hemos utilizado para movimientos en la superficie de la Tierra
(ahora la vamos a llamar Ep*), con sta, de forma coulombiana, dada por
Ep(r) = K/r.
Para hacerlo comencemos notando que y, la altura con respecto a algn nivel
arbitrario, tambin es una coordenada radial: y = r R, de manera que:
Ep* = m g y = m g r m g R
Pero si recordamos que g = G M / R2, entonces:
r 1 r 1
Ep* = m G M 2
mGM =K 2 K
R R R R
sta es una funcin lineal de r, con pendiente K/R2, que vale cero en r = R (ver
Ep*(r) en la figura). Notar que K/R2 = m gsup , es el peso del cuerpo en la
superficie terrestre.
Ahora bien, si calculamos la derivada de Ep(r), nos da K/r2, lo que nos muestra
que en r = R, Ep(r) vale K/R, y tiene la misma pendiente K/R2 que Ep*.
Esto nos dice que si a Ep(r) le sumamos K/R, se va a confundir (va a ser
tangente) con Ep*(r) en la cercana de r = R; es la grfica en lnea de trazos en
la figura.

E Ep*(r)
K
R K
Ep(r) +
R
K
R R r
K
Ep(r) =
r
K

R

De manera que si quisiramos resolver el problema del escape con la funcin


Ep*(r) no podramos, porque tiene pendiente constante, o sea que indica fuerza
peso constante a cualquier distancia, lo cual no es real. Con ella, para
cualquier energa total (cualquier velocidad inicial), siempre encontraramos
un r de retorno, que nos dara el mximo alejamiento posible.
Pero si consideramos que la pendiente de la grfica tiene que ir disminuyendo
con la distancia, entonces la grfica se transforma en la lnea de trazos, y
vemos la respuesta correcta: ya no hay punto de retorno cuando la energa
total llega al valor K/R.
Dado que a las funciones Ep siempre se les puede sumar cualquier constante sin
que se alteren las conclusiones fsicas, la funcin en lnea de trazos es, en su
significado fsico, totalmente equivalente a la coulombiana Ep = K/r, como
puede verse a partir de los siguientes razonamientos, adaptados a cada una de
las funciones:
Segn la funcin en lnea de trazos el cuerpo tiene Ep = 0, en la superficie,
y hay que darle Ec K/R para que escape. Va a llegar infinitamente lejos,
donde va a tener K/R de potencial, y lo que le sobre de cintica.
303

Segn la funcin Ep = K/R, el cuerpo tiene Ep =K/R, en la superficie, y hay


que darle Ec K/R para que escape. Va a llegar Infinitamente lejos, donde
va a tener 0 de potencial, y lo que le sobre de cintica.

Escape del tomo: energa de ionizacin.


Si tenemos un electrn ligado a un ncleo en un estado de energa total E0 (que debe ser un
valor negativo), es elemental calcular qu energa es necesario suministrarle para arrancar este
electrn del tomo.
Tenemos que pasar de un estado de energa total ETinicial = E0 < 0, a un estado de ETfinal 0 (es
suficiente con ETfinal = 0).
r
ET
E Fig. 10.19: Salto de energa
necesario para la ionizacin.

Un agente exterior debe suministrar una cantidad Wext = ET = 0 E0 =E0 (recordar que E0
es un valor negativo).
Esta cantidad de energa se denomina energa de ionizacin, pues el tomo luego quedar
definitivamente separado de este electrn, transformado en un ion positivo.
En general en el proceso de ionizar un tomo de muchos electrones, se denomina energa de
ionizacin a la mnima energa necesaria para ello, o sea a la necesaria para quitar el electrn
menos ligado. De manera que se considera a todos los electrones ubicados en los orbitales de
ms baja energa que les sea posible ocupar. Esto se denomina estado fundamental del tomo.
El electrn menos ligado ser el que est en el nivel ms alto de todos, siempre en el estado
fundamental del tomo, y la energa de este nivel, en valor absoluto, es la energa de
ionizacin.

Fuerza coulombiana repulsiva


Este es un caso de poco inters para nosotros, por lo cual le vamos a dedicar poco espacio.
Pero conceptualmente interesa saber que existe. Ocurre, por ejemplo entre cargas del mismo
signo, y corresponde a las famosas experiencias con las cuales Ernest RUTHERFORD (1871-
1937) en 1911, explor el tomo bombardendolo con partculas (de carga positiva) y
encontr que stas rebotaban como si hubiesen chocado contra algo muy masivo y
extremadamente pequeo (entindase: mucho ms pequeo que el tomo).
RUTHERFORD interpret estos resultados diciendo que el tomo tena un ncleo
extremadamente pequeo (prcticamente un punto comparado con el resto del tomo), en el
que estaba la carga positiva, y prcticamente toda la masa. Aplicando estas mismas leyes que
estamos viendo l encontr que las partculas no llegaban en realidad a chocarlo
mecnicamente, sino que se desviaban por la fuerte repulsin electrosttica, siguiendo las
correspondientes trayectorias hiperblicas (figura 10.20).
Y AS SE DESCUBRI QUE EL TOMO TENA NCLEO. Y EL MUNDO NO VOLVI
A SER EL MISMO.
304

Y aqu lo que interesa saber es que se siempre se pueden aplicar las mismas leyes bsicas:
La trayectoria debe ser una hiprbola con el centro de fuerza (repulsiva en este caso) en
un foco, porque sigue valiendo la Ley de KEPLER que hemos visto,
Se debe conservar el momento angular orbital (Ley de las reas), porque la fuerza es
central, y la energa total, porque es conservativa.
La energa potencial es la misma funcin que en el caso atractivo, pero positiva, de
manera que la fuerza apunta hacia fuera, que es hacia donde disminuye Ep(r) = +K/r.

E v F
Ep(r) = K / r
eje
Ncleo
ET
asntotas
r

Fig. 10.20: Trayectoria hiperblica de una partcula que se acerca a un ncleo atmico y
es repelida por l. A la izquierda el esquema de energas correspondiente, el cual muestra
que hay un acercamiento mximo posible para una partcula con energa dada.

10.3.- El caso de la fuerza elstica.


En este caso tenemos una fuerza central atractiva de mdulo proporcional a la distancia:
Fr = k r
La energa potencial correspondiente es (a menos de una constante cualquiera C que se le
puede sumar sin que afecte las consideraciones fsicas):
Ep = k r2
Por ser una fuerza central ya sabemos que el movimiento debe ocurrir en un plano, y si en ese
plano consideramos los ejes cartesianos x, y, podemos llegar a varias conclusiones
importantes.
Por un lado vemos claramente que las componentes cartesianas de la fuerza pueden
expresarse, cada una, como una fuerza elstica en el eje correspondiente, ambas de la misma

constante k, ya que F k r equivale a decir (Fx ; Fy) = k (x ; y), y esto es:
Fx = k x ; Fy = k y
De manera que podemos tener oscilaciones armnicas independientes en cada eje, de
cualquier amplitud cada una, pero necesariamente de la misma frecuencia:
1 k
f= =
2 2 m
La superposicin de estas oscilaciones ser una curva cerrada, ya que cada vez que transcurra
un perodo T la partcula volver al mismo valor de x, y al mismo valor de y, es decir al

mismo lugar r = (x;y).
Ahora bien, comencemos suponiendo el caso en el que las oscilaciones en x, de amplitud A,
estn adelantadas T/4 respecto de las oscilaciones en y, cuya amplitud es B.
x(t) = A cos(t) ; y(t) = B sen(t) (10.15)
305

Si utilizamos la relacin trigonomtrica cos 2 (t ) sen 2 (t ) 1 , escribiendo cos(t) = x/A, y


sen(t) = y/B, obtenemos:
x 2 y2
1 (10.16)
A 2 B2
Pero esta es la ecuacin cartesiana de una elipse de semi-ejes A y B (figura 10.21).
Ahora bien, las oscilaciones en cada eje son independientes y sabemos que cada una cumple
con:
k x2 + m vx2 = k A2 = energa total de la oscilacin en x.
k y2 + m vy2 = k B2 = energa total de la oscilacin en y.
Sumando estas expresiones, y teniendo en cuenta que r2 = x2 + y2, y que v2 = vx2 + vy2,
obtenemos:
k r2 + m v2 = k (A2 + B2) = ET , del movimiento elptico. (10.17)

Denominamos b = A2 B2 , a la distancia a la cual Ep = ET, o sea ET = k b2 . sta es la


distancia ms all de la cual es imposible encontrar la partcula con la energa total dada, y
como se deduce de (10.17) es la diagonal de un rectngulo cuyos lados son los semi-ejes
mayores de la elipse.
y E (J)
k r2
ET Ep =
Ecmn 2
B A x Ecmx
r
B A b
zona en la cual puede
r=b hallarse la partcula
Fig. 10.21: Elementos de las oscilaciones elsticas.

Ahora bien el caso ms general posible es el caso en que las oscilaciones en cada eje estn
defasadas en cualquier fraccin de perodo, y no slo en T/4. No abordaremos ese caso porque
requiere un trabajo algebraico ms arduo, pero anunciaremos, sin demostrarlo, que nada vara
en las conclusiones generales.
El movimiento ms general posible es una elipse centrada en el origen, cuya excentricidad
(que aqu indica achatamiento de la elipse, pero no ubicacin fuera de centro) puede variar
desde 0 hasta 1.
Dada una energa total ET, determinamos b = 2 ET k , o sea el valor que cumple con ET =
k b2, y luego podemos decir que son posibles todas las elipses cuyos semi-ejes forman un
rectngulo cuya diagonal es b.
Los casos extremos son:
e = 0: movimiento circular de mximo momento angular orbital. Debe ser r0 = b / 2 , y
se verifica fcilmente que se cumple con la ley de fuerza normal: m v02/ r0 = k r0 , pues
ella equivale a: v0 = r0. Esto tambin implica que m v02 = k r02, lo cual dividido por 2
dice que: Ec0 = Ep0 = ET.
306

e = 1: movimiento rectilneo oscilatorio a lo largo de todo el dimetro de la zona de r


b. No tiene momento angular orbital.

En la prctica la situacin de fuerza elstica en el dominio atmico se encuentra siempre que


hay alguna partcula que es mantenida en una posicin de equilibrio por fuerzas que le aplican
los vecinos (en general en el centro de fuerzas no puede haber un agente responsable de una
fuerza como sta, porque all debe haber lugar para la partcula mvil, ya que es su posicin
de equilibrio).
Por ejemplo un ncleo atmico es mantenido en el correspondiente sitio de la red o de la
molcula, por la nube electrnica entrelazada con las de los tomos vecinos.

Propuesta experimental casera


Un ejemplo aproximado de estas trayectorias se obtiene caseramente con un pndulo de hilo muy
largo, haciendo que ejecute oscilaciones de pequea amplitud, para que el movimiento se mantenga
aproximadamente en un plano horizontal. Siendo la amplitud pequea, tendremos que
aproximadamente la fuerza neta sobre el cuerpo estar en ese plano, ser hacia el centro, y
proporcional a la distancia al mismo.
Entonces, dando impulsos suaves adecuados se obtienen movimientos en los cuales el hilo describe
distintos tipos de conos, mientras el cuerpo en el extremo dibuja aproximadamente elipses que pueden
variarse desde circunferencias hasta lneas diametrales.

Cuantificacin
Las consideraciones cunticas siempre intervienen en el dominio atmico planteando la
cuantificacin de la energa total y mdulo y una componente de la cantidad de movimiento
angular.
Ahora bien, la cuantificacin de todos los aspectos relacionados con el momento angular
orbital slo depende de que la fuerza sea central, y es independiente de la funcin Ep(r), es
decir es la misma que ya hemos visto.
Lo nico que en la fuerza elstica es diferente de lo que vimos en la fuerza coulombiana, es la
frmula para los niveles permitidos de la energa.
Ahora tenemos niveles dados por el nmero natural n comenzando desde 0, que son:

1 k
En = n (10.18)
2 m

Recordando que la frecuencia es f = 1 2 k m , a veces es importante escribir:


En = (n + ) h f (10.18)
Veremos ms detalles en el captulo de Vibraciones y Rotaciones Moleculares.
307

10.4.- La reduccin al Sistema Centro de Masa.


Ahora debemos prestar atencin a un detalle que hemos pasado por alto. Los movimientos
posibles que hemos estudiado de una partcula de masa m alrededor de un hipottico centro de
fuerza que est fijo en el origen de coordenadas, corresponden a una situacin ideal.
En la prctica la fuerza sobre la partcula de masa m es aplicada por algn cuerpo, el cual tiene
masa M >> m, y est, segn el Principio de Accin y Reaccin, necesariamente sometido a la
fuerza que sobre l ejerce la partcula; y por no ser un cuerpo libre de fuerzas, no puede
considerarse estrictamente en reposo en el origen de un buen sistema de referencia.
El cuerpo central tambin tiene que moverse!. Claro que si su masa es infinitamente grande,
tiene que moverse infinitamente poco, y tenemos la situacin ideal que hemos estudiado. Ella
puede considerarse como caso lmite cuando la masa del cuerpo central es suficientemente
grande cosa que se cumple bastante bien en el Sistema Solar y en el tomo.
Pero no se cumplira si pensamos en dos estrellas de masas parecidas, o dos tomos
componentes de una molcula. En esos casos cada cuerpo debera considerarse en movimiento
alrededor del centro de masa del sistema, que es el punto que podra considerarse en reposo
en el origen.
Consideremos dos partculas, de masas m1 y m2, unidas por alguna fuerza mutua de atraccin.
Estas partculas pueden moverse de muy variadas maneras, combinando traslacin con
rotacin, pero lo hacen de manera que su centro de masa se mueve con movimiento rectilneo
uniforme.

v1
m1
F1
vCM
CM

F2 v2 Fig. 10.22: Mientras dos partculas viajan


aplicndose fuerzas mutuas, sus trayectorias
m2 son curvas pero el CM viaja con MRU.

Si a este sistema de dos partculas lo acompaamos con un sistema de coordenadas que viaje
con la misma velocidad del CM, en este sistema, denominado sistema CM, dicho punto estar
en reposo en un lugar arbitrario, que elegimos como origen. Habremos descontado as el
movimiento del centro de masa, y tendremos a la vista los posibles movimientos intrnsecos
de las partculas.
La posicin del CM en cualquier sistema est dado por:

m1 r1 m2 r2
rCM (10.19)
m1 m2
Esto significa que, en el sistema CM, en el cual el CM se elige como origen, tendremos

rCM 0 , y por lo tanto:

m1 r1 m2 r2 0 (10.20)
308

m
r2 1 r1 (10.20)
m2
Es decir que ambas partculas tienen el mismo movimiento con respecto al centro de masa, ya

que r1 y r2 son vectores que se mantienen opuestos sobre la misma recta, y cuyos mdulos se
mantienen en una relacin fija:
r2(t) = (m1/m2) r1(t)
Es como si lo que hace una partcula fuera copiado exactamente por la otra del lado opuesto
del CM, con un factor de escala que es la relacin entre las masas.
Ahora bien, a lo largo de este captulo hemos considerado que 1 era la partcula mvil, y 2 era
el cuerpo fijo en el origen (lo cual se justificaba porque m2 >> m1), de modo que cuando

considerbamos que r era la posicin del cuerpo 1 con respecto al origen, tambin lo era con
respecto al cuerpo 2. Pero ahora este cuerpo ya no est en el origen, y debemos a precisar

mejor, de manera que definimos claramente que utilizaremos r para designar:

r = r1 r2

Y si eliminamos r2 de esta expresin utilizando (10.20) se encuentra que el movimiento de la
partcula 1 con respecto a la 2 tambin reproduce al de la partcula 1 con respecto al CM, con

otro factor de escala dado por (notar que r = r1 r2 , pero r = r1 + r2):

m m2
r ( t ) = r1 r2 = 1 r1 ( t ) (10.21)
m2
v1
r1 m1

CM
r
m2 r2 Fig. 10.23: En el sistema CM ambas partculas
describen movimientos de la misma forma, opuestos
v2 respecto del CM, con un factor de escala constante.


Ahora vemos que al no tomar r1 con respecto al CM, como debi ser, hemos estado
introduciendo (sin saberlo) un factor de escala fijo en las distancias:
m1 m 2 m
=1 1 (10.22)
m2 m2
Este factor tambin ha afectado a las velocidades y cantidades de movimiento, ya que, si se

deja transcurrir un pequeo intervalo de tiempo t, los vectores posicin r1 y r2 sufren
desplazamientos que tambin cumplen estas relaciones, las cuales luego, dividiendo por t,
tambin se encuentran entre las velocidades:
m m m2
v 2 1 v1 ; v= 1 v1 (10.23)
m2 m2

Ahora bien, todo el comportamiento mecnico de los cuerpos se deduce de la Ley del Impulso
aplicada en el sistema CM. Esta ley, para el movimiento del cuerpo 1 dice:

F(r) t m1 v1
309

As la Ley est correctamente aplicada, en el sistema CM, pero dado que la fuerza que
estamos considerando est dada en funcin de r, la distancia entre los cuerpos, y no en funcin

de r1, puede sernos cmodo escribir la Ley tambin en trminos de r y v .

Si para ello reemplazamos v1 = m2 (m1 m2 v , tomado de (10.23), queda:
m1 m2
F(r ) t v (10.24)
m1 m2

Pero esto nos dice que aplicando correctamente la Ley del Impulso, los vectores r y v (que
son los que dan el movimiento del cuerpo 1 respecto del 2) evolucionaran en el tiempo como
si la fuerza estuviera actuando sobre un cuerpo de masa , que llamaremos masa reducida,
dada por:
m m
= 1 2 (10.25)
m1 m 2

O sea, afectamos la masa por un factor de escala que compensa al que afecta a la velocidad, y
tenemos la masa reducida. Notemos que, si m2 >> m1, resulta m1. Esto justifica una vez
ms que la aproximacin hecha a lo largo del captulo ha sido buena.

Ejemplo desarrollado
Considere posibles vibraciones de la molcula de H35Cl, y de la de H2 (para este planteo considere las
masas en u.m.a., sin expresarlas en kg).
a) Sabiendo que las ligaduras H-Cl y H-H tienen longitudes de equilibrio 127,510-12 m, y 74,110-12 m
respectivamente, encuentre la ubicacin del centro de masa de cada molcula, y haga un esquema a
escala, ubicando cada tomo en un sistema de coordenadas con el CM fijo en el origen de la
respectiva molcula.
b) Encuentre la masa reducida para cada caso, e interprete el significado de cada valor obtenido. En
particular explique por qu la del sistema HCl es muy parecida a la del hidrgeno, y la otra no.

Desarrollo
a) Dado que la masa del 35Cl es 35,5 uma, es decir 35,4 veces la del H, en la molcula HCl el centro
de masa estar exactamente ese nmero de veces ms cerca del Cl que del H, mientras que en la
molcula H2 el CM estar exactamente al medio entre ambos.
-3,6 124
x

50 100 (10-12 m)
CM
Ncleo H
Ncleo Cl
-37 37
x
50 (10-12 m)
Ncleo H CM Ncleo H
b) La masa reducida resulta: HCl = 0,97 u.m.a. ; HH = 0,5 u.m.a. Para la molcula de HCl, tenemos
un protn oscilando unido a un ncleo 35,4 veces ms masivo. El ncleo de Cl se mantiene cercano al
centro de masa, con oscilaciones de muy pequea amplitud: cualquier movimiento del protn es
310

copiado por el Cl con amplitud 35,4 veces menor. De manera que para el protn esto es muy
parecido a estar unido a un punto fijo, por lo cual la masa reducida describe una partcula hipottica
casi de la misma masa que el protn.
Muy distinta es la situacin en la molcula de H2: cada protn est unido a otra partcula que se mueve
exactamente tanto como l. Para el resorte es como si tuviese un extremo fijo, y en el otro una
partcula cuya masa fuese la mitad de la que realmente hay.

NOTA: LOS RAZONAMIENTOS FUERON CORRECTOS


Aunque las expresiones (10.24) y (10.25) son concluyentes y no es necesario
agregarles ms nada, es interesante la siguiente reflexin.
Todas las deducciones hechas a lo largo del captulo sobre las caractersticas
de los movimientos se basaron en dos cosas:
1) En que la fuerza era central.
2) En la forma de la funcin Ep(r).
Y esas dos cosas se mantienen si pasamos del sistema (incorrecto) en el cual el
cuerpo 2 estaba en el origen, al sistema con CM en el origen, ya que:
1) La fuerza sobre 1, directamente hacia 2, tambin apunta exactamente
hacia CM.
2) La forma de las funciones Ep(r) involucradas es la misma, salvo factores
constantes, que la de las funciones Ep(r1) que se obtienen sustituyendo r por
medio de la expresin (10.21).
De manera que las elipses y trayectorias varias que hemos estudiado para el
cuerpo 1 alrededor del 2, tambin son vlidas para cada cuerpo alrededor del
CM, con el factor de escala adecuado.


Por ltimo, si vamos a resolver cualquier problema en trminos de r y v , debemos expresar
correctamente la energa cintica y el momento angular orbital.

Es decir, para aclarar: vamos a trabajar en trminos de r y v , que expresan las variables del
cuerpo 1 con respecto al 2, pero no consideramos que el cuerpo 2 est fijo en el origen, porque
el que est fijo es CM.
Si considersemos al cuerpo 2 fijo en el origen, nosotros diramos: Ec = m1 v2, y L = m1 v
r, lo cual sera incorrecto (comparar con (10.26) y (10.27)).
Nosotros decimos: Ec = m1 v12 + m2 v22, y LCM = m1 v1 r1 + m2 v2 r2, porque hablamos
en el sistema CM, y queremos expresar estas cosas en trminos de r y v.
Entonces, para ello recurrimos a los factores de escala dados por (10.23):
m v2 m m2 v2
Ec = m1 v12 + m2 v22 = 1 1 2 12 1
2 2 m2
m1 v12 m 2 m1
=
2 m 2
1 m1 m 2 2
= v
2 m1 m 2

= v2 (10.26)
311

Y de la misma manera, para el momento angular orbital, LCM = m1 v1 r1 + m2 v2 r2, con los
mismos reemplazos, se obtiene:

LCM = v r = 2 r (10.27)

Donde es la velocidad angular: = v1 / r1 = v2 / r2 = v / r . Estas igualdades se justifican


porque tanto la lnea entre los cuerpos, como las lneas entre cualquiera de ambos cuerpos y el
CM, estn alineadas y rotan todas juntas.
La conclusin prctica es muy simple:

Cuando dos cuerpos se mueven bajo la accin de fuerzas mutuas alineadas con la
recta que los une, el movimiento relativo de uno de ellos respecto del otro puede
explicarse como si l tuviese la masa reducida, , y el otro estuviera fijo en el origen.

Por ahora dejaremos aqu el tema, y lo retomaremos al tratar las vibraciones y rotaciones
moleculares.

Ejemplo desarrollado.
Consideremos dos cuerpos el 1 de masa m1 = 1 kg, y el 2, de masa m2 = 3 kg, unidos por medio de un
resorte de 40 cm de longitud en equilibrio, y constante elstica k = 750 N/m.
Ambos cuerpos pueden oscilar sin rozamiento sobre una pista horizontal rectilnea a lo largo del eje x.
m2 k m1 x

a) encuentre la ubicacin del centro de masa, y muestre que si se ubica el origen del eje x sobre l,
entonces la posicin de equilibrio del cuerpo 1 estar en x01 = 30 cm, y la del 2 en x02 = 10 cm.
b) Muestre que si en el sistema CM el cuerpo 1 oscila 6 cm a cada lado de su posicin de equilibrio, el
2 oscilar 2 cm a cada lado de su propia posicin de equilibrio.
c) Calcule la fuerza que aplica el resorte a cada cuerpo cuando ambos estn en el mximo
apartamiento, y en la mxima proximidad. Comprelas.
d) Con los valores calculados en c), y los correspondientes apartamientos de cada cuerpo de su
respectiva posicin de equilibrio, muestre que es como si cada cuerpo estuviera unido al origen (y no al
otro cuerpo) por un resorte diferente. Calcule la longitud de equilibrio y la constante elstica de cada
uno.
e) Muestre que cada cuerpo oscilando independientemente en el extremo de su resorte diferente,
oscilara con la misma frecuencia que el otro, y que esa es la misma frecuencia de oscilacin de un
cuerpo de masa reducida en el extremo del resorte real.
Desarrollo
a) Si el resorte est en equilibrio un cuerpo estar a 40 cm de distancia del otro. Ahora, con respecto al
cuerpo 2 ubicado a la izquierda, el CM estar en xCM = (30 + 140)/4 = 10 cm.
De manera que ubicando el origen de un nuevo eje x en el CM, tendremos el cuerpo 1 ubicado en x10
= 30 cm, y el 2 en x20 = -10 cm. Como el resorte est en equilibrio no hay fuerza sobre ninguno de los
cuerpos, y esas posiciones son las posiciones de equilibrio de cada uno en el sistema CM.
312

CM
m2 m1 x

-10 cm x=0 30 cm
x20 x10
x10 - x20 = 40 cm

b) Si alejamos ambos cuerpos una cierta distancia y luego los soltamos a los dos simultneamente, las
oscilaciones continuarn automticamente manteniendo fijo el CM. Si queremos que el CM quede en
el origen, tenemos que soltar los cuerpos con el CM all. Para ello, si inicialmente el cuerpo 1 parte de
en x1 = 36 cm, y el 2 debe partir de x2 = 12 cm (aplicamos siempre la relacin x2 = (m1 / m2) x1).
m2 CM m1 x

x=0
x2 x20 x10 x1

En estas condiciones cada cuerpo oscilar como se ha pedido.


c) Cuando los cuerpos estn en la mxima separacin, 48 cm, sobre cada uno actuar una fuerza de
mdulo F = k0,08 = 60 N, en sentido de atraerse.
En la mxima proximidad, 32 cm, la fuerza tambin ser F = k0,08 = 60 N, en sentido de repelerse
mutuamente los cuerpos.
Las fuerzas son iguales y opuestas sobre ambos cuerpos en cada instante, por P. de Accin y
Reaccin, y tambin en cada cuerpo son de igual mdulo y sentido opuesto en los extremos de su
oscilacin.
60 N CM 60 N x

2 cm mxima separacin
6 cm 6 cm
2 cm
60 N 60 N x
CM

x=0 mxima proximidad

x20 x10

d) Para el cuerpo 1, cuando l se aleja 6 cm de la posicin de equilibrio, o del CM, la fuerza que le
aplica el resorte es 60 N. Entonces es como si estuviera sujeto a un resorte de k1 = 60/0,06 = 1000
N/m, cuyo otro extremo estuviese fijo en el CM, es decir en el origen (y no donde realmente est: en el
otro cuerpo mvil).
Para el cuerpo 2, cuando l se aleja 2 cm de la posicin de equilibrio, o del CM, la fuerza que le aplica
el resorte es 60 N. Entonces es como si estuviera sujeto a un resorte de k 2 = 60/0,02 = 3000 N/m (cuyo
otro extremo estuviese fijo en el origen).
e) Con estos dos resortes distintos el cuerpo 1 oscilara con frecuencia dada por 1 = 1000 1 = 31,62
1/s; y para el cuerpo 2 tendramos 2 = 3000 3 = 31,62 1/s.

Por otra parte, la frecuencia de un cuerpo de masa reducida = 31/ (3+1) = 0,75 kg, oscilando en el
extremo del resorte original, de constante k = 750 N/m, sera la misma: = 750 0,75 = 31,62 1/s.
313

CAPTULO 11:
Vibraciones y rotaciones de molculas
diatmicas.
Algunos conceptos que hemos visto pueden ser aplicados con facilidad al estudio de los
movimientos de algunas molculas simples, como las diatmicas.
Desde el punto de vista de la dinmica se pueden ignorar muchas de las complejidades de la
nube electrnica que une los tomos en la molcula, porque lo que interesa es que toda la masa
de esta nube es despreciable, y el sistema se caracteriza por tener casi toda la masa concentrada
en los ncleos.
Esto nos permite elaborar un modelo el cual la molcula se asimila a un sistema dos partculas
unidas por un resorte. Este sistema puede realizar movimientos vibratorios que consisten en
alejamientos y aproximaciones entre las partculas, as como rotaciones en las diversas
orientaciones del espacio.
En este captulo se estudian estas posibilidades enmarcadas en detalles experimentales reales.

11.1.- Vibraciones moleculares

Energa potencial interatmica en molculas diatmicas.


La unin de dos tomos en una molcula diatmica se produce por la distribucin de la carga
electrnica, que si alcanza valores adecuados entre los ncleos, ejerce fuerzas atractivas sobre
los mismos suficientes para equilibrar la repulsin electrosttica entre ellos.
La forma de la nube electrnica en sus diferentes estados orbitales posibles depende de las
leyes cunticas, y aqu basta con saber que la carga negativa entre los ncleos produce un
efecto atractivo sobre los ncleos positivos, y que cuando las molcula se forma, hay una
distancia de equilibrio internuclear, en la cual la atraccin mencionada equilibra a la repulsin
entre los ncleos.
Un agente externo debera hacer trabajo positivo tanto para separar como para aproximar
los tomos a partir de una situacin inicial hipottica con la separacin de equilibrio, porque
para distancias mayores que la de equilibrio predomina la atraccin, y para menores la
repulsin.
314

Ahora bien, dado que las fuerzas elctricas se debilitan con la distancia, es claro que si los
ncleos se separan suficientemente ms all de esa distancia de equilibrio, la fuerza de
atraccin necesariamente ir desapareciendo, hasta que finalmente podremos considerar que
se ha disociado la molcula.
Por otra parte, si los ncleos se aproximan ms all de la separacin de equilibrio, los estados
orbitales electrnicos no podrn mantener la cantidad de carga negativa entre ellos, y
predominar la repulsin, que podr llegar a ser infinita: la distancia internuclear nunca podr
llegar a ser cero.
Esto es lo que se lee en una grfica tpica de energa potencial Ep(x), donde x es la distancia
interatmica (lase entre los ncleos) en una molcula diatmica (figura 11.1).
Esta funcin se determina con ayuda de modelos tericos y de datos experimentales, como por
ejemplo la energa necesaria para disociar las molculas, la distancia de equilibrio, etc.
E (J)
Ep(x) nube electrnica
E*p(x)
x0 x


(a) x (b)
E(x0)=
Fig. 11.1: a) Energa potencial interatmica en una molcula diatmica. Ep* representa una funcin
de segundo grado que puede aproximar a la funcin real Ep en la cercana de la posicin de
equilibrio x0. b) Esquema de la molcula que muestra que x representa la distancia entre los ncleos.

Oscilaciones pequeas en la cercana de la posicin de equilibrio estable


Ahora bien, dada la forma de la funcin Ep(x), que por razones fsicas elementales siempre
deber ser similar a la mostrada en la figura, es claro que en la cercana de la posicin de
equilibrio siempre podr ser aproximada razonablemente con una funcin de segundo grado
como la Ep*(x) mostrada en lnea de trazos.
Para energas totales negativas no mucho mayores que E(x0), la distancia internuclear x
oscilar dentro de valores prximos a x0, y no habr gran diferencia entre la energa que indica
Ep o la que indica Ep*.
Ahora bien, una funcin de segundo grado, como Ep*, corresponde a una fuerza elstica, y en
estas condiciones las oscilaciones sern armnicas. Esto nos indica uno de los aspectos ms
importantes de la fuerza elstica: sirve para aproximar cualquier caso de vibracin cerca de
una posicin de equilibrio, como se discute a continuacin.
Sabemos que la energa potencial de una fuerza elstica con constante elstica k, y con
posicin de equilibrio en x0 es, en forma general Ep*(x) = k (x x0) + C.
Para este caso queremos hacer coincidir Ep*(x) con la funcin experimental Ep(x) en la
vecindad de x0, y para ello la constante C debe ser el valor de Ep(x) en el mnimo, , como
puede verse sustituyendo x = x0.
De manera que podemos aproximar el problema experimental real, con la funcin energa
potencial elstica:
Ep*(x) = k (x - x0)2 (11.1)
315

El valor de la constante k, en la grfica, regula que las ramas de la parbola sean ms o menos
cerradas, y se relaciona directamente con las caractersticas de las ligaduras qumicas entre los
tomos.
As por ejemplo, es importante saber que, en trminos generales, es tpico encontrar los
siguientes rdenes de magnitud en los valores de la constante de fuerza en funcin del tipo de
ligadura:
Uniones covalentes simples como en H2 k = 5102 Nm-1

Uniones covalentes dobles como en O2 k = 12102 Nm-1

Uniones covalentes triples como en N2 k = 20102 Nm-1

Uniones inicas como en NaCl k = 1102 Nm-1

Para poder utilizar estas ideas en situaciones prcticas del dominio atmico es necesario tener
en cuenta el problema de la reduccin al sistema centro de masa, debido a que la energa
potencial est expresada en trminos de la distancia interatmica x, como si uno de los tomos
estuviese fijo en el origen, pero para aplicar las Leyes de la Dinmica, lo que debe estar fijo es
el CM.
Como ya vimos en el captulo anterior, para esto lo nico que necesitamos es hacer los
clculos como si tuvisemos un cuerpo de masa reducida, , en el extremo de un resorte de
longitud de equilibrio x0, y constante elstica k, con el otro extremo fijo en el origen.
De manera que la frecuencia de las oscilaciones estar dada por la expresin que corresponde
a las oscilaciones elsticas, 2 = k/m, pero con m sustituido por :
1 k 1 k
f = (11.2)
2 m1 m 2 2
m1 m 2

Siendo la masa reducida, que ya hemos encontrado en el captulo anterior:


m1 m 2
(11.3)
m1 m 2
Como veremos a continuacin, experimentalmente resulta bastante sencillo determinar la
frecuencia de vibracin de estas molculas. De manera que, cuando que las masas de los
tomos son conocidas, a partir de la frecuencias puede determinar la constante de fuerza de la
unin (utilizando (11.2), para tener datos de la unin qumica.
Ahora nos falta tratar el problema de cmo se determina la frecuencia de movimientos de
partculas que por su pequeez no pueden ser vistas.

Nociones de espectroscopa de vibraciones moleculares.


Cuando lo que oscila es un tomo, la determinacin de las frecuencias de oscilacin no se
puede hacer directamente, observando cmo oscila una partcula tan pequea, sino que lo que
se hace es analizar la radiacin absorbida o emitida por la partcula oscilante cuando se
interacta con ella perturbndola de alguna manera.
316

Esto es el tema de estudio de la espectroscopa, que es el estudio de los espectros de las


radiaciones. Hablemos un poco sobre los espectros.
Si un haz de luz blanca se hace pasar por un prisma dispuesto adecuadamente, puede lograrse
que luego contine su marcha descompuesto en un conjunto de haces de colores que inciden
separados sobre una pantalla. Si cada uno de estos haces de color se hace pasar nuevamente
por un prisma, ya no se logra una ulterior descomposicin, mientras que si se logra reunir
todos los colores, nuevamente se obtiene el color blanco.
Esto nos sugiere que el haz blanco es mezcla de ciertos colores que constituyen componentes
puros, es decir que ya no son mezcla de otros. El conjunto de todos los colores componentes
del haz, presentados desplegados sobre la pantalla de observacin constituye el espectro de
colores componentes de la luz blanca, o ms simplemente, el espectro de la luz blanca. El
aparato capaz de mostrar separadamente los componentes del haz, en este caso el conjunto de
prisma + pantalla dispuestos adecuadamente (en otros casos puede haber muchos elementos
ms sofisticados), constituye el espectroscopio.
El ojo humano no es buen analizador espectral, pues es sensible slo al efecto global de la
mezcla, y as resulta que la luz blanca, o de cualquier color, puede obtenerse con diferentes
mezclas de colores componentes, sin que el ojo perciba diferencia. Son necesarios diferentes
aparatos y tcnicas espectroscpicas para analizar la composicin espectral de diferentes
radiaciones.
La luz est constituida por ondas de campos electromagnticos. Las ondas electromagnticas
en general son generadas por vibraciones de partculas cargadas, y su espectro completo
abarca, en teora, desde la frecuencia cero hasta infinito, aunque en la prctica las de
frecuencia ms baja suelen ser las ondas que se utilizan para las transmisiones radiofnicas, es
decir, las ondas de radio, de algunos miles de ciclos por segundo, y las de frecuencia ms
alta son los rayos gamma, y los rayos csmicos, con frecuencias de 1023 ciclos por segundo.
La parte del espectro electromagntico que impresiona nustra retina como luz visible es la que
tiene frecuencia desde poco ms de 1014 ciclos por segundo para el color rojo, hasta casi 1016
ciclos por segundo para el color violeta.
Es importante saber que, para cada zona del espectro electromagntico existen diferentes
mtodos, aparatos, y sistemas de deteccin y estudio. As podemos mencionar la
espectroscopa UV (para detectar radiacin ultravioleta), la espectroscopa IR (para
radiacin infrarroja), la de microondas, etc., cada una con diferentes aparatos y tcnicas
especficas. El ultravioleta es la parte de frecuencias ms all de los 1016 ciclos por segundo, y
el infrarrojo es la parte de frecuencias menores que 1014 ciclos por segundo. En esta parte
encontraremos las vibraciones moleculares que ahora nos ocupan. Si llegsemos a frecuencias
de 1011 ciclos por segundo, ya hablaramos de microondas.
Podemos tener un ejemplo diferente, en el campo de las vibraciones mecnicas, pensando en
los sonidos. Las ondas sonoras son vibraciones mecnicas de elementos macroscpicos.
Analizando un sonido cualquiera se encuentra que en general no contiene una frecuencia bien
definida, sino que suele ser una mezcla de vibraciones de diferentes frecuencias. Con un
analizador adecuado puede obtenerse todo el espectro de frecuencias componentes. El sonido
de cada componente de frecuencia bien definida, constituye una nota pura.
Para el caso que nos interesa ahora, de la absorcin o emisin de ondas electromagnticas por
vibraciones de partculas cargadas en el dominio atmico o molecular, la teora que debe
utilizarse para estudiar sus caractersticas es la teora cuntica, no la clsica.
317

El caso con la teora cuntica


Un elemento fundamental que hay que considerar es la cuantificacin de los valores de
energa, que para este caso nos dice que los valores permitidos de la energa son:
En = (n + ) h f + Ep(x0) (11.4)
Donde:
n es cualquier nmero natural de 0 en adelante.
f es la frecuencia de oscilacin determinada por la teora clsica: f = 1 2 k

h 6,62610-34 Js , es la constante de Planck,


Ep(x0) es el valor arbitrario de la energa potencial en la posicin de equilibrio (para este
caso sera , aunque no tiene importancia en lo que vamos a hacer).

En la siguiente figura se ilustran niveles posibles para nuestro caso de inters. Si es el valor
de la energa potencial en el fondo del pozo, el estado de menor energa total posible tiene
energa E0 = + h f (ver figura 11.2)).
Aqu encontramos algo que no tiene explicacin clsica: an con la mnima energa posible,
tiene lugar una vibracin con cierta cantidad de energa cintica media que no puede quitarse
al sistema. Si la temperatura es suficientemente baja, como puede considerarse la temperatura
ambiente en general, el sistema estar en este nivel de menor energa, denominado
fundamental.
La cantidad de energa D0 = h f, se denomina energa de disociacin, pues es la
energa que se debe suministrar al sistema (supuesto que se parte de este estado de mnima
energa) para que la energa total llegue al valor E = 0, con el cual x , y la molcula se
disocia (ver figura).
Ep(x) En lnea de trazos los niveles
E (J)
que corresponderan a oscila-
ciones elsticas puras.

0 x0 x
E4
E3
E2 D0
E1
hf
E0
E(x0) = - hf
2

Fig. 11.2: Se indican niveles de energa total para las vibraciones posibles de los
tomos en una molcula biatmica. En lnea de trazos se indica una funcin
cuadrtica que aproxima la parte cercana a la posicin de equilibrio. Esta funcin
corresponde a una fuerza elstica pura, y para ella vale exactamente la frmula En =
(n + ) h f . sta constituye una buena aproximacin para los niveles ms bajos de
la vibracin de la molcula.
318

No slo la energa total de la oscilacin est cuantificada, sino que la energa de la radiacin
tambin se considera que viaja en paquetes denominados fotones, cada uno de energa hf.
Cuando una partcula oscilando con la energa de un nivel cualquiera En , es perturbada por
algn agente externo, como puede ser un haz de radiacin electromagntica, puede absorber (o
perder) un fotn energa y pasar a un nivel de ms (o menos) energa, siempre que la energa
del fotn sea la exacta para pasar a otro nivel permitido de energa de vibracin. Es decir, en
una radiacin con mezcla de fotones de varias frecuencias, la partcula oscilante slo podr
absorber fotones de la energa que le permita llegar exactamente al nivel En+1 (si los hay en el
haz). Y si la partcula pierde energa, lo hace emitiendo la cantidad hf exacta para quedar en
el nivel En-1.
NOTA:
Si el electrn en el estado n es afectado por un choque con otra partcula,
puede saltar a cualquier nivel m, absorbiendo la energa exacta (m n) h f.
Pero si el choque es con un fotn, es decir absorbe (o emite) radiacin
electromagntica, hay que aplicar una regla de seleccin, que dice que n =
1: un electrn sujeto a una fuerza elstica slo puede absorber/emitir fotones
saltando a un nivel consecutivo superior/inferior.

Ahora bien, tenemos que la energa exacta para saltar al prximo nivel superior, o inferior, en
el caso de las oscilaciones elsticas de frecuencia f, vale:
En+1 - En = hf (11.5)
Pero esa es la energa de un fotn de esa misma frecuencia. De manera que, en resumen, un
conjunto de sistemas en estas condiciones, slo absorber de un haz que lo perturbe, un
conjunto de fotones de la frecuencia exacta prevista por la teora clsica de la oscilacin, o en
caso contrario, si emite, slo emitir fotones de esa misma frecuencia exacta.
Para el caso que nos interesa, podemos considerar el siguiente procedimiento experimental.

Espectros de absorcin y emisin


Una fuente de radiacin electromagntica emite un haz de ondas de frecuencia f hacia una
muestra de la sustancia que se supone contiene molculas con tomos que pueden vibrar con
frecuencia f0.
Dos detectores registran la intensidad de la radiacin que llega a dos lugares diferentes:
D1 detecta en la direccin de marcha original del haz, lo que recibe es lo que hubiera llegado
sin interposicin de la muestra, menos lo que sta absorbe o dispersa en otras direcciones;
D2 detecta en una direccin muy distinta, por ejemplo a 90o de la direccin original; all no
detectara nada proveniente directamente del haz original, y slo puede recibir algo que sea
irradiado por la muestra.
319

muestra
D1 fuente

irradiacin por parte


de la muestra
D2
Fig. 11.3: Esquema de un dispositivo experimental para detectar absorcin o
emisin de radiacin por parte de una muestra.

De manera que, mientras la frecuencia de la radiacin emitida por la fuente, que se va


variando lentamente, no coincide con f0, la muestra se comporta prcticamente como
transparente para ella: no absorbe energa, y deja pasar toda la radiacin sin afectarla. En esta
situacin D1 registra algo constante, y D2 no registra nada.
Por otra parte, cuando f se acerca mucho, y pasa por el valor f0 , una gran cantidad de energa
es absorbida y reirradiada en todas direcciones. D1 registra una disminucin en la intensidad
recibida, mientras D2 registra un pico de emisin.
intensidad en D1 intensidad en D2

f0 f f0 f
Espectro de absorcin Espectro de emisin

Fig. 11.4: Espectros de absorcin y emisin para una muestra que tiene un nica frecuencia de vibracin.

Cualquiera de los espectros nos informa de manera equivalente sobre la frecuencia de


oscilacin del sistema. Con esta idea esquemtica, se puede trabajar slo para detectar
emisin, o slo absorcin, dependiendo de muchas posibilidades de los diferentes aparatos y
tcnicas, detalles en los que no estamos interesados aqu.
La siguiente tabla 11.1 contiene los datos que nos permitirn considerar muchas situaciones
reales.

NOTA: LA VISIN CLSICA.


En la teora clsica hubiramos planteado que los sistemas atmicos que
podan vibrar con frecuencia f0, al ser sacudidos o excitados por radiacin de
frecuencia f, slo entran en resonancia, absorbiendo y reirradiando en todas
direcciones cantidades apreciables de energa, cuando la frecuencia
excitadora coincide con la f0 propia del sistema.
ste es un caso en el cual con la teora clsica se llega a la misma conclusin
que con la cuntica.
320

Tabla 11.1: datos de ligaduras de algunas molculas diatmicas: frecuencia de


oscilacin, constante de fuerza y longitud de la ligadura, y energa de disociacin.
Energa de
frecuencia k longitud ligadura
Molcula disociacin
1013 s-1 ( Nm-1 ) ( pm ) ( kcal / mol )
H2 12,48 520 74,1 104
D2 8,97 530 74,1 104
HCl 8,66 480 127,5 103
O2 4,67 1140 120,7 119
N2 6,99 2250 109,4 227
CO 6,43 1860 112,8 257
NO 5,63 1550 115,1 150
NaNa 0,473 17 307,8 18
NaCl 1,13 120 236,1 98
KCl 0,834 82 266,7 101
F2 2,68 460 142 38
LiCl 1,94 143 202 111
ClCl 1,67 320 198,8 58

Ejemplo desarrollado 1
Dadas las siguientes frecuencias encontradas experimentalmente, encuentre aproximadamente la
constante de fuerza de la ligadura, y segn ese valor determine el tipo de unin que puede estar
involucrada.
F2 2,671013 s-1 ; CO 6,431013 s-1 ; KCl 0,8341013 s-1
Desarrollo
Para cada molcula calculamos la masa reducida, y luego obtenemos la constante de fuerza con la
expresin k = 4 2 f2 , obtenida despejando de (11.2).
Obtenemos as:
F2 : = 9,5 uma = 1,5810-26 kg , k = 444 N/m
CO : = 6,86 uma = 1,1410-26 kg , k = 1858 N/m
KCl : = 18,61 uma = 3,0910-26 kg , k = 85 N/m
Considerando los valores tpicos aproximados de la constante de fuerza en diferentes ligaduras, que
se dieron hace varias pginas en este captulo, encontramos que podemos clasificar:
F2 : covalente simple (valor tpico 500 N/m)
CO : covalente triple (valor tpico 2000 N/m)
KCl : inica (valor tpico 100 N/m)
321

Ejemplo desarrollado 2
La ligadura H-H en la molcula de H2 tiene una longitud de 74,1 pm, una energa de disociacin de 104
kcal/mol, y una constante de fuerza de aproximadamente 520 N/m.
a) Encuentre la frecuencia de las vibraciones posibles de esta molcula. Corroborar con valores de
tablas.
b) Suponiendo que el sistema se pudiera considerar un oscilador armnico para todas las energas
hasta llegar a la disociacin, y que fuera vlida exactamente la frmula cuntica de los niveles de
energa En = (n + ) h f , encuentre cuntos niveles de vibracin posible habra.
c) Considerando que el deuterio, o hidrgeno pesado (D = 2H), slo difiere del hidrgeno en la masa
del ncleo, ya que ste est formado por un protn junto con un neutrn, explique qu valores espera
para la molcula D2 (en funcin de los valores correspondientes a la de H2) para la longitud de la
ligadura D-D, para la energa de disociacin, para la constante de fuerza, y para la frecuencia de las
oscilaciones. Corroborar con valores de tablas.
Desarrollo
a) Para calcular la frecuencia necesitamos la constante de fuerza, que es dato, y la masa reducida.
11
0,5 uma 0,8310-27 kg
11
1 520
f 12,61013 1/s
2 0,83 10 27
(Dentro del 1% corrobora el valor de la tabla: 12,481013 1/s).
b) Debemos contar cuntas veces entra el espaciado entre niveles en la energa de disociacin.
Espaciado entre niveles: h f 6,62610-3412,61013 8,3510-20 J.
104 4180
Energa de disociacin: D0 104 kcal/mol 23
7,2210-19 J
6,022 10
Nmero de niveles: parte entera de (D0/hf) = 8
Esto es una aproximacin considerando que el espaciado entre niveles se mantuviese constante e
igual al del oscilador armnico para todas las energas. No tenemos elementos para mejorar esta
aproximacin (un clculo mejor indicara que entran algunos niveles ms, como se podra corroborar
experimentalmente).
c) Para la molcula de deuterio se espera muy aproximadamente la misma energa de disociacin, la
misma longitud de la ligadura, y la misma constante de fuerza que para la de hidrgeno, porque la
nube electrnica que forma la ligadura slo depende de la carga nuclear, que es la misma para ambas
molculas.
En cambio se espera una diferencia importante para la frecuencia de oscilacin, ya que depende de la
masa.
Para el deuterio sera:
2 2
1 uma =1,6610-27 kg
22
1 520
f =8,911013 1/s
2 1,66 10 27
(Dentro del 1% corrobora el valor de la tabla: 8,971013 1/s).
322

11.2.- Rotaciones de molculas diatmicas rgidas

El espectro del rotador rgido.


El caso ms simple de rotacin de una molcula consiste en el caso de dos tomos unidos de
manera que pueda asimilarse a un sistema rgido, el cual constituir un primer modelo til
para lograr familiaridad con los conceptos ms bsicos del tema.
Este modelo se denomina modelo del rotador rgido. Ya hemos visto que la unin entre los
tomos de la molcula no es rgida, pero, dado que como veremos al final, se necesita mucha
ms energa para cambiar el estado de vibracin que el estado de rotacin, resulta que es fcil
encontrar situaciones en las cuales este modelo representa bastante bien la realidad. Por otra
parte, al final del captulo se muestra que es sencillo combinar la teora de la rotacin con la
de vibracin.
Consideremos dos partculas, de masas m1 y m2, unidas por algn elemento ideal que
mantiene una distancia d, fija, entre ambas. Este sistema, que es el llamado rotador rgido,
puede moverse de muy variadas maneras, combinando traslacin con rotacin, pero ya
sabemos que se lo debe describir desde el sistema CM, y all slo podr ejecutar rotaciones
puras alrededor de ejes que pasen precisamente por el CM.

m2
v2
CM

d
v1

m1
Fig. 11.5: Esquema de una posible rotacin de una molcula rotador rgido, descripta en el sistema CM.

En este caso tendremos dos partculas desarrollando un movimiento circular uniforme en el


plano de la hoja alrededor del CM, y, desde el punto de vista del sistema rgido completo,
podemos decir que es un cuerpo con momento de inercia I (respecto del CM), que realiza una
rotacin intrnseca. En el lenguaje de las rotaciones podemos escribir:

J = I = m1 r12 + m2 r22 (11.6)

Ec = I 2 = m1 r12 2 + m2 r22 2 (11.7)

Y ya hemos visto que estas expresiones tambin se pueden escribir en funcin de la masa
reducida y la distancia relativa entre las partculas, utilizando I = d2, con lo cual tendremos:

J = d2 = I (11.8)

J2
Ec = d2 2 = (11.9)
2 d2
323

La Mecnica Cuntica no impone directamente niveles cuantificados a la energa en este caso,


pero que s lo hace, de la manera que ya hemos visto, con la cantidad de movimiento angular,
y a travs de sta resulta la cuantificacin de la energa, de la siguiente manera.

El mdulo de la cantidad de movimiento angular resulta cuantificado segn el nmero j, que


puede tomar cualquier valor entero desde cero en adelante (j = 0, 1, 2, 3, ...), segn la
expresin (fue la expresin (13.13) del captulo anterior):
h
J = j( j 1) = j( j 1) (11.10)
2
En donde como ya hemos visto = h/2 1,0545910-34 Js, se considera la unidad de
momento angular para los fenmenos de nivel atmico.

De manera que, introduciendo en (11.9) la cuantificacin de J, resultan valores cuantificados


para la energa cintica, la cual adems, en este modelo simple, es tambin la energa total:

j ( j 1) 2
Ej = (11.11)
2 d2
El espectro contendr entonces las radiaciones con energas que correspondan a los saltos que
sean posibles entre estos niveles, los cuales estn dados por lo que se denomina reglas de
seleccin.
Para el caso de radiacin electromagntica perturbando a este sistema las reglas de seleccin
son:
j = 1 (11.12)
De manera que slo puede haber transiciones entre niveles consecutivos. Ahora tenemos que
la energa es una funcin cuadrtica del nmero cuntico j, y la energa del correspondiente
fotn, dada por la diferencia entre niveles consecutivos ser diferente para cada nivel, segn la
expresin:
2
Ej+1j = Ej+1 - Ej = [( j 1)( j 2) j( j 1)]
2 d2
( j 1) 2
= (11.13)
d2
Dado que a su vez la energa del fotn depende de la frecuencia segn: Ef = h f = 2 f , las
frecuencias que podrn encontrarse en el espectro estarn dadas por:
( j 1)
fj = (11.14)
2d 2
Es decir, a partir de la frecuencia f0 = /2d2, correspondiente a la transicin j = 1 j = 0,
el espectro contiene todos sus mltiplos enteros: 2 f0 , 3 f0 , . . . , etc.
324

Intensidad en un detector que


registra espectro de emisin

f0 2f0 3f0 4f0 f

Fig. 11.6: Frecuencias que contiene un espectro de rotacin pura.

Es importante tener idea del orden de magnitud de los valores de frecuencia que pueden
corresponder a estos espectros. Si tenemos en cuenta que la masas tpicas pueden ir desde 0,5
hasta 102 uma (tomemos 10 uma), mientras que la distancia internuclear tpica puede ser de
0,2 nm, obtenemos para f0 valores del orden de 1011 Hz (energa del orden de 10-22 J para cada
fotn), que estn entre las microondas y el infrarrojo lejano. En general las lneas
correspondientes a los dems valores de j, estarn tambin en el infrarrojo.
Ahora bien, la figura 11.6 muestra en qu valores de frecuencia se puede encontrar absorcin
o emisin de radiacin, pero no pretende indicar en qu valores la intensidad detectada puede
ser mayor o menor por eso se han dibujado todas las lneas de la misma altura. Para
especular sobre la posible altura de las lneas tenemos que pensar en otro aspecto del
fenmeno: cuando irradiamos una muestra con la frecuencia exacta que corresponde a una
absorcin, digamos del nivel j para pasar al j+1, slo podemos esperar mucha absorcin si
hay muchos sistemas que estn en nivel j.
Es decir esperamos una absorcin proporcional al nmero de sistemas que haya previamente
en el nivel j (lo que se denomina nmero de ocupacin del nivel).
No podra haber absorcin, por ejemplo, si previamente no hubiera ninguna molcula con
energa Ej.
Lo mismo vale para la emisin: la emisin desde el nivel j al j1, necesariamente tiene que ser
proporcional al nmero de sistemas que ocupan previamente el nivel j.
De manera que la intensidad de las distintas lneas del espectro debe depender fuertemente de
la temperatura, que es la que determina la distribucin, o sea la cantidad relativa de molculas
en cada nivel de energa. Si se tiene en cuenta que a temperatura ambiente las energas
cinticas promedio de las molculas debido a la agitacin trmica estn en el orden de k T
10-23300 > 10-21J, y que eso es lo que se puede esperar en promedio que reciba cada molcula
cuando es chocada al azar, se encuentra que (a temperatura ambiente) slo una porcin
pequea del total de molculas de un gas estar en los niveles ms bajos, y que
mayoritariamente estarn distribuidas al azar entre niveles correspondientes a valores de j
relativamente grandes.
Esto significa que al irradiar una muestra tpica con frecuencias en el infrarrojo lejano, se
detecte un espectro como el mostrado en la figura 11.7, en la cual se ve que la intensidad es
muy pquea en las frecuencias ms bajas, tiene un mximo en una zona que corresponde a la
energa media de la agitacin trmica, y decrece para frecuencias mayores, debido a la
ausencia de molculas con energas mucho mayores que el promedio correspondiente a la
temperatura.
325

Intensidad

Fig. 11.7: Forma tpica de un espectro de rotacin puro. La forma global est
regulada por la poblacin de los distintos niveles, la cual depende tanto de la
temperatura como de la multiplicidad de los niveles.

NOTA:1 LA MULTIPLICIDAD DE LOS NIVELES


Adems hay otro efecto que influye sobre la intensidad de las lneas del
espectro, y es de lo que se denomina multiplicidad de los niveles. sta tiene
que ver con el nmero cuntico m, que se refiere a las orientaciones en el
espacio de las rotaciones, como ya vimos en el captulo anterior.
El nmero m es entero, y va, de 1 en 1, desde j hasta +j.
Esto significa que para cada nmero de momento angular j (es decir, en este
caso para cada nivel de energa), hay 2 j + 1 valores posibles de m, y eso quiere
decir, 2 j + 1 posibilidades de existencia diferentes.
O sea, ejemplificando: hay un estado sin rotacin, de energa E0 = 0, tres
estados de rotacin 1, y energa E1, cinco estados de rotacin 2, y energa E2,
etc.
En la prctica esto se traduce en que cada nivel tiene probabilidad de poblarse
proporcional a las posibilidades que ofrece, o sea, a la multiplicidad 2 j + 1.
Esto tiende a hacer ms intensas las lneas correspondientes a las energas
mayores, mientras estas energas no lleguen a ser mucho mayores que la
energa media de la agitacin trmica.

NOTA 2: EL EFECTO CENTRFUGO


Como aclaracin vale decir que en la prctica las lneas no resultan
exactamente equiespaciadas porque para valores grandes de j, el aumento de
momento angular trae aparejado (debido a la fuerza centrfuga) un aumento
de la distancia interatmica d, que significa, segn las expresiones (11.13) o
(11.14), que las lneas se juntan un poco, para valores grandes de j, con
respecto al espaciamiento uniforme que se esperara para el caso del sistema
idealmente rgido.

Ejemplo desarrollado
Un espectro de absorcin de radiacin infrarroja por parte de una muestra gaseosa consiste en una
serie de varias lneas equiespaciadas, con una separacin de 1,15810 11 Hz .
a) Determine si lo que se observa puede ser un espectro de rotacin de CO, de NO, o de O 2 .
b) Si la parte ms intensa del espectro llega aproximadamente hasta la 7 ma u 8va lnea, desapareciendo
prcticamente el espectro algunas lneas ms all, determine aproximadamente la energa de rotacin
que es mayoritaria en las molculas de la muestra, y a travs de ella, estime la temperatura.
326

Desarrollo
a) Por la descripcin es claro que s puede ser un espectro de rotacin.
Ahora, a partir de la expresin (11.14), teniendo los valores de todas las frecuencias, podemos calcular
el momento de inercia de la molcula. Por ejemplo tomamos j = 0 f0 = 1,158 1011 Hz I =
1,45 10-46 kgm2.
Luego, a partir de las distancias interatmicas de la tabla calculamos los momentos de inercia de estas
molculas (I = d2), y obtenemos (en 10-46 kgm2): 1,45 para CO, 1,64 para NO, y 1,93 para O2.
Queda claro que lo que se observa puede ser un espectro de CO, y no de los otros compuestos.
b) Si las lneas alrededor de la 7 u 8 son fuertes, significa que esos niveles estn muy poblados, y el
hecho de que ms all la intensidad decaiga tambin significa que es pobre la poblacin de los niveles
correspondientes. Podemos esperar que el valor medio de la energa cintica de rotacin alrededor de
un eje, dado por kB T, est cercano a E8. Calculando E8 con (11.11) obtenemos E8 = 2,7 10-21 J, y de
all T alrededor de 200 K.

Rotacin y vibracin de molculas diatmicas.


Despus de introducir el tema estudiando un sistema de dos partculas que rota rgidamente,
sin posibilidades de vibracin a lo largo de la ligadura entre las partculas, estamos en
condiciones de considerar el caso real, en el cual la unin no se comporta rgidamente, y la
vibracin resulta combinada con la rotacin del sistema en diversos grados.
La energa total de un sistema que puede vibrar y rotar (no existe la posibilidad de la
traslacin porque se considera en reposo el centro de masa) es la suma de la energa cintica
de rotacin, ms la energa total (cintica ms potencial) correspondiente a la vibracin. Como
cada trmino de esta suma est cuantificado, la energa total estar determinada por los dos
nmeros cunticos independientes correspondientes:
k j ( j 1) 2
En,j = En + Ej = (n + ) + (11.15)
2 d2
Ahora bien, como ya hemos visto, el primer trmino es mucho ms grande que el segundo, el
cual en la prctica tiene el efecto de producir una pequea correccin al valor entregado por el
primero.
Si lo interpretamos en trminos de un esquema de niveles de energa, podremos imaginar que
cada nivel de energa vibracional dado por un nmero n, se ha transformado, o desdoblado
en un conjunto de niveles muy prximos, correspondientes a los diferentes valores de j que
son posibles.
327

E (J)
Ep(x)
Fig. 11.8:
x Esquema de
niveles y
E4 j=3 transiciones
j=2 posibles teniendo
E3 j=1
en cuenta rotacin
j=0
E2 y vibracin. Se ha
representado cmo
n=1 n = 1 n =1
E1
j =1 j =-1
la separacin entre
E0 n=0 j=3 niveles
j=2 vibracionales es
j=1 muy superior a la
j=0
de niveles
rotacionales.
Segn las reglas de seleccin impuestas por la mecnica cuntica, las transiciones permitidas
para emisin/absorcin de fotones son:
n = 0, 1, y j = 1 (11.16)
Tenemos as dos tipos de transiciones. Por una parte las transiciones sin cambio de estado
vibracional (n = 0), que corresponden a cambios solamente en el momento angular,
regulados por la regla de seleccin j = 1, y dan lugar al espectro de rotaciones que se
describi en el punto anterior, en el infrarrojo lejano.
Por otra parte, cuando hay cambio de estado vibracional, tenemos transiciones como las
indicadas en la figura, dadas por las reglas n = 1, y j = 1. Como se deduce fcilmente de
la figura, la energa absorbida o emitida en estas transiciones tiene aproximadamente el valor
correspondiente a la diferencia de energa entre niveles vibracionales, con pequeas
perturbaciones o correcciones dadas por el trmino indicativo de la variacin de estado
rotacional.
Es decir, si fv es la frecuencia correspondiente a la vibracin armnica pura, y fj son las
frecuencias correspondientes al espectro de rotacin pura, ahora se encuentra un espectro con
absorciones y emisiones en todas las frecuencias:
fv fj , con j = 0, 1, 2, 3, . . . (11.17)
Es decir, para cada frecuencia fj en la cual hay una lnea de absorcin en el espectro de
rotacin pura (en el infrarrojo lejano, como se dijo), hay dos lneas cerca de fv, que aparecen
como alteraciones o desdoblamientos del espectro de vibracin: una en fv + fj , y otra en fv fj
.
De manera que, como se muestra en la figura, en la vecindad de la frecuencia fv, se encuentra
todo el espectro de rotacin dos veces, una a cada lado del valor fv. La lnea situada
exactamente en fv , en cambio, paradjicamente, ha desaparecido, ya que correspondera a una
transicin n = 1, j = 0, que no est permitida.
328

Intensidad

f
fv

Fig. 11.9: Forma general del espectro de vibraciones combinadas con rotaciones.
Aplicando un poco de deduccin mezclada con un poco de imaginacin podemos decir que el
hecho de que la separacin entre niveles de vibracin sea mucho mayor que entre niveles de
rotacin nos muestra que en un ambiente suficientemente fro una molcula puede recibir por
los inevitables choques, energas que le alcanzan para cambiar su estado de rotacin pero no
para sacarlo de su estado fundamental de vibracin, con lo cual su comportamiento
corresponde muy bien al del rotador rgido que se ha propuesto como modelo inicialmente
para las rotaciones. Por ejemplo, en la molcula N2, la energa para saltar de nivel vibracional
(510-20 J) sera aproximadamente 10 veces ms grande que la energa cintica media de
cualquier grado de libertad a temperatura ambiente (410-21J), y a su vez esta ltima sera 10
veces ms grande que la energa necesaria para cambiar entre los primeros modos rotacionales
(10-22 J). Esto significa claramente que a temperatura ambiente las molculas de N2 viajan
chocando y cambiando caticamente tanto la orientacin como la rapidez de la traslacin y la
rotacin, pero sin comenzar a vibrar (ms all de la vibracin fundamental que no puede
eliminarse).
329

CAPTULO 12:
Elementos de mecnica
aplicados a fluidos
Los fluidos son los lquidos y los gases, y aunque su movimiento, como el de todo lo
material, cae dentro de la incumbencia de las Leyes de la Mecnica, en la prctica
muestra aspectos extremadamente sorprendentes para el que est habituado a tratar con el
movimiento de partculas y cuerpos slidos.
Por esto es que para el estudio de los fluidos se ha desarrollado un captulo especial de la
Mecnica, denominado Mecnica de los Fluidos.
Dado que la Mecnica de los Fluidos en general necesita recurrir a un aparato
matemtico del que no podramos disponer aqu, nuestra pretensin es solamente
completar un poco una visin general del comportamiento de la materia, asomndonos a
algunas perspectivas nuevas, y revisando algunos fenmenos simples que pueden
entenderse fcilmente con los elementos de mecnica que ya hemos visto.
Dentro de esas limitaciones veremos entonces esencialmente dos grandes temas:
En primera instancia algo de Hidrosttica, que es una aplicacin de los criterios de
equilibrio de fuerzas a fluidos en reposo, y luego algunos conceptos de Hidrodinmica,
sobre todo para redondear nociones sobre rozamiento en presencia de fluidos, y definir
conceptos generales sobre flujo, que es precisamente el comportamiento que da lugar al
nombre fluidos.

12.1.- Hidrosttica

Presin hidrosttica y Principio de PASCAL


Ya hemos comentado sobre las caractersticas bsicas de los esfuerzos que se desarrollan en
un fluido esttico, descriptos esencialmente por el llamado Principio de PASCAL (enunciado
por Blaise PASCAL -1623-1662).
Ahora bien, este principio es un enunciado anterior a las leyes de la dinmica, que puede
haber sido considerado un Principio mientras no existan leyes de las que se poda deducir,
sobre todo teniendo en cuenta que resume conocimientos antiqusimos sobre el
comportamiento de los fluidos.
330

Pero desde el punto de vista del hilo de conceptos que hemos seguido en este libro, debemos
pensar que las leyes de la dinmica deben ser suficientes para explicar el comportamiento de
los fluidos, sin necesidad del aludido principio, el cual tambin debe surgir como
consecuencia de ellas.
De manera que comenzaremos ocupndonos de mostrar cmo surge el enunciado de PASCAL
de las leyes que ya tenemos, es decir, para los fluidos estticos, de los conceptos de equilibrio
de fuerzas.

Una parte del principio de PASCAL enuncia que la presin acta siempre perpendicularmente a
cualquier superficie que est en contacto con el fluido, independientemente de su orientacin.
Actuar perpendicularmente significa que el fluido aplica, sobre cualquier superficie, fuerzas
que no pueden tener componente tangencial.

En la segunda parte de este captulo mostraremos que un fluido s puede aplicar fuerzas
tangenciales, pero slo cuando est fluyendo. De manera que ahora consideraremos eso
suficiente para enunciar que cuando un fluido alcanza su estado esttico, necesariamente han
desaparecido todas las fuerzas tangenciales.

Una vez aceptado esto, ahora podremos ver fcilmente, aplicando nociones de equilibrio de
fuerzas, que la imposibilidad de la existencia de tensiones tangenciales garantiza que la
presin se manifieste con igual valor independientemente de la orientacin de la superficie
sobre la que acta.

Consideremos para demostrar esta afirmacin un elemento de volumen pequeo dentro de un


fluido en reposo. Este elemento es prismtico y est delimitado por las superficies S, S1, y S2,
todas perpendiculares al plano de esta hoja (en la figura 12.1 se ve un tringulo, y la tercer
dimensin que completa el prisma es perpendicular a la hoja). Las superficies S1 y S2 son las
proyecciones de S sobre planos perpendiculares respectivamente a los ejes x e y. La figura
12.1 muestra estas superficies vistas desde el eje perpendicu,lar a la hoja, junto con las fuerzas
cuyo equilibrio vamos a plantear, ejercidas sobre ellas por el fluido exterior. Para que sea
lcito ignorar la accin de la gravedad, consideremos que el plano de la hoja es horizontal.
y
F2
S2 Fig. 15.1 : S, S1, y S2 delimitan una
F1 x porcin de fluido esttico. El fluido
Fx
exterior ejerce la fuerza F sobre S,

S1 F1 sobre S1, y F2 sobre S2. Fx y Fy
Fy
F S son las componentes de F .

La condicin de que estas fuerzas sean perpendiculares a las respectivas superficies se traduce

en que (como muestra la figura), F1 est alineada con el eje x , F2 est alineada con el eje y, y

el ngulo que forma F con el eje x , , es el mismo que forma S con el eje y.
Esto significa, por un lado, que Fx = F cos , Fy = F sen , S1 = S cos , y S2 = S sen, y por
otro, que el equilibrio se verifica cuando (en valor absoluto) F1 = Fx , y F2 = Fy.
Y si ahora planteamos el cociente de cada fuerza sobre su respectiva superficie, tendremos la
presin ejercida sobre cada una, y claro est, es el mismo valor para todas:
331

F1 F cos F
p1 p
S1 S cos S
F2 F sen F
p2 p
S2 S sen S
De este modo se ha mostrado cmo, para fluidos en equilibrio, la afirmacin de que la presin
debe actuar con igual valor independientemente de la orientacin de la superficie se puede
deducir de la afirmacin de que acta siempre perpendicularmente.
Por otra parte es interesante analizar el equilibrio para un elemento de volumen cilndrico de
gran longitud L, ubicado a lo largo del eje horizontal x (Fig. 12.2):

S1 x
S2
L
Fig. 12.2 : se representa un tubo de fluido, con dos tapas iguales, S1 y S2 , y
una longitud horizontal L tan grande como se quiera.

Supongamos que en el extremo izquierdo existe una presin p , y nos preguntamos si en el


extremo derecho, situado a una gran distancia L, necesariamente la presin tendr el mismo
valor.
Podemos imaginar que el volumen considerado est limitado por paredes reales (est dentro
de un cao de longitud L), o ideales (slo es una porcin seleccionada de un fluido que ocupa
una gran regin) - el resultado ser el mismo: al estar imposibilitada la existencia de fuerzas
tangenciales en ninguna pared, no puede haber fuerzas con componente x a travs de las
superficies laterales, y entonces necesariamente la fuerza horizontal en la tapa izquierda
deber ser equilibrada por una fuerza de exactamente igual valor en la tapa derecha, por
distante que est. Y esto es lo mismo que decir que las presiones deben ser iguales en ambos
extremos, ya que este volumen, por construccin tiene las dos tapas iguales.
De manera que entonces vemos que en un fluido esttico la presin se transmite de cada punto
del fluido a todos los puntos vecinos, y de stos a los otros, etc., llegando con igual valor a
todos los puntos del fluido del mismo nivel horizontal, por lejanos que estn.
Ntese que lo fundamental para esta afirmacin es la ausencia de fuerzas tangenciales. Con un
fluido que no estuviese perfectamente esttico, por ejemplo un fluido viajando por un largo
conducto, como veremos oportunamente, la fuerza de rozamiento tangencial a lo largo de las
paredes hara que llegue con una presin cada vez menor a los lugares ms distantes.
Por otra parte la accin de la gravedad, que hasta aqu hemos ignorado, es responsable de otra
caracterstica fundamental de la presin hidrosttica: la variacin con la profundidad, como
veremos a continuacin.

Teorema Fundamental de la Hidrosttica.


Imaginemos un lquido en reposo, y en su interior una porcin prismtica rectangular
separada del resto por superficies ideales, como se ilustra en la figura 12.3.
332

p1
ysup. libre
S1

Slateral
y1 h = y1 y2
P

S2
y2 p2
y=0
Fig. 12.3 : Cualquier porcin del lquido puede ser definida como un sistema cuyo
equilibrio se analiza. En este caso se considera un trozo prismtico. Las alturas y se
indican con respecto a un nivel elegido arbitrariamente. A la derecha se ilustran las
fuerzas actuantes sobre el cuerpo: en su centro se ha dibujado P , aplicada por el
campo gravitatorio, y sobre su superficie acta la presin del fluido exterior.

Sobre este prisma actan las fuerzas debidas a la presin del lquido exterior ejercidas sobre
los distintos lugares de toda la superficie, y adems acta el campo gravitatorio empujando al
prisma lquido con la fuerza peso hacia abajo.
Es claro que este prisma, por ser parte del mismo fluido que est en reposo, debe ser
considerado como un cuerpo sobre el cual todas las acciones exteriores se hallan en perfecto
equilibrio.
Entre estas acciones exteriores tenemos, por un lado, las fuerzas horizontales que aplica la
presin sobre las superficies laterales, que es fcil ver que se equilibran naturalmente entre s.
Y por otro lado tenemos las fuerzas que actan en sentido vertical, mostradas en la figura
12.4, que tambin deben estar equilibrndose.
F1
S1

P Fig. 11.4 : Diagrama de fuerzas verticales


S2 actuantes sobre una porcin de fluido
tambin vlido en general para un cuerpo
F2 cualquiera sumergido en el fluido.

F1 vale p1 S1 porque resulta de la accin de p1 sobre S1 , F2 vale p2 S2 porque resulta de p2


actuando sobre S2 , y es claro que ambas no deben equilibrarse, ya que F2 debe superar a F1 ,
en la medida necesaria para equilibrar el peso del prisma de lquido: P = p2 S2 p1 S1.
Como en valor S2 = S1 = S , llamando Pe al peso especfico, podemos escribir: P =
PeVolumen = Pe S h , y entonces queda:
Pe S h = p2 S p1 S
333

Y si con y indicamos altura con respecto a algn nivel (horizontal) arbitrario elegido como
referencia, entonces h = y1 y2 , tenemos finalmente:

p2 p1 = Pe ( y1 y2 ) (12.1)

expresin que se conoce como Teorema Fundamental de la Hidrosttica.

NOTA: INTERPRETANDO DETALLES DEL TEOREMA


a) La forma como la presin aumenta con la profundidad: p 2 = p1 + Pe h , da
cuenta, por unidad de superficie, de un hecho muy natural: la fuerza que el
prisma lquido considerado ejerce a travs de S2 sobre el lquido que queda
debajo de l es igual a la suma de la fuerza que el lquido encima del prisma
ejerce sobre l a travs de S1 , ms el peso del prisma:

F1 fuerza ejercida sobre el prisma
por el lquido encima de l
F2 F1 Pe Volumen
peso del prisma lquido
S S

P

b) La expresin (12.1), que fue deducida para dos puntos de la misma vertical,
es vlida en realidad para dos puntos cualesquiera en el seno de un fluido
homogneo, dado que, segn ya hemos mostrado, la presin hidrosttica es la
misma en dos puntos cualesquiera de igual profundidad, por muy distantes
que estn. En la figura siguiente se muestra un ejemplo de cmo se puede
escribir la presin en un punto a partir de la presin en otro, cuando se puede
hallar un camino formado solamente por tramos horizontales y verticales,
totalmente contenido en el mismo fluido.

A y 1 son puntosde la misma vertical , A ms alto :


pA = p1 Pe (yA y1 )

3 Pero p1 = p2 e y1 = y2,por ser 1 y 2 puntos del mismo


B nivel.
Entonces: pA = p2 Pe (yA y2 )

A Pero, por ser 2 y 3 puntos de la misma vertical,


p2 = p3 + Pe ( y3 y2 )
1 2 Entonces, sustituyendo esto en la expresin para pA:
pA = p3 + Pe (y3 y2 ) Pe (yA y2 )
= p3 + Pe (y3 yA )
y finalmente all sustituimos p3 por pB, e y3 por yB:

pA = pB + Pe ( yB yA )

Ahora tenemos derecho a decir que el Teorema Fundamental de la


Hidrosttica se puede plantear para dos puntos cualesquiera A y B en el seno de
un lquido homogneo esttico, tales que se pueda llegar de uno a otro por
algn camino que no salga de ese fluido en ningn punto.
334

c) Para nada importa si hablamos de un lquido o de un gas. Todo lo dicho se


aplica a ambos, vale para fluidos en general. La nica diferencia notable
entre lquidos y gases es que como en general el peso especfico de los gases es
mucho menor (alrededor de 1000 veces menor) que el de los lquidos, resulta
que la diferencia de presin en determinado desnivel dentro de un gas puede
ser despreciable comparada con la diferencia de presin en igual desnivel
dentro de un lquido. Por ello en muchos problemas prcticos se toma como
constante la presin del gas en todos los puntos, aunque tengan diferente
altura.
d) Este teorema completa la idea del Principio de PASCAL: los fluidos transmiten
la presin en todas direcciones, y en ausencia de gravedad la presin llegara
a ser la misma en todos los puntos. Pero como la gravedad existe, todas las
cosas pesan y la presin slo toma el mismo valor en todos los puntos de cada
plano horizontal, variando con la profundidad como hemos visto.

Aplicaciones directas del Teorema Fundamental de la hidrosttica

1. Manmetros en U - Unidades prcticas de presin


Se denomina manmetro al instrumento capaz de medir la presin de un fluido. Hay muchos
tipos de ellos, pero el que ahora nos interesa es el tipo U, que consiste esencialmente en un
tubo delgado y transparente de forma parecida precisamente a una U con cierta cantidad de
lquido en su interior. Existen dos variantes principales de este manmetro: abierto o cerrado,
segn se ilustra seguidamente (figura 12.5):
pa
pa pa
p1

p p

manmetro manmetro
abierto cerrado

Fig. 12.5 : ilustracin del empleo de un manmetro abierto y de uno cerrado para la
medicin de la presin p en el interior del baln.

La idea esencial (en cualquiera de las dos variantes) es que el lquido tiende a ubicarse con el
mismo nivel en las dos ramas de la U, pero se desnivela segn sea la diferencia de presin en
ellas. El lquido casi exclusivamente usado es el mercurio debido sobre todo a su gran peso
especfico.
En nuestros dos dibujos, en la rama izquierda se aplica la presin a medir, y en la rama
derecha est, o bien la presin atmosfrica en el modelo abierto, o bien la presin del gas que
queda encerrado en la rama cerrada (esta presin es variable y se la puede calcular).
Para calcular la presin a partir de estos instrumentos debemos medir el desnivel entre los
niveles del lquido en ambas ramas, y razonar segn el siguiente esquema (figura 12.6).
335

pA
A
p
yA h = yA yB

yB

Fig. 12.6 : Lecturas en un manmetro en U. Con y se indican


alturas con respecto a un nivel arbitrario

Consideremos A y B, puntos de cada una de las superficies libres que pueden ser conectados
por una lnea totalmente situada dentro del lquido, y a los cuales se puede por lo tanto aplicar:
pB pA = Pe (liq) ( yA yB )

Ahora bien, por encima de B hay un gas. Su peso especfico es despreciable y por lo tanto la
presin all no vara con la altura por encima de B: es la misma presin p del recipiente, la que
queremos medir. Por otra parte, por encima de A hay una presin que por ahora llamaremos
pA (y que en el caso del manmetro abierto es simplemente la presin atmosfrica pa ), de
manera que la diferencia pB pA = p pA resulta determinada simplemente en funcin del
desnivel h = (yA yB ) entre las ramas:
p pA = Pe(liq) h

NOTA: UNA AYUDA PARA LOS RAZONAMIENTOS


Resulta muy prctico a veces razonar trazando una lnea
horizontal por la superficie libre ms baja del lquido, es A
decir, en este caso por B ; esta lnea determina el punto B p
en la otra rama que, por tener el mismo nivel y estar
conectado con B sin interrupcin por el mismo lquido,
B B
tiene la misma presin:
pB = pB
Por otra parte, la presin en B, directamente debajo de A,
con una profundidad igual al desnivel h, se calcula
sumando a la presin en A, lo que corresponde a la
profundidad h en el lquido en cuestin:
pB = pB = pA + Pe h
(lq)

Las expresiones e ideas presentadas hasta aqu valen tanto para el manmetro abierto como
para el cerrado; la diferencia entre ambos slo aparecer cuando tratemos de expresar el valor
de la presin pA , lo cual es ms complicado para el manmetro cerrado, y no nos ocuparemos
de l en detalle. No obstante debe estar claro que el manmetro cerrado es ventajoso para
medir presiones grandes, y que la dificultad para expresar la presin en la rama cerrado puede
obviarse en la prctica, ya que el manmetro se puede graduar empricamente.
336

Veamos ahora especficamente, ms detalles del manmetro abierto:

pa
pa

p
h p pa = Pe (lq) h

Fig. 12.7: El manmetro abierto toma la atmsfera como referencia.

Se llama presin relativa a la atmsfera, (a veces tambin se la llama presin manomtrica) a


esta diferencia p pa = pr ; y as resulta:
pr = Pe h (12.2)
La llamada presin absoluta, p , queda determinada por:
p = pa + pr = pa + Pe h (12.3)
Como la presin indicada por estos manmetros resulta proporcional al desnivel h, resulta que
puede ser prctico tomar como unidad de presin el cm o el mm de desnivel de lquido; stas
no sern unidades S.I., claro est, pero son de uso muy cmodo y difundido.
As es que tenemos las importantes unidades:
mmHg : presin que desnivela 1 mm al mercurio en un manmetro abierto; (esta unidad
tambin se denomina torr , en memoria de Evangelista TORRICELLI (1608-1647), fsico
discpulo de GALILEO)
o lo que es igual:
x mmHg = presin (relativa a la que haya en la superficie) a x mm de profundidad en Hg .
cmHg : presin que desnivela 1 cm al mercurio en un manmetro abierto; o que hay a 1 cm
de profundidad en mercurio; etc.

Siendo la densidad del mercurio 13,604 kg/dm, resulta:


1 mmHg = 1 torr 13604 (kg/m) 9,8 (N/kg) 103 m
133,32 103(N/m3) 103 m = 133,3 N/m2
133,3 Pa

1 cmHg 1333 Pa
El mercurio es el lquido ms tpicamente usado, pero, claro est, podramos usar como
unidad de presin el mm o el cm de agua, o de cualquier lquido, definindolos segn la
misma idea.

Un lquido como el agua, de mucha menor densidad que el mercurio, se desnivela mucho ms
por la misma diferencia de presin, y permite por ello lecturas mucho ms sensibles. Adems
permite hacer fciles estimaciones con nmeros ms redondos:
337

1 mm agua = 1000 (kg/m) 9,8 (N/kg) 10- m


9,8 Pa 10 Pa
1 cm agua 100 Pa = 1 hectopascal

Ejemplo desarrollado
Considere un recipiente que contiene aire conectado al manmetro en U que se muestra. El lquido del
manmetro es mercurio, y la presin atmosfrica vale 740
patm = 740 mmHg
mmHg.
a) Calcule en mmHg la presin relativa en los puntos indicados pint A A
A, A, B, B, C, y C. B B 8 cm
b) Calcule la presin absoluta y la relativa del gas en el 10 cm
C C
depsito, expresadas en mmHg y en Pa. 12 cm
Desarrollo
a) Por lo que se ha comentado sobre el pequeo peso especfico de los gases, podemos considerar
que:
o La presin en A y en B, es la misma de la atmsfera.
o La presin en A, en B, y en C, es la misma del gas en el interior del depsito, la cual, en valor
relativo, est dada por la diferencia de nivel entre B y C: 100 mmHg.
o La presin en C es la misma que en C, por estar al mismo nivel dentro del mismo lquido.
De manera que la presin relativa en A y B es cero, y en A, B, C, y C, es 100 mmHg.
b) Para el gas en el depsito: pr = 100 mmHg = 13332 Pa, y pabs = 840 mmHg 111888 Pa. Si
quisiramos expresar con el nmero adecuado de cifras significativas, diramos p r 133 hPa, y pabs
112 kPa.

Barmetro de mercurio
Si pensamos en un manmetro en U cerrado, en cuyo extremo cerrado se haya hecho el vaco4,
tendremos un instrumento que mide la presin absoluta:

pa p1 0 (vaco)

p
h p p1 p
Pe (lq) h

Fig. 12.8 : Si p1 o , el desnivel h es indicador de la presin absoluta en el otro extremo.


Si el extremo izquierdo est abierto a la atmsfera, el aparato medir la presin atmosfrica
(figura 12.9). Este instrumento se denomina barmetro; su forma ms tpica es la mostrada
esquemticamente a la derecha de la misma figura.

4No se puede pretender tener un vaco absoluto sobre un lquido: inevitablemente sobre el lquido deber estar, al menos,
su propio vapor -y puede haber otros gases. Pero si el lquido es mercurio y todo anduvo bien, es decir que slo hay vapor
de mercurio, resultar que la presin de ste, a temperatura ambiente, es una pequea fraccin de mm que no alcanza a
alterar la lectura.
338

p1 0 (vaco)
h indica la presin
h atmosfrica en cmHg
(o mmHg) directamen-
pa te sobre la escala.
h pa Pe (lq) h
En la prctica resulta,
segn el lugar y el
pa estado del tiempo,
h 76 cm 4 5 cm

Fig 12.9: a) un manmetro cerrado aplicado a la medicin de la presin


atmosfrica se denomina barmetro. b) Forma tpica del barmetro de mercurio.

Se define la presin atmosfrica normal con un valor de 760 mmHg , valor que se considera
como el promedio anual a nivel del mar, y con este valor se define una unidad de presin
llamada atmsfera, que se abrevia atm (y que es muy prctica, aunque no es del SI,
claramente):
1 atm = 760 mmHg
760 103 m 13604 (kg/m) 9,8 (N/kg)
101325 Pa
1013, 25 hPa

NOTA:PRCTICA
Es interesante, entre otras cosas porque ilustra acerca de la idea de cmo acta
la presin atmosfrica, describir la forma prctica de lograr el vaco en el
extremo del tubo: simplemente se llena de mercurio un tubo (cerrado en un
extremo) de ms de 80 cm de largo, se lo tapa, se lo invierte, se introduce el
extremo tapado dentro del recipiente con mercurio, y se lo destapa, como se
muestra en la figura 12.10. Si como se dijo el largo del tubo es de alrededor de
80 cm o ms, la presin atmosfrica no podr sostener tanto mercurio, y ste
caer, dejando un vaco en la porcin superior del tubo hasta que se llegue
al equilibrio.

el mercurio
se
quita cae dejando
se se invierte columna de
se tapa el un vaco en
llena e introdu- mercurio que
tapn la parte
total- cuidando produce en A
ce en el
mente de que no L>80cm superior
mercurio una presin
queden igual a pa
del reci-
burbujas pa
piente
de aire A

Fig. 12.10: forma prctica de construir un barmetro de mercurio.


339

2. Principio de Arqumedes
Una consecuencia directa de las condiciones de equilibrio en el seno de un fluido esttico es el
enunciado de este conocido Principio:

Un cuerpo sumergido (parcial o totalmente) en el seno de un fluido en


reposo recibe un empuje de abajo hacia arriba igual al peso del fluido
desalojado por la parte sumergida del cuerpo.

Es muy claro cmo esto se desprende de lo que ya hemos dicho:


La presin del fluido siempre es mayor en la parte de abajo de un cuerpo sumergido que en
la de arriba la fuerza resultante sobre el cuerpo siempre ser hacia arriba, y de valor tal que
si el volumen que ocupa el cuerpo estuviera lleno del mismo fluido, ste estara en equilibrio
(su peso igualara a la resultante de la accin de las presiones).



E resultante de la accion
E de la presion sobre toda Pefluido Vsumergido
la superficie del cuerpo

Y como la fuerza aplicada por el fluido exterior sobre cualquier superficie no depende de lo
que hay dentro de sta, entonces resulta el enunciado tradicional.
A partir de estos enunciados queda claro que el problema del equilibrio de un cuerpo
sumergido en un lquido (o un gas: vale para cualquier fluido5) se reduce al anlisis de slo
dos fuerzas: el peso del cuerpo, y el empuje hidrosttico. Y se halla que pueden ocurrir
esencialmente tres cosas distintas, las que se suelen denominar casos de flotabilidad.

Casos de flotabilidad
1er Caso: Pe cuerpo < Pe fluido
Ilustraremos los enunciados con un cuerpo de madera de Vcuerpo 0,5 dm y Pecuerpo 6
N/dm3, sumergido en agua, es decir, Pefluido 10 N/dm3 .
En este caso, multiplicando ambos miembros por Vcuerpo , y recordando que si nos referimos a
un cuerpo totalmente sumergido tendremos que Vcuerpo = Vfluido desalojado , entonces queda:
Pecuerpo Vcuerpo = 3 N = P < Pefluido Vfluido desalojado = 5 N = E

Es decir que el empuje supera al peso. El cuerpo no permanecer sumergido por s mismo sino

que deber estar sujeto, y si se lo libera ascender impulsado por la resultante entre E y

P hasta quedar finalmente en equilibrio, flotando, sobresaliendo del lquido de manera de
recibir un empuje E igual a su peso.

5 El empuje es proporcional al Pe del fluido, y por ello suele ser poco perceptible en los gases. Claro que en enormes
volmenes de gas como los que se movilizan en la atmsfera, los efectos del empuje debido al menor peso especfico del
aire ms caliente, son muy notables: brisas, vientos, huracanes, etc. El empuje tambin es notable en volmenes mucho
menores que stos pero an as grandes, tales como los de los globos aerostticos.
340

3N
3N

5N 3N

2N
(a) (b)
Fig. 12.11. a) E > P, el cuerpo est sumergido porque se lo mantiene sujeto con un cable,
cuya tensin vale: F = E P = 2N . b) si se suelta el cable, como E > P, entonces el
cuerpo asciende hasta el estado final de equilibrio: E' = P = 3N. En esta condicin slo
tiene sumergida una parte de su volumen, Vs = E/Pefluido = 0,3 dm3.

2do Caso: Pecuerpo = Pelquido E = P


El cuerpo queda en equilibrio indiferente, a dos aguas. No flota ni se hunde.

Fig. 12.12 : cuerpo de igual densidad que el fluido flotando a dos aguas.
3er Caso: Pecuerpo > Pelquido E < P
Ejemplificamos con un cuerpo de igual volumen que el anterior, Vcuerpo= 0,5 dm, pero de
aluminio: Pecuerpo 26,5 N/dm, que se sumerge en agua.
En este caso el fluido no puede sostener al cuerpo, el cual se hunde hasta que se apoya en
algo.

5N
Fig. 11.13: cuerpo ms denso que el fluido:
el equilibrio slo puede ocurrir con la
intervencin de la fuerza de mdulo PE,
8N 13 N
aplicada en este caso por el fondo.

El peso del cuerpo es P = 0,5dm26,5N/dm 13 N; el empuje es el mismo que en el


ejercicio anterior, 5 N, pues sigue siendo el peso de los 5 dm3 de agua desalojada, y la fuerza
que aplica el cuerpo al fondo es la diferencia 13 5 8 N .
La fuerza de 8 N, con la que el fondo sostiene al cuerpo, igual por accin y reaccin a la que
el cuerpo aplica al fondo, resulta menor que el peso del mismo y a veces mucho menor, casi
nula. Debido a esto es que se tiene la sensacin de que los cuerpos pesan menos dentro del
agua.
341

Lo que en realidad sucede es que el campo gravitatorio aplica al cuerpo sumergido la misma
fuerza peso, P, que si estuviese fuera del lquido, pero hay que aplicar menos fuerza para
sostenerlo: F = P E , por lo cual el acto de pesarlo, entendido como sostenerlo y medir la
fuerza necesaria, nos dar un valor F < P, y nos tentar a decir que pesa menos. Pero slo es
el peso aparente6 el que disminuye dentro del lquido.

3. El Principio de los vasos comunicantes


Nos hallamos aqu nuevamente ante un enunciado denominado Principio que no es
formalmente tal cosa, pero que ha heredado esa denominacin y que no es importante
quitrsela:
Si varios recipientes con un mismo lquido estn abiertos, de modo que acta la
misma presin pa sobre la superficie del lquido en todos ellos, y estn
comunicados de manera que se puede pasar de uno de ellos a cualquier otro sin
salir del lquido, entonces el estado de equilibrio slo es posible con todas las
superficies libres en el mismo plano horizontal.
El lector seguramente puede obtener este enunciado directamente del Teorema Fundamental
de la Hidrosttica.

Fig. 12.14: Ilustracin del llamado Principio de los vasos comunicantes. Si sobre todas las superficies
libres acta la misma presin pa, y cualquier punto de una de ellas puede ser unido con cualquier punto
de otra por una lnea que no sale del lquido, entonces deben estar al mismo nivel. El tubo que comunica
el recipiente de ms a la derecha con el resto se denomina tubo sifn.

Ejemplo desarrollado 1
Un bloque de cierta madera pesa 90 N en el aire. Una pesa de plomo tiene un peso de 130 N
cuando est sumergida en agua. Cuando se atan juntos y se los sumerge en agua pesan 100 N.
Cul es la densidad de la madera? Explique cmo se interpreta en este enunciado la palabra peso
escrita entre comillas. Qu relacin guarda con peso = mg ?
Desarrollo.
Este enunciado slo se entiende si interpretamos que peso es lo que podramos llamar peso
aparente, que sera la fuerza que debera hacer un agente exterior para sostener al cuerpo
correspondiente en cada caso, es decir lo que indicara un dinammetro del cual colgase suspendido

6 Esto slo tiene el valor de una propuesta de vocabulario a usar. Si alguien desea decir que los cuerpos pesan menos
dentro del agua est en todo su derecho, pero no le est llamando peso a la fuerza gravitatoria ; lo que debe tener bien en
claro es que la fuerza que aplica el campo gravitatorio sobre el cuerpo no cambia en lo ms mnimo porque ste est
dentro del agua, y que lo que disminuye es la fuerza que el cuerpo aplica sobre el fondo (o sobre lo que sea que lo
sostiene ). Pensar que la fuerza gravitatoria sobre un cuerpo pudiera cambiar por estar ste sumergido, es decir rodeado de
agua, obligara tambin a pensar que cambia cuando el cuerpo es guardado en una caja o en algn otro recipiente
adecuado.
342

cada cuerpo en cada caso. Este peso, o peso aparente, sera entonces la diferencia entre la
atraccin gravitatoria y el empuje hidrosttico. Aunque no es imprescindible hacer un dibujo para
aclarar esto, consideramos que siempre se entiende mejor un problema de stos con una figura
esquemtica de por medio, motivo por el cual a continuacin resumimos el enunciado con los
correspondientes esquemas.

90 N 130 N 100 N

En el caso del bloque de madera, podemos despreciar el empuje hidrosttico del aire, por la pequea
densidad de ste. As que si m1 y V1 son la masa y el volumen del bloque de roble, y m2 y V2 los
correspondientes al cuerpo de plomo, podremos reescribir los datos de la siguiente manera:
m1 g 90 N
m2 g - Peagua V2 = 130 N
m1 g + m2 g - Peagua (V1 + V2) = 100 N
En el ltimo rengln podemos reemplazar m1 g por 90 N, y m2 g - Peagua V2 por 130 N, de esta
manera queda:
90 N + 130 N - Peagua V1 = 100 N ,
As que: Peagua V1 = g agua V1 = 120 N
Haciendo el cociente de el primer rengln sobre ste ltimo, finalmente queda:
m1 g 90 D = 1 = 0,75
1
V1 agua g 120 agua
El problema est terminado. No obstante en este tipo de problemas es fcil revisar lo que se ha
encontrado para ver si es correcto, de manera que procederemos a mostrar una forma posible de
revisar este problema.
La masa del bloque de madera se puede averiguar a partir del primer dato: m1 = 90/9,8 9,2 kg. Si su
densidad es 0,75, entonces su volumen deber ser 9,2/0,75 12,27 dm3, y el empuje que recibir al
estar sumergida ser de 12,27 kg, o en newtons, 120,2 N.
Por otro lado el cuerpo de plomo, sumergido, recibe un empuje que es 130 N menor que su peso. As
que con ambos cuerpos unidos sumergidos podremos decir que entre el peso y el empuje del cuerpo
de plomo el resultado es 130 N hacia abajo; si a esto le agregamos los 90 N que pesa la madera, y le
restamos los 120 N de empuje que ella recibe, tenemos la fuerza resultante total, de 100 N hacia
abajo, indicada por el dinammetro correspondiente.

Ejemplo desarrollado 2
El bloque A de la figura est suspendido de un dinammetro D y se encuentra sumergido en un lquido
(C), contenido en un recipiente (B). La masa del recipiente es 505 g y la del lquido 402 g. La balanza
(E) en la cual est apoyado el recipiente seala 1,137 kg. El dinammetro marca 2,150 kg. El volumen
del bloque A es 295 cm3.
343

a) Explique por qu el equilibrio de las fuerzas que actan sobre el


sistema B + C, puede plantearse con la expresin: D
PESOB + PESOC +empuje = 1,1379,8N/kg
(dibuje las fuerzas involucradas actuando sobre B+C)
2,15 kg
b) Calcule el empuje de esta expresin, y a partir de l la densidad
del lquido C.
C
c) Dibuje el diagrama de cuerpo libre del bloque A, mostrando las B A
fuerzas actuantes sobre l.
d) Qu indicar cada instrumento si se saca el bloque A fuera del
lquido? Justifique. kg 001,1370

e) Dado que tanto en el enunciado de este problema, como en el


punto d), la balanza indica la fuerza que el recipiente B ejerce sobre ella, y dado que en ambos casos
esa fuerza se compone de el peso de B, ms la fuerza vertical que el lquido C aplica contra el fondo
de B,
entonces:
por qu es distinto el valor que indica la balanza en los dos casos?
qu tiene de distinto el lquido que est en contacto con el fondo de B en cada caso? por qu?
Desarrollo
a) Denominemos (para simplificar tomemos todas las fuerzas en kgf, sobreentendiendo que 1 kgf = 9,8
N):
PA : peso del cuerpo A.
PB = 0,505 kgf : peso del recipiente
PC = 0,402 kgf : peso del lquido C
FD = 2,150 kgf : fuerza que indica el dinammetro
FB = 1,137 kgf : fuerza con que la balanza sostiene al sistema, indicada en el visor.
E : empuje hidrosttico del lquido sobre el cuerpo A.
En la siguiente figura se muestran las fuerzas esquemticamente (no se puede indicar bien dnde est
aplicada cada una, de manera que slo se intentan representar los sentidos).

FD

PA
PC

PB
FB

Las fuerzas exteriores sobre el sistema deben estar en equilibrio (el empuje hidrosttico sobre el
cuerpo A es una fuerza interior, y por ello no figura explcitamente aqu):
344

FB + FD - PA - PB - PC = 0 .
En esta expresin PA - E = FD podemos reemplazar PA - FD = E , y queda:
FB = E + PB + PC
b) Partimos de la expresin:
E = FB - PB - PC = 1,137 - 0,505 - 0,402 = 0,230 kgf
Como el empuje es el peso de un volumen de lquido igual al del cuerpo, 295 cm 3, entonces el peso
especfico del lquido y su densidad sern, respectivamente 0,23/0,295 = 0,78 kgf/dm3, y 0,78 kg/dm3.
c) En el esquema siguiente se muestran estas fuerzas con sus valores. El peso del cuerpo A se
obtiene sumando E + FD , como se desprende claramente de la condicin de equilibrio aplicada al
esquema.
FD (2,15 kgf)

E (0,23 kgf)

PA (2,38 kgf)
d) Eel dinammetro indicar el peso del bloque, 2,38 kgf. La balanza indicar P A+ PC = 0,402 + 0,505
= 0,907 kgf.
e) Cuando el cuerpo A est sumergido, el empuje que ste recibe del lquido, por accin y reaccin
tambin es aplicado por l sobre el lquido hacia abajo, y detectado por la balanza, que por ello indica
0,907 + 0,230 = 1,137 kgf. Ahora bien, cmo se explica esta fuerza distinta si se considera que sobre
el fondo del recipiente siempre acta el mismo lquido?
Pues una manera de justificarlo es diciendo que cuando el cuerpo A est sumergido, el nivel del lquido
es mayor, y con ello la presin sobre el fondo es tambin mayor. El volumen sube exactamente tanto
como si hubisemos agregado 0,230 kgf de lquido, ya que ese valor, el del empuje, es el peso del
lquido desalojado.

12.2.- Movimiento de fluidos

Consideraciones generales
Cuando intentamos hablar del movimiento de un fluido, encontramos que debemos hablar del
movimiento de cada punto del mismo, dado que los distintos puntos se mueven de distinta
forma, casi independientemente, unos de otros. Pero dado que sabemos que el fluido est
compuesto de partculas (molculas) con movimientos caticos independientes, ste es un
momento conveniente para discutir un poco el problema de decir a qu le vamos a llamar la
velocidad de un punto del fluido.
No tendra sentido considerar que un punto sea algo ms pequeo que una molcula, y
tampoco llamar velocidad de un punto a la velocidad de una molcula, porque sabemos, a
345

partir de la Teora Cintica de los Gases, que cada una se mueve caticamente con grandes
velocidades, an con el fluido en reposo.
El problema se resuelve desde dos puntos de vista, que en cierta forma son complementarios:
el punto de vista macroscpico, y el microscpico.

Descripciones macroscpica y microscpica


Desde el punto de vista macroscpico se considera el modelo continuo de la materia. En este
modelo se hace abstraccin de la estructura microscpica de la misma. Se la trata como si
fuese divisible hasta el infinito, y cuando se habla de un punto, se alude a un punto
estrictamente geomtrico.
Este punto es una idealizacin que no se pretende que refleje lo que realmente sucede en el
nivel microscpico, pero s que se mueva segn las leyes que se establecen
macroscpicamente, como haremos en este captulo.
Luego se hace intervenir el punto de vista microscpico, apoyado en algn modelo
corpuscular de la materia, para explicar o justificar la razn de ser de algunas leyes del mbito
macroscpico.

Por ejemplo la Ecuacin General de los Gases Ideales: p V = n R T, establece


fenomenolgicamente una relacin entre las variables macroscpicas p, V, y T, que describen
un fluido macroscpico, que podra ser continuo, y divisible hasta el infinito, o no, sin que
ello influya sobre lo que describen estas variables. Esta ecuacin permite resolver cuestiones
de termodinmica, sin necesidad de hacer intervenir consideracin alguna sobre la estructura
microscpica.
Pero es necesario que cuando se haga intervenir algn modelo
de la estructura microscpica del gas se obtenga algo que sea EL MODELO CORPUSCULAR
coherente con lo que se tiene macroscpicamente. Por ejemplo Para los gases ideales el modelo
corpuscular es el modelo cintico
en el caso del gas ideal, con la Teora Cintica de los Gases se
molecular presentado en la
obtiene la misma ley, siempre que interpretemos la variable T Teora Cintica de los Gases, para
de determinada manera. Eso nos permite explicar la razn de los lquidos tendramos un
ser de la ley (la cual no tendra ninguna justificacin terica en modelo parecido, pero que
el marco macroscpico), y ganar en comprensin sobre el contempla las interacciones entre
significado microscpico de la temperatura, lo cual nos sirve molculas, debido a que en este
para extender la teora a la explicacin de otros fenmenos. caso se mantienen prximas, y
para los slidos tenemos todas las
De manera que ahora tenemos el problema de definir a qu se teoras que describen las diversas
le puede llamar punto desde el punto de vista microscpico posibilidades de unin de los
(teniendo en cuenta el modelo corpuscular de los fluidos), de tomos entre s.
manera que sea coherente con la teora macroscpica que
estamos desarrollando.

El punto en el modelo corpuscular de la materia.


Teniendo en cuenta el modelo cintico molecular, definimos un punto de un fluido como una
regin del fluido tal que:
o Es tan pequea que puede ser considerada un punto en comparacin con cualquier
parte macroscpica que se considere del sistema.
346

o Es suficientemente grande como para contener un nmero N muy grande de


molculas. Este nmero tiene que ser tan grande que en cualquier promedio que se
haga de cualquier variable sobre las N molculas, no se deben poder detectar las
variaciones debidas a las molculas individuales que continuamente estn saliendo y
entrando de la regin.

De esta manera, el punto del fluido ser una pequesima regin con un enorme nmero de
molculas que se mantienen viajando juntas durante cierto tiempo, aunque continuamente las
que estn cerca del borde estn intercambindose con las del exterior. Luego de suficiente
tiempo, es probable que todas las molculas originales ya se hayan ido del punto, lo cual no
tiene ninguna importancia.

Velocidad de flujo.
Cada una de la N molculas en un punto P tiene un movimiento catico, con una velocidad

v i , cuyo mdulo depende de la temperatura de la manera similar a la que ya hemos visto en la

Teora Cintica de los Gases, y definimos el vector velocidad del punto, v P , o velocidad de
flujo, como el vector velocidad media del conjunto:
1
v P vi (12.4)
N
NOTA:
Si todas las molculas tienen la misma masa m, multiplicando ambos
miembros de (12.4) por Nm, que es la masa total de las molculas del punto,
tenemos:

mtotal vP m vi pi = vector cantidad de movimiento total

Esto nos dice que v P es lo mismo que la velocidad del centro de masa de todas
las molculas en la regin que hemos llamado punto (recordar que

p total mtotalvCM ).

Regmenes de flujo
Bsicamente hay dos regmenes de flujo, denominados flujo laminar y flujo turbulento, y en
este captulo slo desarrollaremos algunos aspectos simples del flujo laminar.
En el rgimen laminar cada punto sigue una lnea denominada lnea de flujo, la cual es estable
en el tiempo. Por cada punto del sistema pasa una nica lnea de flujo, la cual permanece fija
durante intervalos considerables de tiempo. El vector velocidad (de flujo) en cada punto es
tangente a la lnea de flujo que pasa por l.
En el rgimen turbulento, como su nombre lo indica, el vector velocidad en cada punto vara
desordenamente, y el punto no puede describir una lnea continua. Las lneas de flujo se
entremezclan en torbellinos que varan continuamente sin permanecer en el tiempo.
347

Fig. 12.15: Lneas de flujo correspondientes a un cilindro o una esfera fijos en


medio de una corriente de fluido. A la izquierda flujo laminar, y a la derecha
turbulento.

Nmero de Reynolds
Para que el flujo sea laminar es necesario que la velocidad sea suficientemente baja, los
conductos delgados y la viscosidad grande (la viscosidad es el rozamiento fluido que
definiremos ms adelante). La turbulencia tiende a producirse cuando la inercia del
movimiento predomina sobre el rozamiento (y viceversa) y el criterio en general para decidir
si el flujo es de un tipo o de otro depende de un coeficiente denominado Nmero de Reynolds,
N R:
vd
NR = (12.5)

Donde:
es la densidad del fluido
v la velocidad de flujo
el coeficiente de viscosidad, que definiremos ms adelante
d es alguna distancia transversal caracterstica del proceso (en una caera, por
ejemplo, es el dimetro, y en el ala de un avin, el grosor)
Se verifica fcilmente (haciendo uso de las unidades de : Pas) que NR es un nmero
adimensional, y el rgimen de flujo se decide de la siguiente manera:
o NR < 2000 Flujo Laminar
o 2000 < NR < 3000 Flujo Inestable (puede ser laminar, y pasar espontneamente a
turbulento)
o NR > 3000 Flujo Turbulento

Adems este nmero, que debe su nombre a Osborne REYNOLDS (1842-1912), sirve en
general para poder comparar situaciones que involucren movimientos de fluidos
aparentemente muy distintas.
Por ejemplo pensemos en el problema de disear un nuevo avin que deber volar a
centenares de kilmetros por hora. Las pruebas inicialmente se hacen sobre una maqueta en
tamao reducido del avin, dentro del tnel de viento. Qu criterios deben utilizarse para
decidir si algo que ocurra con la maqueta tambin debera ocurrirle al avin? Si la maqueta es
30 veces ms pequea que el avin, debe sometrsela a ensayos con mayor, o menor
velocidad del aire que la que encontrar el avin? Concretamente con qu velocidad del aire
deben hacerse los ensayos? El nmero de Reynolds es uno de los elementos claves a tener en
cuenta para responder estos interrogantes: la velocidad correcta para los ensayos es aquella
para la cual resulte el mismo nmero de Reynolds.
348

EL MUNDO DE LAS COSAS VISCOSAS


Las situaciones con bajos nmeros de Reynolds son notables: estn dominadas
por la viscosidad, y por ello la inercia de los cuerpos no se manifiesta ya que
todos los mviles se detienen en cuanto se deja de empujarlos, los flujos son
todos laminares pues los torbellinos no llegan a establecerse ya que disipan su
energa inmediatamente.

Movimiento estacionario de fluidos


Imaginemos un tanque de agua del cual sale un cao en cuyo extremo hay una canilla abierta
de manera que el agua fluye mientras el cao de alimentacin repone el agua del tanque,
manteniendo constante el nivel (los tanques tienen para ello un mecanismo que no hemos
dibujado). En la figura se ilustra la situacin, y se muestran adems varias secciones
transversales que servirn para sostener algunas discusiones en algunos ejemplos.
S2

S1
S3 Llave de paso abierta

S4 S5 S6 S7
Fig. 15.16
Si se cumple el requisito de que se mantenga constante el nivel en el tanque, entonces
estaremos en la situacin muy importante de rgimen estacionario, de la cual nos ocuparemos
ahora en ms detalle.

Decimos que un proceso ocurre con rgimen estacionario cuando hay flujos
de determinadas sustancias a ritmo constante, sin que vare ninguno de los
parmetros que definen el estado del sistema.

Caudal
Llamaremos caudal, o flujo, Q, en una seccin de un conducto de un fluido, a la cantidad del
fluido que atraviesa dicha seccin por unidad de tiempo.
Su pongamos que en el caso de la figura 12.16 el caudal es de 0,2 litros/segundo. Esto
significa que si imaginamos cualquier superficie S que seccione (idealmente) al conducto en
cualquier lugar (incluso puede ser que lo haga oblicuamente, o que S sea curva, con tal de que
seccione completamente al conducto), entonces S ser atravesada por 0,2 litros en cada
segundo. En la figura se indican varias secciones S para las cuales valdra lo dicho.
Segn la conveniencia prctica podemos definir el caudal volumtrico, o el caudal msico:
volumen a traves de S
QV (12.6)
t
masa a traves de S
Qm (12.7)
t
Las unidades de QV son m/s, L/s, cm/s, etc., mientras que las de Qm son kg/s , g/s , etc.
349

Para nuestro ejemplo:


Qv = 0,2 L/s = 2104 m/s = 200 cm3/s = 12 L/min, etc.
Qm = 200 g/s = 0,2 kg/s = 12 kg/min
Est claro que la relacin entre ambos debe ser la misma que entre masa y volumen:
Qm = densidad Qv (12.8)

Conservacin de la masa y ecuacin de continuidad


Una forma de indicar la conservacin de la masa es a travs de la llamada "Ecuacin de
continuidad", que presentamos a continuacin con ayuda de un esquema simblico.
Si con el rayado representamos toda la
S
materia, S es una superficie que delimita m(S)
nuestro sistema, y m(S) es toda la masa en materia
nuestro sistema, es decir, en el interior de S,
entonces:
m(S) puede variar slo a condicin de que lo
que aumenta o disminuye debe ser, a cada flujo positivo fujo negativo
instante, exactamente igual al total de lo que
ingresa menos lo que egresa fluyendo a travs de S:
m(S)
Qm(total a travs de S) (12.9)
t
Expresin en la cual el flujo de masa o caudal msico, se debe tomar en este caso particular
con signo positivo si el flujo es hacia el interior y viceversa (en general suele usarse la
convencin opuesta a sta, pero aqu eso no nos interesa).
Esta ecuacin implica que la ley de conservacin de la masa no se enuncia necesariamente
diciendo que m no vara, sino tambin indicando que su variacin debe relacionarse
adecuadamente con el flujo a travs de las superficies fronteras del sistema.
El caso de rgimen estacionario
Si aplicamos la ecuacin de continuidad a dos secciones Sa y Sb cualesquiera, la definicin
de rgimen estacionario significa que m = 0 en cualquier intervalo de tiempo que se
considere, y por ello:
Qm(b) Qm(a) = m = 0 ,
t
o lo que es lo mismo,
Qm(b) = Qm(a)
Esto significa que en rgimen estacionario Qm tiene el mismo valor en todas las secciones, y
eso es consecuencia de un principio fundamental: la conservacin de la masa.
Si hay cambios de densidad en el fluido a medida que avanza por el conducto (como puede
ocurrir con los gases si hay distinta presin y/o temperatura en los distintos lugares) ambos
caudales sern conceptos diferentes: en general tendremos Qm igual en todas las secciones
(por estar en estado estacionario), en contraste con QV que ser mayor donde la densidad sea
menor y viceversa.
350

Con los lquidos, aproximadamente incompresibles, la densidad se mantiene


aproximadamente constante, y se puede razonar equivalentemente con Qm o QV.

Seccin transversal y velocidad


Hay una relacin simple entre el caudal, la velocidad del fluido, y la seccin transversal. En la
figura 12.17 se muestra un trozo de conducto, en el cual elegimos esta vez con especial
cuidado que la seccin sea exactamente transversal, es decir perpendicular al eje del mismo
(la denominamos S para indicar eso).
L

v
S

Fig. 12.17: Se muestra un trozo de conducto. Viajando con velocidad v, todo el


fluido que est dentro del trozo dibujado de longitud L, tardar t = L/v en pasar
por S.

Haciendo inicialmente la simplificacin de que el flujo se realiza con una velocidad de


mdulo v, uniforme en toda la seccin transversal, tenemos que L = v t es la distancia
recorrida por cualquier porcin de agua en el lapso t, y dado que el volumen de un cilindro
de base S y longitud L es S L, entonces S v t es el volumen de agua que pasa por S en
t. Dividiendo por t tenemos el caudal volumtrico:
Q = S v (12.10)

Esta expresin de gran valor prctico nos dice que el mismo caudal se puede conseguir en
conductos de distinta seccin, a condicin de que la velocidad sea mayor en las partes ms
estrechas y viceversa.
Para el caso en que la velocidad no tiene el mismo valor en todos los puntos de la seccin
transversal, entonces, como se muestra en la siguiente nota, la expresin (12.10) debe
utilizarse con la velocidad media vm del fluido en la seccin.

VELOCIDAD MEDIA EN LA SECCIN TRANSVERSAL


Si tenemos una velocidad diferente en cada punto de la seccin transversal,
podemos subdividir sta en N superficies iguales de extensin Si = S / N cada
una, suficientemente pequeas como para considerar que cada una de ellas la

velocidad es un vector v i cuyo mdulo tiene un valor constante.
vi Si
S vj
Sj

En condiciones de flujo laminar en un conducto rectilneo todos los vectores

v i sern paralelos al eje del conducto, y cada uno definir una lnea de flujo
(que a la vez ser un delgado tubo de flujo) para el cual valdr Qi = Si vi =
(S/N) vi.
La suma de todos los caudales correspondientes a cada tubito ser el caudal
total:
351

S 1
Q= Nv i S
N
vi
Dado que todas la superficies Si son iguales, podemos decir que (1/N)vi es el
mdulo de la velocidad media en la seccin, vm , y entonces podemos escribir:

Q = S vm (12.10)

Ejemplo desarrollado con reflexiones acerca del caudal:


Considere el tanque con la caera de la figura 12.16, en rgimen estacionario. Suponga que del cao
sale suficiente agua como para llenar por minuto exactamente 2 baldes de 15 L c/u. Suponga que todo
el cao tiene 2,5 cm de dimetro.
a) Considere las secciones mostradas, y discuta dnde son mayores y dnde menores el caudal y la
velocidad, realizando los clculos necesarios.
b) Discuta qu cambia si se cierra parcialmente la canilla.

Desarrollo
a) Comencemos calculando el caudal a travs de la seccin de salida S6: Q6 = 2 15L/1min = 30 L/min
= 0,5 L/s. Vamos a hablar siempre de caudal volumtrico.
Podemos conocer el valor del caudal en las dems secciones? S, debe valer exactamente lo mismo
en cualquiera de ellas, pues de lo contrario se estara acumulando o perdiendo agua en alguna parte, y
hemos dicho que estamos en rgimen estacionario.
Vayamos ms al detalle. Miremos un trozo cualquiera de caera, como el que queda entre S4, una
seccin rigurosamente plana y transversal, y S5 , una seccin oblicua y curva cualquiera:
v
S4
S5

Qu debera ocurrir si fuese Q4 > Q5 , por ejemplo Q4 = 0,6 L/s, y Q5 = 0,5 L/s?
Pues que eso significara que a la zona entre S4 y S5 entran 0,6 litros de agua en cada segundo, y slo
salen 0,5; o sea que debera aumentar constantemente la cantidad de agua que contiene. Eso es
imposible en este caso porque las paredes son rgidas y el lquido es incompresible. Si en cambio
estuvisemos hablando de flujo de un gas, compresible, la situacin sera posible, pero no estaramos
en rgimen estacionario, ya que estara variando, por ejemplo, la presin en esa regin que cada vez
contendra ms gas.
De manera que en rgimen estacionario todo lo que entra atravesando S4 en un lapso dado
cualquiera, debe salir por S5 en el mismo lapso. Y como el razonamiento vale para cualquier trozo de
caera, queda claro que en rgimen estacionario el caudal es el mismo en cualquiera de las secciones
posibles en cada instante dado, independientemente de que sean planas o curvas, exactamente
transversales u oblicuas.
Notar que es vlido escribir Q = S4 v4, pero no Q = S5 v5, ya que S5 no es una seccin transversal.
Qu podemos decir de la velocidad?
En cada seccin la velocidad media vale v = Q / S. Dado que el caudal es el mismo en todas las
secciones, la velocidad media de flujo resulta inversamente proporcional a la seccin transversal. Es
decir que tiene un valor en el cao de ingreso (Q/S 1), otro valor parecido en todas las secciones del
cao de salida (Q/S4), y un valor muy pequeo en el cuerpo del tanque (Q/S 3).
352

Por ejemplo si el dimetro (interior) del cao es de 2,5 cm, y el del tanque 60 cm, aplicando S = R2
obtendramos v4 = v5 = v 6 = 1,02 m/s, y v3 = 0,0018 m/s = 1,8 mm/s. Prcticamente no se advierte que
el agua se mueva dentro del tanque. En S 2 y S7 la velocidad y la seccin del chorro van variando
mientras el agua cae.
b) Supongamos que se cierra parcialmente la canilla, disminuyendo Q6 y Q7 que pasan a tener nuevos
valores Q6 = Q7 = 0,3 L/s.
Como la caera es rgida y el agua incompresible, en S 3 S4 y S5 el caudal deber ser en todo instante
igual a Q6 , por las mismas razones que analizamos antes. Pero entre S 1 y S3 el tanque tiene un
espacio variable para el agua, y adems es abierto. Qu pasar entonces?
Cada segundo entrarn en la regin entre S 1 y S3 , 0,2 litros ms de los que salen el nivel de agua
comenzar a aumentar no se mantiene constante la variable "altura del nivel del agua en el
depsito", lo cual significa que ya no estamos en rgimen estacionario.
Pero como ste es un sistema diseado para buscar automticamente el rgimen estacionario (merced
al sistema regulador del tanque), tenemos que el flotante subir y paulatinamente limitar el pasaje en
S1 hasta que finalmente Q1 = 0,3 L/s.
Se habr alcanzado entonces un nuevo estado estacionario.

Cuando se altera algo en un sistema, por ejemplo en este caso el


grado de apertura de la canilla, eso afecta a todo el sistema:
no slo a lo que est despus, sino tambin a lo que est antes.
El rgimen estacionario anterior a la modificacin se alterar,
dejar de ser estacionario por un cierto intervalo de tiempo, y
luego, si se dan las condiciones, se llegar a un nuevo rgimen
estacionario con otros valores de todas las variables.

Viscosidad
La viscosidad es la propiedad de los fluidos que expresa el grado de dificultad para fluir.
Llamamos fuerza viscosa a esa fuerza tangencial que se opone al deslizamiento:
ya sea de un fluido dentro de un conducto
ya sea de un cuerpo slido dentro de un fluido
ya sea de porciones (capas) de un fluido respecto de porciones adyacentes
ya sea en situaciones que resultan de combinar estas anteriores, como por ejemplo, la
friccin fluida entre dos superficies slidas que deslizan con interposicin de un
lubricante7.
Aplicando el Principio de Accin y Reaccin a estos ejemplos, tambin puede decirse que la
viscosidad es la fuerza tangencial por medio de la cual un fluido en movimiento tiende a:
arrastrar al conducto

7
Un aceite "pesado" es ejemplo de lquido muy viscoso, y tambin es un ejemplo del cuidado que hay que tener con las
palabras: el aceite "pesado", no es pesado, es viscoso; pero en el lenguaje comn se suele emplear el concepto de
"pesadez" para designar la dificultad de los movimientos, lo cual es un efecto de la viscosidad. Tambin se suele utilizar en
la vida diaria, "densidad", para esto mismo. No obstante, en la fsica, el empleo del vocablo "densidad" es para otra cosa
muy distinta, como ya hemos visto. Por ahora digamos solamente: el agua fluye con ms facilidad que el aceite a travs de
un orificio determinado, porque es menos viscosa; pero si ponemos agua y aceite en un mismo frasco, el agua ir al fondo,
porque es ms densa. La densidad, recordemos, mide la concentracin de la masa.
353

arrastrar a cuerpos interpuestos en su camino


arrastrar a porciones adyacentes del mismo fluido
etc.
Son distintas formas de decir lo mismo. Slo debe estar claro que de todas las fuerzas que
intervienen en estos ejemplos, la viscosidad es la parte tangencial.
La viscosidad es siempre responsable de la disipacin de la energa cuando hay movimiento
de cuerpos en el seno de fluidos.

Coeficiente de viscosidad
Para un lquido que fluye en un conducto en rgimen laminar (que es el nico caso que
veremos), podemos imaginar capas de fluido deslizando unas sobre otras de manera continua
y permanente. En general deberemos imaginar cada capa con velocidad levemente diferente
que la adyacente, y que la capa ms rpida aplica una fuerza hacia adelante a la ms lenta a
travs de la superficie (ideal) de separacin, y viceversa. Estas fuerzas son accin y reaccin, y
ambas desapareceran si una porcin no fuese ms rpida que la otra. Como es razonable
esperar, estas tensiones evidentemente tangenciales, denominadas tensiones viscosas, son
proporcionales a la diferencia de velocidad entre capas prximas del fluido (o entre el fluido y
la pared sobre la cual fluye).
y y
D D C C
y4 v4
y3 v3
y2 v2 E E
y1 v1
x x
A B

Fig. 12.18: (izquierda) El fluido puede ser imaginado subdividido en capas que viajan con distinta
velocidad cada una, aplicndose mutuamente fuerzas de arrastre, tangenciales, llamadas fuerzas
viscosas. Estas fuerzas, divididas por el rea de contacto entre las capas, definen las tensiones
viscosas.
(derecha) Un rectngulo ABCD se deforma continuamente, y en cierto t, se transforma en ABCD.
Para cualquier punto E, la deformacin es EE, proporcional a yE . El cociente EE/yE es independiente
del punto, y dividido por t da una cantidad proporcional a la tensin viscosa.
Ntese adems que el rea de ABCD es igual a la de ABCD; esto (multiplicando por la dimensin
perpendicular al dibujo) equivale a la afirmacin de que la deformacin tangencial no altera el volumen,
y tiene que ver con la propiedad caracterstica de los lquidos, de fluir conservando el volumen.

La figura 12.18 ilustra cmo se imagina la subdivisin en capas planas de distinta velocidad
cerca de la pared de un conducto en condiciones de rgimen laminar, y tambin muestra los
elementos necesarios para expresar el flujo como ejemplo de deformacin tangencial. Para
plantear una situacin ms simple se ha considerado un lquido que fluye en direccin x sobre
el fondo plano de un conducto. En la direccin perpendicular a la hoja (que sera el eje z), el
conducto y el fluido se extienden indefinidamente. De manera que la velocidad slo puede
variar en la direccin y que es perpendicular a la pared del conducto.

La figura 12.19 muestra las fuerzas aplicadas sobre una cualquiera de las capas, por las capas
vecinas. Dado que en un sistema estacionario todas las velocidades son constantes, todas las
354

fuerzas actuantes sobre cualquier porcin de fluido estn en equilibrio, y esto nos dice que las
fuerzas entre las distintas capas son todas iguales: la fuerza que la capa 4 aplica a la 3
tendiendo a arrastrarla, es igual a la que aplica la 2 sobre la uno, e igual a la que sta le aplica
a la pared, etc.

v4 v4
v3 v3
v2 v2
v1 v1
pared pared
Fig. 12.19: (Izquierda) Las flechas huecas son las fuerzas sobre la capa 3, ejercidas, hacia
adelante por la capa 4, ms rpida, y hacia atrs por la capa 2, ms lenta. Pegadas a ellas, las
flechas negras son las respectivas reacciones sobre esas capas 4 y 2. (Derecha) Lo mismo entre
la capa 1+2+3 y la pared. Todas las flechas blancas deben indicar siempre equilibrio ya que todas
las velocidades son constantes en el tiempo, por lo cual todas las fuerzas son de igual mdulo.

Teniendo en cuenta que capas ms alejadas tienen proporcionalmente ms diferencia de


velocidad entre s, es decir proporcionalmente ms desplazamiento relativo por unidad de
tiempo, la tensin tangencial (viscosa) deben ser proporcional, no a la diferencia de velocidad
entre las capas, sino a dicha diferencia de velocidad dividida por la distancia (transversal)
entre capas, cantidad que denominaremos gradiente transversal de velocidad de flujo:
v x
tensin viscosa proporcional a
y

Para cada fluido (y para cada temperatura) se define el coeficiente de viscosidad, , como la
constante de proporcionalidad planteada en esta expresin, es decir, como el cociente entre la
tensin viscosa y el gradiente transversal de la velocidad de flujo.

(12.11)
vx
y

Donde v = F/S es la tensin tangencial, o tensin viscosa, es decir la fuerza de arrastre por
unidad de superficie (S es la superficie de deslizamiento, no debe confundirse con seccin
transversal).

La unidad de viscosidad del SI, como se desprende de la definicin (12.11) es Pas = Ns/m2.

De la definicin podemos despejar la fuerza de arrastre, o fuerza viscosa:


v
Fv = S v = S x (12.12)
y

NOTA PRCTICA
La viscosidad de los lquidos disminuye con la temperatura, mientras que la de
los gases aumenta levemente con ella. La explicacin para ambas
caractersticas de la viscosidad debe buscarse en el modelo corpuscular de
cada uno.
355

La viscosidad de los lquidos en general vara tanto con la temperatura que en


las aplicaciones prcticas suele ser necesario buscar datos referidos a la
temperatura particular para la cual se trata cada cuestin.
Para aplicaciones prcticas puede utilizarse la expresin:
b
a exp
T
Con los valores:
Agua: a = 8,1510-7 Pa ; b = 2089 K
-6
Alcohol: a = 2,1110 Pa ; b = 1859 K
Puede verificarse que a temperatura ambiente, tanto para el alcohol como
para el agua, se obtiene del orden de 10 -3
Pa s.

Aplicacin a un conducto cilndrico


Veamos estas ideas aplicadas a un conducto cilndrico, con flujo laminar y estacionario en su
interior.
Podemos tomar S como la superficie interior del conducto, cada capa sera una cscara
cilndrica de radio r y espesor r, y la direccin perpendicular a las capas sera la direccin
radial. As tendramos que la velocidad de cada capa dependera de r (sera una funcin v(r)) y
el gradiente transversal de velocidad de flujo, v/r, podra tomarse como el valor absoluto de
la derivada de v respecto de r.

v
v0
p1 v p2

v(r) v

Fig. 12.20 : (Izquierda) Ilustracin de la subdivisin en capas de distinta velocidad adaptada a un


conducto cilndrico. (Derecha) Se indican las fuerzas distribuidas por unidad de rea en la seccin
izquierda (p1), en la seccin derecha (p2), en la superficie lateral (v).

Consideremos una porcin cilndrica cualquiera de fluido dentro del conducto en un instante
dado, como se muestra a la derecha en la figura 12.20. Aunque esta porcin es un cilindro slo
durante un instante, ya que luego se deforma porque las capas cercanas al eje van ms rpido
que las del borde, del hecho de que ninguna velocidad vara en el tiempo (el flujo es
estacionario), deducimos que la fuerza resultante sobre toda la porcin debe ser nula.
Es decir que las fuerzas hacia delante deben equilibrarse con las que actan hacia atrs (dado
que hablamos de un conducto horizontal, no es necesario considerar el peso, que ya sabemos
que es equilibrado por la accin vertical de la presin sobre la superficie lateral).
Ahora bien, para las fuerzas horizontales, tenemos:
o En la seccin izquierda, S1, acta una fuerza hacia la derecha, de mdulo F1 = p1 S1,
o En seccin la derecha, S2, acta una fuerza hacia la izquierda, de mdulo F2 = p2 S2,
o Y en la superficie lateral SL, actan las fuerzas de rozamiento, que son las viscosas, de
mdulo Fv = v SL.
Esto vale tanto para un cilindro cuyo radio es todo el radio del conducto, R, y para el cual la
fuerza lateral resulta aplicada por la pared del conducto sobre la ltima capa de fluido en
356

contacto con ella, como para cualquier cilindro de fluido que se considere dentro de ste, en
cuyo caso la fuerza lateral resulta ser la accin viscosa de la capa de fluido inmediatamente
exterior al cilindro de fluido considerardo.
Ahora bien la superficie lateral, para un cilindro de radio r y longitud L vale SL = 2 r L, y por
lo tanto Fv = 2 r L v = 2 r L v/r, de manera que, teniendo en cuenta que S1 = S2 =
r2, la condicin de equilibrio de fuerzas queda:
p1 r2 = p2 r2 + 2 r L v/r
Es decir:
v
(p1 p2) r = 2 L (12.13)
r
Podemos despejar de aqu:
v (p1 p 2 )
= r (12.14)
r 2L
Vemos que v/r es una funcin lineal de r, lo cual significa que v(r) es una funcin de
segundo grado. Por otra parte, dado que v(r) alcanza su mximo valor v0 en el centro (r = 0),
su expresin debe ser v(r) = v0 A r2, siendo A una constante positiva que habr que
determinar con la condicin de que v debe anularse en el borde (r = R).
Efectivamente, escribiendo en la expresin anterior v(R) = 0, se obtiene A = v0 / R2, y con
ello:
v0 2 r2
v(r) = v0 r = v0 1 2 (12.15)
R2 R
De donde (derivando) se obtiene:
dv 2 r v0
=
dr R2
Y por lo tanto la expresin (12.13) queda:
4 L v0
(p1 p2) = (12.16)
R2

Esto nos dice algunas cosas importantes para flujo laminar en un conducto horizontal:
o Para mantener el flujo estacionario es necesaria una diferencia de presin, p= p1
p2, proporcional a la longitud del conducto que se considere.
o El flujo es desde la presin mayor a la menor.
o La velocidad en el centro (y por tanto tambin la velocidad media de flujo, como
veremos) es proporcional a la diferencia de presin aplicada, e inversamente
proporcional a la viscosidad, como corresponde a la idea de viscosidad como
rozamiento.
o Al aumentar el dimetro del conducto, tenemos que la misma diferencia de presin
produce mayor velocidad de flujo, de manera que al multiplicar la velocidad por la
seccin transversal se obtiene que el caudal aumenta con una potencia de R mayor que
si fuera proporcional a la seccin.
357

Vemos que la presin del fluido disminuye proporcionalmente con la distancia


en el sentido del movimiento a lo largo de la caera segn: p2 = p1 4Lv0 /R2 .
De manera que dada una presin p1 en un extremo de una caera horizontal
de gran longitud L, la presin en el otro extremo slo podr ser igual si v=0 , es
decir si estamos en el caso esttico, tal como ya lo hemos discutido; pero si hay
flujo, la presin deber ser menor en el extremo hacia el cual se dirige el
fluido.

Ahora bien, en general interesa ms obtener expresiones en las que figure el caudal, y no la
velocidad. El problema para determinar el caudal es que la velocidad vara con r. Si
adoptamos la subdivisin del flujo en capas de radio r y espesor r, tenemos que cada una de
ellas, con una seccin transversal Sr = 2 r r, viajando con v(r), aporta un caudal Q = 2 r
v(r) r.
As resulta que Q = Q = 2 r v(r) r, y sustituyendo la expresin para v(r), queda una
integral a resolver:
R r2
Q = 2 r v0 1 2 dr
0
R
= v0 R2 = S v0 (12.17)
Si aplicamos (12.10), obtenemos algo bastante esperable:
vm = v0 (12.18)
Decimos que es esperable porque v es cero en el borde, y crece hasta v0 en el centro; se
espera que vm est entre ambos valores, y (12.18) lo confirma.

Finalmente sustituyendo (12.17) en (12.16), y despejando Q, tenemos la expresin de


POISEUILLE, de gran aplicacin prctica:
p R 4
Q= (12.19)
8L

Ejemplo desarrollado 1
Considere una manguera cilndrica rectilnea horizontal de 20 metros de longitud y 2 cm de dimetro
interior, uno de cuyos extremos est abierto a la atmsfera, mientras que por el otro se inyecta un flujo
continuo de agua con cierta presin (para el agua tomaremos 10-3 Pas).
a) Calcular la presin con que es necesario inyectar el agua, para que a la salida se puedan llenar 3
baldes de 10 litros en cada minuto.
b) Calcular la velocidad media, y la velocidad mxima (en el centro de la seccin) del agua en la
manguera en estas condiciones.
c) Calcular por cunto se multiplicara la velocidad media de flujo, y por cunto el caudal, si
manteniendo la presin y se duplicara el dimetro de la manguera.
358

Desarrollo
a) El caudal (volumtrico) del enunciado vale Q = 30 L/min = 0,50 L/s = 0,510 -3 m3/s. Con esto y los
dems datos podemos calcular la diferencia de presin:
8 Q L 8 0,50 103 (m3 / s) 20m 103 Pas
p = = 2546 Pa
R4 0,014 m4
Dado que en el extremo final de la manguera est la presin atmosfrica, el valor que hemos obtenido
representa la presin relativa.
b) La seccin transversal de la manguera vale S = 0,012 = 3,1410-4 m2 = 3,14 cm2; de manera que
vm = Q / S = 1,59 m/s, y v0 = 3,18 m/s.
c) Duplicar el dimetro es duplicar el radio, y segn (12.16) v0 es proporcional a R2, de manera que se
multiplicara por 4 (y lo mismo vm). En cambio Q, proporcional a R4, se multiplicara por 16.

Ejemplo desarrollado 2
Se desea reemplazar un cao de 5 cm de dimetro y 50 m de largo, por dos caos que se colocaran
paralelos, en el mismo lugar (con igual longitud), y de los cuales se pretende que aplicndoles la
misma diferencia de presin, conduzcan el mismo caudal (entre los dos) que el cao original.
Para ello estos nuevos caos deberan tener, cada uno:
a) 2,50 cm de dimetro
b) la mitad de la seccin transversal del cao original, para lo cual su dimetro debera ser: . . .
c) otro valor del dimetro - explique, calcule.
Elija la opcin correcta, y justifique realizando los clculos que correspondan.

Desarrollo
Si se pretende que aplicando determinada diferencia de presin circule determinado caudal, lo que
debe aplicarse es la frmula de Poiseuille (suponiendo la aproximacin de que el rgimen es laminar).
No hay ninguna razn para proponer que se conserve el rea transversal total; mucho menos para que
la suma de los dimetros mantenga el valor inicial del dimetro.
Es decir queremos que dos conductos iguales de longitud L y dimetro interior D transporten el mismo
caudal en total (2Q = Q) que uno solo de la misma longitud L, y dimetro interior D, cuando se los
somete a la misma diferencia de presin:
p R 4 p R 4
2
8 L 8 L
Simplificando obtenemos la condicin 2 R4 = R4, es decir:
R = (1 4 2 ) R = 0,84 R
Si comparamos con lo que hubiera sido la primera opcin, R = R = 0,5 R, vemos que ese dimetro
hubiera sido muy insuficiente, y an el propuesto por la segunda, S = S (R = 0,71 R), lo hubiera
sido, ya que vemos que los caos necesarios para repartir en dos el mismo caudal, totalizan una
seccin transversal mayor que la del cao original.

S-ar putea să vă placă și